SlideShare a Scribd company logo
1 of 327
Q. 1
In Zellweger syndrome, which of the following is absent?
1. ER
2. Golgi apparatus
3. Mitochondria
4. Peroxisomes
CORRECT
SOLUTION
Zellweger syndrome is a rare disease characterized by the absence of functional
peroxisomes.
 the long chain fatty acids (C26-C36) are not oxidized. They accumulate in tissues,
particularly in brain, liver and kidney. Hence the disorder is also known as
cerebrohepatorenal syndrome.
 This condition is apparent at birth and is characterized by profound neurologic
impairment, victims often dying within a year.
 Two closely related conditions are neonatal adrenoleukodystrophy and infantile Refsum
disease. Zellweger syndrome and these two conditions represent a spectrum of
overlapping features, with Zellweger syndrome being the most severe (many proteins
affected) and infantile Refsum disease the least severe
REFERENCE
Lippincotts-biochemistry-6th-edition Q NO 16.5
Q. 2
Ammonia causes depletion of which of the following in TCA cycle?
1. Oxaloacetate
2. Alpha-Keto glutarate
CORRECT
3. Fumarate
4. Malate
SOLUTION
In Hyperammonemia (i.e. excess amino groups in cells), a large amount of alpha keto glutarate
(intermediate of TCA) combine with an excess of ammonia in cells to form glutamate which
further give rise to glutamine by taking up more ammonia (see fig). This leads to depletion of
alpha-KG.
As alpha-KG is an intermediate of TCA cycle, so TCA cycle stops and the brain is affected as
brain is highly dependent on this aerobic pathway for energy.
So in case of hyper ammonia alpha-ketoglutarate & glutamate will decrease and glutamine will
increase.
REFERENCE
SATYANARAYANA BIOCHEMISTRY PG NO 340
Q. 3
Defect in Menke disease:
1. Lysyl hydroxylase
2. Lysyl oxidase
CORRECT
3. Prolyl hydroxylase
4. Prolyl oxidase
SOLUTION
MENKE DISEASE / kinky OR STEEL HAIR SYNDROME- deficiency of copper results
in defective crosslinking of collagen and elastin by the copper-dependent enzyme lysyl
oxidase.
 Defect in dietary absorption of copper
 Die with in 5 years of age
 Kinky hair and growth retardation.
 menke disease is X linked disease due to mutation in ATP 7 A gene. This gene is present
on Xp 12-13.
REFERENCE
Harper Illustrated biochemistry 31st edition PG NO 1459
Q. 4
Glutamine is increased in CSF, blood & urine in which defect:
1. Arginosuccinate synthetase
2. OTC
3. CPS-I
CORRECT
4. Arginase
SOLUTION
Defect in urea cycle
Defect Enzyme involved
Hyperammonemia type I Carbamoyl phosphate synthase I
Hyperammonemia type II Ornithine transcarbamoylase
Citrullinemia Arginosuccinate synthase
Arginosuccinic aciduria Arginosuccinase
Hyperarginemia Arginase
In all enzyme deficiency ammonia will increase but cps 1 is rate limiting enzyme so it will
increase ammonia most & and increase in ammonia lead to increase in glutamine.
NOTE: The only urea cycle disorder that is inherited X-linked recessively is hyperammonemia
type II.
REFERENCE
LIPPINCOTT 7TH /258
Q. 5
Amino acid required for conversion of norepinephrine to epinephrine:-
1. Lysine
2. Phenyl-alanine
3. Methionine
CORRECT
4. Tryptophan
SOLUTION
Norepinephrine to Epinephrine conversion requires SAM (S-Adenosyl Methionine). Enzyme
involved is Phenylethanolamine N-methyl transferase. Cortisol is the activator.
Synthesis of Catecholamines (Dopamine, NE and E) from Tyrosine.
SAM (S - adenosyl methionine) is synthesized from dietary methionine with the help of enzyme
METHIONINE ADENOSYL TRANSFERASE (MAT).
REFERENCE
SATYANARAYANA BIOCHEMISTRY PG NO350
Q. 6
Creatinine, NO & urea are synthesized from which amino acid?
1. Arginine
CORRECT
2. Citrulline
3. Aspartate
4. Glycine
SOLUTION
1. Creatinine – Three amino acid -glycine, arginine and methionine-are required for
creatine formation
FORMATION OF CREATININE
2. NO is synthesized from arginine by enzyme NOS (Nitric Oxide Synthase).
FORMATION OF NO
3. Urea is formed from arginine by enzyme arginase
UREA CYCLE
REFERENCE
SATYANARAYANA BIOCHEMISTRY PG NO 366,367,368
Q. 7
Which of the following is not a dietary fiber?
1. Inulin
2. Cellulose
3. Pectin
4. Gum
CORRECT
SOLUTION
Dietary Fibre
Insoluble Dietary Fibres Soluble Dietary Fibres
Excreted unchanged from the intestine. get converted to gel form, and then they are excreted
 Cellulose
 Hemicellulose
 Lignin
 Resistant starch
 Pectins
 Beta – glucans (β-D-glucose)
 Psyllium husk
 Inulin
 Resistant starch
 Most common dietary fibre is cellulose.
REFERENCE
Lipponcott 7th ed. pg. 365
Q. 8
Which of the following is not the source of cytosolic NADPH:-
1. Isocitrate dehydrogenase
2. ATP citrate lyase
CORRECT
3. Malic enzyme
4. G6PD
SOLUTION
SOURCE of Cytosolic NADPH
HMP pathway Malic enzyme Isocitrate dehydrogenase
 Main enzyme - G6PD
 alternative pathway to
glycolysis and TCA cycle for
the oxidation of glucose
 the main pathway for
biosynthesis of NADPH and
pentoses.
 Converts malate to
pyruvate
 NADPH and CO2 are
generated
 NADPH mainly
utilized for fatty acid
synthesis
 3 isoenzymes
 One, which uses
(NAD+) in
Mitochondria
 other two use
NADP+ in
Mitochondria and
cytosol
REFERENCE
SATYANARAYANA BIOCHEMISTRY PG NO 274
Q. 9
Which of the following is increased in LCAT deficiency?
1. HDL
CORRECT
2. LDL
3. VLDL
4. Chylomicron
SOLUTION
 LCAT(Lecithin Cholesterol Acyl Transferase,)
o Converts HDL cholesterol to HDL cholesterol ester by adding one fatty acid in
cholesterol.
 LCAT deficiency
o Proper HDL is not formed
o Spherical HDL ↓
o Nascent or discoidal HDL ↑
o Free cholesterol ↑ ( not getting incorporated in HDL)
o RBCs have increased cholesterol content and thus hemolysis occurs
Disease Norum’s disease Fish eye disease
LCAT
Complete
deficiency
Partial
deficiency
Hemolytic
Anemia
Severe Absent
End stage
renal disease
Present Absent
REFERENCE
harper's 31st edition PG NO 254
Q. 10
True regarding mitochondrial DNA is:
1. Linear double stranded
2. Inherited from mother
CORRECT
3. Low mutation rate
4. All respiratory proteins are synthesized within mitochondria itself
SOLUTION
The mitochondrial DNA (mtDNA) has structural and functional resemblances with
prokaryotic DNA
 mtDNA is circular in nature and contains about 16,000 nucleotide bases
 A vast majority of structural and functional proteins of the mitochondria are synthesized
in the cvtosol under the influence of nuclear DNA. However, certain proteins (around
13), most of them being the components of electron transport chain, are synthesized in
the mitochondria (e.g. cytochrome b of complex lll, two subunits of ATP synthase.
 mtDNA is inherited from the mother
 Mitochondrial DNA is subjected to high rate of mutations (about 10 times more than
nuclear DNA)
 Diseases related to mitochondrial DNA
o MELAS (mitochondrial encephalopathy lactic acidosis stroke like episode)
o Leber hereditary optic neuropathy
o Leigh syndrome
o Kearns sayre syndrome
o NARP syndrome
REFERENCE
SATYANARAYANA BIOCHEMISTRY p.g no 563
Q. 11
Protein not synthesized in liver:-
1. Albumin
2. Immunoglobulins
CORRECT
3. Plasma enzymes
4. Acute phase proteins
SOLUTION
Immunoglobulins – are synthesized from plasma cell.
Other all plasma protein are synthesized from liver
 They are glycoproteins
 Consist of two identical heavy (H) Chains and two identical light (L) chains
 Is a Y-shaped tetramer (H2L2)
 Humans have five classes of immunoglobulins-namely IgG, IgA, IgM, IgD and IgE-
containing the heavy chains ϒ, α, µ, 6 and ε, respectively.
REFERENCE
Harper’s 31st ed pg. 289
Q. 12
Vitamin A is stored in:-
1. Kupfer cells
2. Hepatocytes
3. Ito cells
CORRECT
4. Endothelial cells of liver
SOLUTION
HSCs (Hepatic Stellate Cells) are:
 Pericytes which exist in the space between parenchymal cells and liver sinusoidal
endothelial cells of the hepatic lobule.
 Also called Vitamin A-storing cells, Lipocytes, Interstitial cells, Fat-storing cells or
Ito cells, Perisinusoidal cells
 Store 50-80% of vitamin A in the whole body as Retinyl Palmitate.
Extra information:
Vitamin A –
 The fat-soluble vitamin A, as such is present only in foods of animal origin. However, its
provitamins carotenes are found in plants
 Retinol, retinal and retinoic acid are regarded as vitamers of vitamin A.
Function of vit A -
 Vitamin A is necessary for a variety of functions such as vision, proper growth and
differentiation, reproduction and maintenance of epithelial cells.
 Deficiency manifestations of the eyes – nyctalopia is one of the earliest symptoms of
vitamin A deficiency.
REFERENCE
Satyanarayana BIOCHEMISTRY BOOK PG NO 122
Q. 13
In Wilson disease there is less urinary excretion of:-
1. Phosphorous
2. Methyl-Histidine
CORRECT
3. Phosphotyrosine
4. Serine
SOLUTION
Wilson disease is a condition of excess copper in body:
 There exists a renal lesion in Wilson’s disease.
 Plasma amino acid levels are normal both qualitatively & quantitatively, both in fed &
fasting state.
 Aminoaciduria (specially lysine, threonine, cystine) occurs.
 But Taurine & 1 methyl-histidine & 3 methyl-histidine are less in urine.
This is a journal based very rare question, just cram it.
REFERENCE
Harpers Illustrated Biochemistry 31th Edition PG NO
Q. 14
Which type of cholesterol is present in gall stones?
1. Amorphous cholesterol dihydrate
2. Amorphous cholesterol monohydrate
3. Crystalline cholesterol dihydrate
4. Crystalline cholesterol monohydrate
CORRECT
SOLUTION
Gallstones are composed:
 Principally of cholesterol monohydrate crystals or
 Acid salt of calcium bilirubinate.
REFERENCE
Harper 31st/e pg. 597
Q. 15
A 20 year old alcoholic malnourished patient presented to hospital with respiratory distress. His
pulse was 112/minute. Patient had edema, hypertension, systolic murmur along the left sternal
edge. Bilateral murmur along the left sternal edge. Bilateral crepitations were felt in the lungs. A
diagnosis of congetsive high output cardiac failure was made. Which vitamin is deficient?
1. Vitamin B1
CORRECT
2. Vitamin C
3. Vitamin B2
4. Vitamin B 6
SOLUTION
vitamin B 1 – water soluble vitamin
 It has a specific coenzyme TPP (thiamine pyrophosphate ) this help in carbohydrate
metabolism .
 TPP Dependent enzyme
1. pyruvate dehydrogenase - conversion of pyruvate to acetyl CoA.
2. alpha -Ketoglutarate dehydrogenase – CTA CYCLE
3. Transketolas - (HMP shunt)
4. The branched chain ALPHA - keto acid dehydrogenase -: oxidative decarboxylation
pyrophosphate of branched chain amino acids .
 RDA ( Recommended dietary allowance)
o Adult – 1 – 1.5 mg/day
o Children – 0.7 – 1.2 mg /day
 The requirement increases in pregnancy and lactation (2 mg/ day), old age and
alcoholism.
 Deficiency of vitamin b 1 is known as beriberi
 In adults, two types of beri-beri
 WET BERI – BERI - This is characterized by edema of legs, face, trunk and serous
cavities.
o Dyspnea and palpitation are present
o The systolic blood pressure is elevated while diastolic is decreased
o Pulse is fast and bouncing
o Death is due to heart failure
 Dry beri-beri - This is associated with neurological manifestations resulting in
peripheral neuritis.
o edema is not seen
o muscle become week so patient has difficulty in walking.
 Infantile beri-beri
o infant born to mother who is suffering from thiamine deficiency
o Brest milk is contain low thiamine
o Sleeplessness , vomiting , convulsions
o Bouts of screaming
o resembles to abdominal colic
o These all symptoms are due cardiac dilation
o Death occur suddenly due to heart failure
 Wernickes Korsakoff syndrome
o Seen in chronic alcoholics
o insufficient intake or impaired intestinal absorption of thiamine will lead to this
syndrome
o Symptoms of wernickes encephalopathy - G – GLOBAL CONFUSION
O – OPHTHALMOPLEGIA
A – ATAXIA

o Korsakoff syndrome – chronic neurological complication
1.
1. Amnesia - mainly anterograde amnesia
2. Confabulations – false story and honest lying

o Thiamin Nutritional Status Can Be Assessed by Erythrocyte Transketolase
Activaity
REFERENCE
Satyanarayana biochemistry book pg no 135 &136
Q. 16
Type of collagen maximum in skin:-
1. Type I
CORRECT
2. Type II
3. Type III
4. Type IV
SOLUTION
Collagen - the most abundant protein in mammals contains 4-hydroxyproline and 5-
hydroxylysine.
Vitamin C plays the role of a coenzyme in hydroxylation of proline and lysine while
protocollagen is converted to collagen
 The hydroxylation reaction is catalysed by lysyl hydroxylase (for lysine) and prolyl
hydroxylase (for proline)
 This reaction is dependent on vitamin C, molecular oxygen and a-ketoglutarate
Type Distribution
I Noncartilaginous connective tissues, including bone, tendon, skin
II Cartilage, vitreous humor
III Extensible connective tissues, including skin, lung, vascular system
IV Basement membranes
REFERENCE
Harper Illustrated biochemistry 31st edition PG NO 1450
Q. 17
In type I diabetes, which of the following is true:-
1. Increased lipolysis
CORRECT
2. Decreased protein catabolism
3. Decreased hepatic glucose output
4. Increase glucose uptake
SOLUTION
diabetes mellitus 1
Metabolic change in DM
1. Hyperglycemia - Hyperglycemia is primarily due to reduced glucose uptake by tissues
and its increased production via gluconeogenesis and glycogenolysis.
2. ketoacidosis – due to increase in plasma free fatty acid
3. Hypertriglyceridemia – conversion of fatty acid into triacylglycerols
 The activity of the enzyme lipoprotein lipase is low in diabetic patients. Consequently,
the plasma levels of VLDL, chylomicrons and triacylglycerols are increased
4 protein metabolism – insulin increase protein synthesis. But in DM 1 NUSULIN deficiency is
present so protein synthesis is decrease and protein degradation is increase.
Function of Insulin
Metabolism Net effect
Crabohydrate metabolism
Glycolysis
Gluconeogenesis
Glycogenesis
Glycogenolysis
HMP shunt
Increased
Decreased
Increased
Decreased
Increased
Lipid metabolism
Lipogenesis
Lipolysis
Ketogenesis
Increased
Decreases
Decreased
Protein Metabolism
Protein synthases
Protein Degradation
Increased
Decreased
REFERENCE
Satyanarayana biochemistry book pg no 682
Q. 18
Amino acid linking Kreb's cycle & urea cycle:-
1. Aspartate
CORRECT
2. Fumarate
3. Alanine
4. Arginine
SOLUTION
Urea cycle andTCA Cycle is linked by three way
1. Fumarate in urea cycle is the most important integrating point with TCA cycle. Fumarate
is converted to malate and then to oxaloacetate in TCA cycle.
2. Oxaloacetate undergoes transamination to produce aspartate which enters urea cycle.
However, Fumarate is not an amino acid so amino acid linking Kreb's cycle and
Urea cycle is aspartate.
Fig- Kreb’s Bicycle
Extra information:
1. 12 ATPs are generated in the TCA cycle while 4 ATPs are utilized for UREA CYCLE.
2. The CO2 liberated in the TCA cycle (in the mitochondria) can be utilized in the urea
cycle
REFERENCE
HARPER 31ST/275
Q. 19
In case of LPL deficiency, which of the following will increase after a fat rich diet?
1. Chylomicron
CORRECT
2. HDL
3. Lipoprotein (a)
4. LDL
SOLUTION
Lipoprotein Lipase deficiency causes Familial Hyperchylomicronemia or Type I
Hyperlipoproteinemia. This enzyme breaks the TGs of Chylomicrons into Fatty Acids and
Glycerol.
Chylomicrons move throughout the body in blood and they are acted upon by enzyme
Lipoprotein Lipase. Hence In LPL deficiency, chylomicrons and TG will rise in plasma.
Extra Reading:
Hyperlipoproeinemia
Type
Increased plasma
lipoproteins
Increased plasma
lipid (most)
Probable metabolic
defect
I Chylomicrons Triacyglycerols
Deficiency of
lipoprotein lipase
IIa LDL Cholesterol
Deficiency of LDL
receptors
IIb LDL and VLDL
Triacylglycerols and
cholesterol
Overproductionn of
apo-B
III IDL
Triacylglycerols and
cholesterol
Abnormality in apo-E
IV VLDL Triacylglycerols Overproduction of TG
V
Chylomicrons and
VLDL
Triacylglycerols -
REFERENCE
Satyanarayana biochemistry book pg no 321
Q. 20
Which of the following is not a Ribozyme?
1. Transpeptidase
2. Ribonuclease
3. Peptidyl transferase
4. Poly A polymerase
CORRECT
SOLUTION
Ribozyme
 RNA having enzymatic activity
 E.g. Ribonuclease P, peptidyl transferase or transpeptidase & snRNA (small nuclear
RNA).
Poly A Polymerase
 Not a ribozyme.
 Adds poly A tail in mRNA.
REFERENCE
Biochemistry 5th Edition by Satyanarayana pg no 82
Q. 21
Retinitis pigmentosa patients doesn't have:-
1. DHA
CORRECT
2. Eicosa pentaenoic acid
3. Arachidonic acid
4. Timnodonic acid
SOLUTION
Not all, but many studies have found that patients with retinitis pigmentosa tend to have lower
blood levels of DHA (Docosa Hexaenoic Acid, ω3, 22:6), an omega -3 fatty acid found in the
photoreceptor cells.
ADDITIONAL READING ON DHA:
 Also known as Cervonic acid. This is having 22 C and 6 double bonds (Docosa means
22, Hexaenoic means 6 double bonds).
 These days health drinks (bournvita, horlicks) are fortified with DHA, as it is important
for the brain and retina development in infants and children.
 Decreased amounts lead to increase risk of Retinitis Pigmentosa.
 DHA is present in high concentration in sperms, retina, cerebral cortex.
 Constant source of DHA is breast milk.
 Synthesized to a limited extent from α-linolenic acid or obtained directly from fish oils.
REFERENCE
Harper Illustrated biochemistry 31st edition PG NO 553.
Q. 22
Which apolipoprotein is responsible for Alzheimer's disease:-
1. APOE3
2. APOE1
3. APOE4
CORRECT
4. APOE2
SOLUTION
Relative risk of Alzheimer's disease is maximum with Apo E4
Apo E Risk of Alzheimer's
E2, E2 0.6
E2, E4 2.6
E3, E4 3.2
E4, E4 14.9
REFERENCE
https://medlineplus.gov/alzheimersdisease.html
Q. 23
Which amino acid is used to synthesis Nitric oxide?
1. Glycine
2. Arginine
CORRECT
3. Tyrosine
4. Histidine
SOLUTION
Nitric oxide (NO) –
 Arginine is the substrate for the production of nitric oxide (NO)
 ENZYME - nitric oxide synthase
 Half life – about 5 sec.
 Functions –
o vasodilator and relaxation of smooth muscles
o regulation of blood flow and BP
o inhibitor of platelet aggregation
o neurotransmitter
o NO mediates the bactericidal actions of macrophages
o erection of penis
 Nitric oxide promotes the synthesis of cGMP.
Glycine –
1. Formation of purine ring (2)
2. Synthesis of glutathione (3)
3. Conjugation reactions – two reactions A - The bile acids-cholic acid and chenodeoxy
cholic acid-are conjugated with glycine B - glycine is important for detoxification of
benzoic acid
4. Heme synthesis
5. Biosynthesis of creatine
Histidine - generation of histamine
 This amino acid is oxidatively deaminated by histidase to urocanic acid, which
subsequently forms N-formiminoglutamate ([FIGlu]
 Individuals deficient in folic acid excrete increased amounts of FIGlu in the urine,
particularly after ingestion of a large dose of histidine. The FIGlu excretion test has been
used in diagnosing a deficiency of folic acid.
Tyrosine - Tyrosine is formed from phenylalanine by phenylalanine hydroxylase
 Biosynthesis of thyroid hormones ,catecholamines ,melanin
REFERENCE
SATYANARAYANA BIOCHEMISTRY PG NO 366
Q. 24
Type-I hyperlipoproteinemia is characterized by:-
1. Elevated LDL
2. Elevated lipoprotein lipase
3. Elevated cholesterol
4. Elevated triglycerides in plasma
CORRECT
SOLUTION
Hyperlipoproeinemia
Type
Increased plasma
lipoproteins
Increased plasma
lipid (most)
Probable metabolic
defect
I Chylomicrons Triacyglycerols
Deficiency of
lipoprotein lipase
IIa LDL Cholesterol
Deficiency of LDL
receptors
IIb LDL and VLDL
Triacylglycerols and
cholesterol
Overproductionn of
apo-B
III IDL
Triacylglycerols and
cholesterol
Abnormality in apo-E
IV VLDL Triacylglycerols Overproduction of TG
V
Chylomicrons and
VLDL
Triacylglycerols -
REFERENCE
SATYANARAYANA BIOCHEMISTRY PG NO 321
Q. 25
Gestational Trophoblastic neoplasm does not include:-
1. Choriocarcinoma
2. Placental site trophoblastic tumour
3. Invasive mole
4. Partial mole
CORRECT
SOLUTION
Benign trophoblastic conditions Gestational trophoblastic Neoplasms
1. Normal pregnancy
2. Vesicular mole
3. Abortion
1. Invasive mole
2. Choriocarcinoma
3. Placental site trophoblastic tumor
REFERENCE
WILLIAMS obstetrics 25th ed/p- 393
Q. 26
Which vaccine is contraindicated in pregnancy:-
1. Chicken pox
CORRECT
2. Rabies
3. Tetanus toxoid
4. Hepatitis B
SOLUTION
All live vaccines are contraindicated in pregnancy and chicken pox is live vaccine
Q. 27
45 years female with 3 months menorrhagia. USG showing 2 cm submucosal fibroid. First line
of management is:-
1. OCP for 3 months
2. Progesterone for 3 months
3. Endometrial sampling
CORRECT
4. Hysterectomy
SOLUTION
First line management always will be endometrial sampling ( curettage )to rule out . this is
polyps or cancerous part
Then treatment always be next step.
Q. 28
In low ovarian reserve, Anti Mullerian hormone level will be:-
1. <1
CORRECT
2. 1-4
3. >7
4. >10
SOLUTION
Ovarian Reserve < 1 is low , more than 2 is normal
Markers of a REDUCED ovarian reserve are:
 Increasing age, especially beyond 35
 Decreased ovarian volume
 Decrease estrogen
 Decrease Antral Follicular count
 High serum FSH, especially more than 15
 Reduced Inhibin, especially lesser than 45 pg/ml
 Low AMH - (SINGLE BEST TEST )
REFERENCE
Shaws Textbook of Gynecology 17e p54
Q. 29
Presenting diameter of full flexed head:-
1. Suboccipito-bregmatic diameter
CORRECT
2. Suboccipito-frontal diameter
3. Occipito-frontal diameter
4. Occipito-posterior position
SOLUTION
Important diameters of the skull
Important Diameters of the skull
Diameters
Measurement in
cm (inches)
Attitude of
the head
Presentation
Suboccipitobregmatic- extends from the nape of
the neck to the center of the bregma
9.5 cm (3¼")
Complete
flexion
Vertex
Subocipitofrontal - extends from the nape of the
neck to anterior end of the anterior fontanel or
center of the sinciput
10 cm (4")
Incomplete
flexion
Vertex
Occipitofrontal- extends from the occipital
eminence to the root of the nose (glabella)
11.5 cm (4½")
Marked
deflexion
Vertex
Mentovertical - extend from the midpoint of the
chin to the highest point on the sagittal suture
14 cm (5½")
Partial
extension
Brow
Submentovertical - extends from junction of
floor of the mouth and neck to the highest point
on the sagittal suture
11.5 cm (4½")
Incomplete
extension
Face
Submentobregmatic - extends from junction of
floor of the mouth and neck to the center of the
bregma
Complete
extension
Complete
extension
Face
Q. 30
When used for emergency contraception, What is the dose of ulipristal acetate?
1. 300mg
2. 30mg
CORRECT
3. 300ug
4. 30ug
SOLUTION
Emergency Contraceptives
Drug Dose
Levonorgestrel (POP) 1 dose 1.5 mg within 72 hrs
Copper IUDs ( NOT THE DOC ! ) Insertion within 5 days
Ulipristal acetate (SPRM) 30 mg orally within 120 hours
Ethinyl estradiol 50 µg + Norgestrel 0.25 mg (COC) 2 tab stat and 2 after 12 hour
Mifepristone RU 486 (PA) 25 to 100 mg single dose
ICUDs are the most effective methods, no doubt, but they can never be the DOC. Thats because
they require seeing a gynecologist take a prescription and then a specialist to insert the device.
Emergency contraception is based on the principle of 'over the counter' easy to use medication to
prevent a pregnancy and when there is unplanned intercourse, which can happen many times in a
year, a person cannot be expected to take an IUCD every time!
However, a LNG tablet can be easily, and effectively, be taken whenever there is unplanned
intercourse
* Please note: We are in the early part of year 2020 now. Mifepristone or Ullipristal may be
soon termed as the best drugs for EC. Please be on the look out for recent updates.
REFERENCE
inputs from Williams 25th Ed Contraception chapter
Q. 31
The major contribution of the amniotic fluid after 20 weeks of gestation:-
1. Ultrafiltrate and maternal plasma
2. Fetal urine
CORRECT
3. Fetal lung fluid
4. Fetal skin
SOLUTION
Amniotic fluid (Main Source) -
 First 12 weeks: Ultrafiltrate of Maternal plasma
 12-20 weeks: Transudate across fetal skin
 >20 weeks: Fetal urine
REFERENCE
William’s obstetrics 25th edition Page no: 188
Q. 32
Nuchal translucency in USG can be detected at ------weeks of gestation:-
1. 11-13
CORRECT
2. 18-20
3. 8-10
4. 20-22
SOLUTION
Nuchal translucency is a collection of fluid under the skin at the back of your baby's neck. The
amount of fluid is measured during a nuchal translucency (NT) ultrasound scan: between 11
weeks and 14 weeks of pregnancy.
Q. 33
Firsl line of treatment of mastitis in a lactating mother is:-
1. Cloxacillin
CORRECT
2. Cefazolin
3. Ceftriaxone
4. Ampicillin
SOLUTION
FIRST LINE TREATMENT of MASTITIS in lactating mother is cloxacillin
Doc is co-amoxiclav
REFERENCE
Bailey & love 27th e , page 864
Q. 34
Patient with recurrent abortion diagnosed to have antiphospholipid syndrome. What will be the
treatment:-
1. Aspirin only
2. Aspirin + Low molecular weight heparin
CORRECT
3. Aspirin + Low molecular weight heparin + Prednisolone
4. No treatment
SOLUTION
DOC for APLA - Warfarin (Teratogenic)
DOC for APLA in Pregnancy - Heparin + Aspirin
 Heparin to be added only when there is H/O Recurrent abortions or Thrombosis
 Otherwise we give only low dose Aspirin
REFERENCE
Williams obstetrics 25th Ed,p-1143-1146
Q. 35
A 60 year woman comes with 3rd degree uterine prolapse. What will be the management?
1. Vaginal hysterectomy with pelvic floor repair
CORRECT
2. Pelvic floor repair
3. Sacrospinous fixation
4. Pessary
SOLUTION
Uterine prolapse is classified using a four part grading system:
Grade1:Descent of the uterus to above the hymen
Grade2:Descent of the uterus to the hymen
Grade3:Descent of the uterus beyond the hymen
Grade4:Total prolapse
Treatment ,
 if 14year girl then do sling operation
 If 30year female then do Manchester/ fothergill operation
 If postmenopausal women do vaginal hysterectomy with pelvic floor repair (WARD
& MAYO)
REFERENCE
Williams obstetrics 25th edition
Q. 36
Day 20 of menstrual cycle falls under which phase?
1. Menstrual phase
2. Follicular phase
3. Ovulation phase
4. Luteal phase
CORRECT
SOLUTION
Menstrual Cycle
Cycle days 1-5 6-14 15-28
Endometrial phase Menstrual (bleeding) phase Proliferation phase Secretory phase
Ovarian phase Early follicular Late follicular Luteal
Estrogen/Progesterone Low Estrogen↑ Progesterone↑
Gonadotropins (FSH/LH Low FSH↑ LH↑
Q. 37
Chromosome number of partial hydatidiform mole most commonly is:-
1. 46 XX
2. 45 XO
3. 46 XXY
4. 69 XXX
CORRECT
SOLUTION
Refer to Q 16 in this section
REFERENCE
williams obstetrics 25th ed/p- 392
Q. 38
Vulvar atrophy and itching are treated by-
1. Estrogen ointment
CORRECT
2. Antihistamines
3. Tamoxifen
4. Oral antifungal treatment
SOLUTION
Vulvar atrophy and itching in menopausal age group are due to deficiency of estrogen that cause
dryness of vulva which causes itching.
Ideal treatment of this manifestation of post menopausal urogenital atrophy is some form of
Hormone Replacement. Topical estrogen is most effective with minimal systemic side effects
REFERENCE
Shaws Textbook of Gynecology 17e p 374
Q. 39
PGF2 alpha maximum dose in PPH management which can be given over 24 hours is-
1. 250 μg
2. 200 μg
3. 2 mg
CORRECT
4. 20 mg
SOLUTION
PGF2∝ is CARBOPROST
250 µg I.M. up to 8 doses and maximum dose is 2 mg
given I.M. always ( i.v. can cause sudden hypertension )
REFERENCE
Williams Obstetrics 25th ed/p-760
Q. 40
Which of the following is false as physiological change in pregnancy?
1. Increase cardiac output
2. Increase total protein
3. Increase residual volume
CORRECT
4. Increase GFR
SOLUTION
In Pregnancy, diaphragm pushed upwards by enlarged Uterus thus reducing Residual volume
of lungs.
 Diaphragm is elevated by around 4cm
 subcostal angle increases from 68º to 104º.
 Transverse diameterof chest increases by 2 cm
Changes in Respiratory System
Parameters Non pregnant Pregnancy near term Change
Respiratory rate/min 15 15 unaffected
Vital capacity (mL) 3200 3300 Almost unaltered
Tidal volume (mL) 500 700 + 40%
Residual volume (mL) 965 765 - 20%
Inspiration capacity (IC) 2500 2650 + 10%
Minute ventilation 7.5 L/min 10.5 L/min + 40%
Total lung capacity (mL) 4200 4000 - 5%( almost unchanged)
REFERENCE
Williams obstetrics 25th Ed,p-151
Q. 41
MgSO4 have no role in prevention of-
1. Seizures in severe pre-eclampsia
2. Recurrent seizures in eclampsia
3. RDS in premature baby
CORRECT
4. Neuroprotection
SOLUTION
MgSO4 is the Drug of choice which prevents seizures in severe pre ecclapmsia and also for
seizure control in eclampsia
This is also used for control of recurrent seizures
Neuroprotective action: low-birthweight neonates whose mothers were treated with magnesium
sulfate for preterm labor or preeclampsia were found to have a reduced incidence of cerebral
palsy at 3 years
REFERENCE
williams 25th ed/p-736-737
Q. 42
Green frothy vaginal discharge is produced by –
1. Herpes simplex
2. Candida albicans
3. Trichomonas vaginalis
CORRECT
4. Normal vaginal flora
SOLUTION
Greenish frothy vaginal discharged is produced by trichomonas vaginalis , also seen strawberry
vagina
White curdy discharged is produced by candida albicans
Q. 43
Which of the following is an absolute CONTRAINDICATION to OCP use:
1. Chronic renal disease
2. DVT
CORRECT
3. Diabetes mellitus
4. History of amenorrhea
SOLUTION
OCP use is absolute CONTRAINDICATION IN DVT
Q. 44
Which of the following statement is correct about acute fatty liver of pregnancy?
1. Occurs in 1 in 1000 pregnancy
2. Mostly seen in last trimester
CORRECT
3. Common if female fetus is present
4. May be associated with decreased uric acid
SOLUTION
OCCUR in 1 in 10,000 pregnancy
Mostly seen in last trimester 37-38 weeks
Common if male fetus is present
Q. 45
Female with 41 wk gestation confirmed by radiological investigation, very sure of her LMP, no
uterine contractions, no effacement and no dilatation. What should not be done?
1. Intracervical foley’s
2. PGE1 tab
3. PGE2 gel
4. PGF2alpha
CORRECT
SOLUTION
We can give misoprostol, Dinoproston , intracervical foley’s
PGF2 alpha – carbo prost don’t give here {this is used in PPH )
Q. 46
Testosterone secreted by:-
1. Gonadotropic cells
2. Sertoli cells
3. Acidophilic cells
4. Leydig's cells
CORRECT
SOLUTION
LEYDIG CELLS:
 Testosterone is secreted by Leydig cells located in interstitium of testis.
 It is essential for growth & division of testicular germinal cells, which is 1st stage in
forming sperm.
 L.H secreted by anterior pituitary gland, stimulates Leydig cell to secrete testosterone.
Sertoli cells:
 FSH secreted by anterior pituitary gland, stimulates Sertoli cells
 This stimulation causes conversion of spermatids to sperm (spermiogenesis)
 Without this stimulation, spermiogenesis do not occur.
 Secrete inhibin, Mullerian inhibiting substance
 Forms blood testes barrier
Gonadotropic cells:
 These cells are present in anterior pituitary gland which secrete both FSH & L.H under
the influence of GnRH.
Acidophilic cells:
 Somatotrophs in anterior pituitary stain strongly with acid dyes so they are called
acidophil cells.
 These secrete growth hormone, which stimulates lipolysis, inhibits action of insulin on
carbohydrate & lipid metabolism.
REFERENCE
Guyton; chapter 81, page no. 1023; 1022, figure 81-2
Guyton, chapter 81, page 1031
Guyton, chapter 76, page 940
Q. 47
Aquaporin channel mediated through ADH:-
1. GLUT
2. Aquaporin 1
3. Aquaporin 2
CORRECT
4. Aquaporin 3
SOLUTION
ADH:
 Most important renal action of ADH is to increase water permeability of distal tubule,
collecting tubule & collecting duct epithelia.
 This helps the body to conserve H2O in circumstances like dehydration
 ADH binds to specific V2 receptors in late distal tubules, collecting tubules & collecting
ducts, increases formation of cAMP & activates protein kinases.
 This action in turn, stimulates movement of intracellular protein called aquaporin – 2
(AQP 2) to luminal side of cell membranes
 Molecules of AQP 2 cluster together & fuse with cell membrane by exocytosis to form
H2O channels that permit rapid diffusion of water through cells.
 Other aquaporins, AQP 3 & AQP 4, on basolateral side of cell membrane provide path
for H2O to rapidly exit the cells, but these aquaporins are not believed to be regulated by
ADH [REFER GUYTON AND HALL TEXTBOOK OF PHYSIOLOGY,CHAPTER-76,
Page-948,949]
GLUT:
 Group of membrane proteins that facilitate transport of glucose across the plasma
membrane
Q. 48
All are true about Decerebrate posture except:-
1. Extension of both upper and lower extremity
2. Flexion of upper extremity and extension of lower extremity
CORRECT
3. Exaggerated gamma motor neuron discharge
4. Reticulo spinal tract is also involved
SOLUTION
Decerebrate rigidity:
 When brain stem of animal is sectioned below midlevel of mesencephalon but pontine &
medullary reticular systems as well as vestibular system are intact, this is decerebrate
animal
 This results in rigidity of antigravity muscles à muscles of neck & trunk & extensors (i.e
all extensors go into spasm)
 This decerebrate rigidity is due to blockage of normally strong input to medullary
reticular nuclei from cerebral cortex, Red nuclei & the basal ganglion.
 Lacking this input, medullary reticular inhibitor system becomes nonfunctional, full
overactivity of pontine excitatory system occurs & rigidity develops.
 (Due to exaggerated gamma motor neuron discharge)
Decorticate rigidity:
 Thick layer of cortex is removed & all other structures are preserved.
 This results in decorticate rigidity à flexors of upper limb & extensors of lower limb goes
into spasm (flexion).
REFERENCE
Guyton, chapter 56, page 714
Q. 49
Bainbridge reflex is due to:-
1. Stretching of atria
CORRECT
2. Baroreceptor reflex
3. Decreased venous return
4. Increased ventricular activity
SOLUTION
Bainbridge reflex:
 Increase in atrial pressure causes increase in heart rate by nervous reflex called
Bainbridge reflex 40 – 60 % increase in rate is due to this reflex while 15% increase is by
direct effect of increased atrial volume to stretch sinus node.
 Stretch receptors of atria that elicit Bainbridge reflex transmit their afferent signals
through vagus nerve to medulla of brain.
 These efferent signals are transmitted back through vagal & sympathetic nerves to
increase H.R & strength of heart contraction.
 This reflex helps prevent damming of blood in atria, veins & pulmonary circulation
 Best known nerve mechanisms for arterial pressure control
 Reflex is initiated by stretch receptors called baroreceptors /pressoreceptors located at
specific points in walls of several large systemic arteries
 A rise in arterial pressure stretches baroreceptor & causes them to transmit signals into
CNS.
 “feedback” signals are then sent back through autonomic nervous system to circulation to
reduce arterial pressure downward towards the normal level
REFERENCE
Guyton, chapter 18, page 223] Baroreceptor reflex
Guyton, chapter 18, page 219, 220
Q. 50
Function of Golgi tendon organ is to detect:-
1. Tension
CORRECT
2. Length
3. Pressure
4. Proprioception
SOLUTION
→ Muscles & their tendons are supplied abundantly with 2 special types of sensory receptors
1.muscle spindles → Distributed throughout belly of muscle & send information to nervous
system about muscle length / rate of change of length.
2. Golgi tendon organs → Located in muscle tendons & transmit information about tendon
tension or rate of change of tension.
→ Signals from these 2 Receptors are almost entirely for intrinsic muscle
REFERENCE
Guyton, chapter 55, page 697
Q. 51
Peripheral chemoreceptors respond to hypoxia using which channel?
1. Calcium channel
2. Sodium channel
3. Potassium channel
CORRECT
4. Chloride channel
SOLUTION
PERIPHERAL CHEMORECEPTOR:
→ Located in several areas outside the brain, mostly in carotid bodies & aortic bodies
→ They are especially important for detecting changes in O2 in the blood (lesser extent to
changes in CO2 & H+ ion concentration)
→ Aortic & carotid bodies have multiple, highly characteristic glandular like cells called
“GLOMUS CELLS” that synapse directly or indirectly with nerve endings, (they serve as
chemoreceptor)
→ Low Po2 stimulate glomus cells in aortic & carotid bodies
↓
O2 sensitive potassium channels in glomus cells are inactivated
↓Resulting in
Cell depolarization
↓Which in turn
Opens voltage gated calcium channels & Increases Intracellular Ca ion concentration
↓
Stimulate neuro transmitter release
↓
Which activates afferent neurons that sends signals to CNS
↓
Stimulates respiration
→ During hypoxia ATP (mainly), ach / dopamine are excitatory Neurotransmitters.
REFERENCE
Guyton page 543, 544, figure 42.6
Q. 52
Functional residual capacity represents the volume of air remaining in lungs:-
1. After normal inspiration
2. After normal expiration
CORRECT
3. After forceful expiration
4. After forceful inspiration
SOLUTION
Functional residual capacity:
→ It equals the EXPIRATORY RESERVE VOLUME + RESIDUAL VOLUME
→ This capacity is amount of air that remains in lungs at end of normal expiration (about 2300
ML)
Pulmonary capacities:
Inspiratory capacity:
→ Tidal volume + inspiratory reserve volume [about 3500ml]
→ This capacity is amount of air a person can breathe in, beginning at normal expiratory level &
distending the lungs to maximum amount.
Vital capacity:
→ Inspiratory reserve volume + tidal volume + expiratory reserve volume.
→ This capacity is max amount of air a person can expel from lungs after first filling the lungs to
their maximum extent & then expiring to maximum extent (about 4600ml)
Total lung capacity:
→ Max vol to which lungs can be expanded with greatest possible effort [5800 ml]
→ Vital capacity + residual volume
REFERENCE
Guyton and Hall Textbook of Medical Physiology 13th Edition, Page No:501, 502.
Q. 53
Tubuloglomerular feedback control is useful for which one of the following:-
1. GFR
2. Plasma sodium
3. Plasma volume
CORRECT
4. Determining tubular secretion
SOLUTION
Tubulo-glomerular feedback:
 Increase in blood pressure (changes in B.P)
Increase GFR
↓
Sodium filtration
↓
Sensed by macula densa cells of thick ascending limb
↓
Sends tubulo-glomerular feedback by adenosine
↓
Adenosine results in constriction of afferent arteriole
↓
Resulting in decrease GFR & decrease Na+ filtration
This feedback is to prevent loss of sodium & avoid electrolyte imbalance.
REFERENCE
GUYTON AND HALL TEXTBOOK OF MEDICAL PHYSIOLOGY 13TH EDITION-PAGE-
343, 344, fig (27-11)
Q. 54
Antegrade peristalsis due to?
1. Ach
CORRECT
2. Serotonin
3. Substance P
4. VIP
SOLUTION
→ Excitatory neurotransmitters in ENS are Acetylcholine & substance P
→ Inhibitory neurotransmitters in enteric nervous system are NO & VIP
→ Acetylcholine is main excitatory neurotransmitter responsible for antegrade peristalsis
REFERENCE
Guyton and Hall Textbook of Medical Physiology 13th Edition, Page No. 800
Q. 55
C- peptide is seen:-
1. In pre-proinsulin
2. In Proinsulin
CORRECT
3. As a combined entity with insulin after secretion
4. As a gastrointestinal proactive molecule
SOLUTION
→ Insulin is synthesized in BETA- cells of pancreas by usual cell machinery for protein
synthesis.
→ Beginning with translation of insulin RNA by ribosomes attached to E.R to form pre
proinsulin.
→ Pre proinsulin is cleaved in E.R to form pro-insulin.
→ Insulin & C peptide are packaged in secretory granules & secreted in equimolar amounts.
→ So, c – peptide levels can be measured by radioimmunoassay in insulin-treated diabetic
patients to determine how much of their own natural insulin they are still producing
REFERENCE
Guyton, chapter 79, page 984, figure (79.2)
Q. 56
In forceful expiration, which of the following neurons gets fired?
1. VRG
CORRECT
2. DRG
3. Pneumotaxic centre
4. Chemoreceptors
SOLUTION
→ Located in each side of medulla, about 5mm anterior & lateral to dorsal respiratory group of
neurons is ventral respiratory group of neurons, found in nucleus ambigus rostrally & nucleus
retroambigus caudally.
→ Neurons of VRG remain almost totally inactive during normal quiet respiration. so, normal
quiet breathing is caused only by repetitive inspiratory signals from DRG transmitted mainly to
diaphragm & expiration results from elastic recoil of lungs & thoracic cage.
→ When respiratory drive for increased pulmonary ventilation becomes greater than normal,
respiratory signals spill over into VRG from basic oscillating mechanism of DRG
→ As a consequence, VRG contributes extra respiratory drive as well
→ Electrical stimulation of few of neurons in VRG cause inspiration & others cause expiration,
thus these neurons contribute to both inspiration & expiration.
→ They are especially important in providing powerful expiratory signals to abdominal muscles
during very heavy expiration.
Pneumotaxic center: Primary effect is to control “switch off” point of inspiratory ramp, thus
controlling duration of filling phase of lung cycle.
→ Main function is to limit inspiration which has secondary effect of increasing rate of breathing
because limitation of inspiration also shortens expiration & entire period of each respiration
Chemoreceptors:
→ Sensors that detect changes in Co2, O2, pH.
→ Based on anatomical position divided into

o Central chemoreceptors
o Peripheral chemoreceptors
REFERENCE
Guyton, chapter 42, page 540, figure 42.1
Q. 57
Slow conduction velocity is seen with which of the following nerve fibers?
1. Preganglionic autonomic nerve fibers
2. Postganglionic autonomic nerve fibers
CORRECT
3. Motor nerves
4. Sympathetic nerve fibers
SOLUTION
 Slow conduction velocity is seen in ‘C’- fibers because they are unmyelinated.
 These are post ganglionic sympathetic nerve fibers
REFERENCE
Ganong’s Review of Medical Physiology 26th Edition, Page No. 256
Q. 58
Which of the following is water soluble contrast?
1. Barium
2. Iodine
CORRECT
3. Bromium
4. Calcium
SOLUTION
Radiocontrast
Positive Negative
Barium (Water isoluble) Iodinated Compounds (water soluble) Air Water
• Iohexol
• Iopamidol
• Iodixanol
Q. 59
Identify the infection from the chest x-ray of patient with low grade fever?
1. ILD
2. Bronchopenumonia
3. Miliary TB
CORRECT
4. Consolidation
SOLUTION
• Chest X-ray shows Millet seed like opacities which is a feature of miliary TB.
• Millet seed like opacities are also seen in
- Healed varicella pneumonia
- Mitral stenosis
REFERENCE
GRAINGER AND ALLISON’S DIAGNOSTIC RADIOLOGY 6TH EDITION Page No: 259-
261
Q. 60
A patient presented with history of diplopia and restricted eye movements. A clinical image and
CT image is shown below What will be the diagnosis?
1. Le-fort fracture
2. Fracture maxilla
3. Fracture zygomatic
4. Blow out fracture
CORRECT
SOLUTION
• Clinical image- shows Enophthalmos, congestion and edema of left eye
• Coronal CT- shows orbital floor fracture causing orbital contents hanging
in left maxillary sinus
Blow out # or tear
drop #
• Best X-ray view for Orbital floor and maxillary sinus is WATER’S
VIEW
REFERENCE
GRAINGER AND ALLISON’S DIAGNOSTIC RADIOLOGY 6THEDITION Page No: 1037
Q. 61
Identify the radiological procedure shown below?
1. Barium meal follow through
CORRECT
2. Barium enema
3. Enteroclysis
4. Proctography
SOLUTION
• The given radiological image of barium study shows the presence of contrast in stomach,
duodenum, jejunum (shows feathery appearance- valvulae conniventes) and ileum (featureless
app.) - indicating that it is a barium meal follow through study.
• Barium studies:
→ Barium swallow – esophagus
→ Barium meal – stomach & duodenum
→ Barium meal follow through – small intestine
→ Barium enema – large intestine
• Enteroclysis: NJ Tube is inserted and contrast is injected into jejunum directly to visualize
small bowel
Enteroclysis: Used to detect small bowel lesions or any malabsorption.
* CT vs MRI
 Look at CORTICAL BONE – if dark = MRI
REFERENCE
GRAINGER AND ALLISON’S DIAGNOSTIC RADIOLOGY 6TH EDITION Page No : 653
Q. 62
Identify the condition in the below image?
1. Lacunar infarct
CORRECT
2. Embolic infarct
3. Thrombotic infarct
4. Intracerebral hemorrhage
SOLUTION
• The given CT image shows hypodensity in thalamic region→ lacunar infarct
• Lacunar infarct occurs due to occlusion of lenticulostriate branches or thalamostriate branches
or deep penetrating branches of brain.
• Lacunar infarct appears hypointense on T1 and hyperintense on T2 image
REFERENCE
GRAINGER AND ALLISON’S DIAGNOSTIC RADIOLOGY 6TH EDITION Page No: 1456
Q. 63
Not true about somnambulism among the following is?
1. Sleep walking
2. Patient consciousness is preserved
CORRECT
3. Disorder of sleep arousal
4. Low level motor skill/function is present
SOLUTION
Sleepwalking (also called somnambulism) is a disorder of arousal(usually seen in children). It
generally occurs in deep nonrapid eye movement (NREM) (stages 3 and 4) sleep, most often
takes place in the first third of the night when these sleep stages are most common. The event
may last from several minutes to an hour. a child appears to wake up during the night and walk
or do other activities without any memory of having engaged in the activities.Difficult in waking
up the child and may physical attacks by the child after wakeup.
Rx- as the child grows up these symptoms will improve.
Q. 64
Confusion assessment scale used for which of the following:-
1. Schizophrenia
2. Delirium
CORRECT
3. Dementia
4. Depression
SOLUTION
Confusion Assessment method (CAM) is used for delerium.
METHOD-
-Ask patient to answer simple questions and perform simple task.
-Can be used readily at the bedside.
REFERENCE
Kaplan and Sadock's Comprehensive Textbook of Psychiatry, 10th edition, Page No 1183
Q. 65
Which of the following type of OCD has the poor response exposure and response prevention?
1. Magical thinking
2. Dirt contamination
3. Pathological doubt
4. Hoarding
CORRECT
SOLUTION
Obsessive Compulsive Disorder (OCD)
Obsessions:-
 Repetitive, intrusive thoughts, images or impulses which cause anxiety.
 Patient consider them as product of their own mind (D/D – thought insertion)
Compulsions:-
Behaviour on mental acts that follow & reduce anxiety temporarily #Both obsessions and
compulsion are Ego dystonic – not agreeable to self Prevalence – 2 – 3 % 50% - acute onset
symptoms persist for 2 weeks
Etiology:- seratonergic dysregulation
Nor epinephrine
CSTC Tract Cortico – striato – thalamico – cotical tract
Orbito – frontal – Straita – Thalamus Cortex
Presentantion
 Obsession of contamination followed by compulsion of washing (MC) –
 Pathological doubt – Obsession of doubt with compulsion of checking.
 Intrusive thoughts
↓
Aggressive / Religious / Sexual content
 Symmetry – obsession of symmetry or precision Compulsion of slowness Rx –
combination of pharmacotherapy and psychotherapy
Hoarding is no longer subtype of OCD. In DMS 5 AND ICD-11 hoarding disorder is
separate diagnosis.
People with hoarding disorder excessively save items that others may view as worthless. They
have persistent difficulty getting rid of or parting with possessions, leading to clutter that disrupts
their ability to use their living or work spaces.Hoarding is not the same as collecting. Collectors
look for specific items, such as model cars or stamps, and may organize or display them. People
with hoarding disorder often save random items and store them haphazardly. In most cases, they
save items that they feel they may need in the future, are valuable or have sentimental value.
Some may also feel safer surrounded by the things they save.
Q. 66
Intense depression & misery without any cause is?
1. Melancholia
CORRECT
2. Major depressive disorder
3. Mania
4. Schizophrenia
SOLUTION
Melancholic depression –
A. Prominent biological symptom
1. Early morning awakening (waking up at least 2 hours before usual time)
2.Significant anorexia and weight loss.
B. Significant psychomotor agitation or retardation.
C. Anhedonia and lack of mood reactivity.(the patients mood doesn’t improve even if some
positive event occurs )
D. A distinct quality of intensely depressed mood, often referred to as a state of despondency
or despair,also called empty mood.the patients feels miserable.
E. Depression is worse in morning.
F. Excessive guilt.
REFERENCE
Kaplan and sadocks synopsis of psychiatry edition 11 page 347
Q. 67
Which of the following is not true regarding delusional disorder?
1. Early immigration
2. Social isolation
3. Sensory impairment
4. Occurs at early age
CORRECT
SOLUTION
Delusional disorders do not occur at an early age.
Risk factors for development of delusional disorders-
 Advanced age
 Social isolation
 Sensory impairment or isolation (e.g. auditory or visual disturbances)
 Family history of delusional disorder
 Recent immigration (is also risk factor for schizophrenia)
 Certain personality features ,like excessive interpersonal sensitivity (even trivial
interpersonal problems cause lot of negative emotions)
REFERENCE
Kaplan and sadocks synopsis of psychiatry edition 11 page 330-336
Q. 68
Identify the instrument:-
1. Maddox rod
CORRECT
2. Maddox wing
3. Maddox glass
4. Red glass
SOLUTION
Maddox rod test is used for diagnosis of heterophoria. The patient is placed in front of a bright
spot of light in a dark room. A Maddox rod, which consists of four or five cylinders of red glass
side by side in a supporting disc, is placed in the trial frame before one eye . The spot of light
seen through the red cylinders appears as a long red line perpendicular to the direction of the
cylinders or grooves. If the cylinders are placed with their axes horizontal, the red line will be
vertical. If there is orthophoria the bright spot will appear to be in the centre of the vertical red
line; if there is eso- or exophoria the red line will be to one side of the spot. The angle of the
deviation is measured by the strength of the prism which is necessary to be placed in front of the
Maddox rod (or the other eye) to bring the red line and the spot together. The nature of the
deviation is indicated by the position of the base of the prism, whether out (eso- phoria) or in
(exophoria).
Q. 69
Esotropia is commonly seen in which type of refractive error?
1. Myopia
2. Hypermetropia
CORRECT
3. Astigmatism
4. Presbyopia
SOLUTION
These are generally more common in childhood and hyper- metropes.
The continuous effort of accommodation in the hypermetrope to see clearly, even in the distance,
stimulates convergence to a greater degree than is compatible with binocular fixation and results
in esotropia.
REFERENCE
Comprehensive Ophthalmology AK Khurana
Q. 70
In the given condition, cause of defective vision is:-
1. Visual axis is obscured
2. Astigmatism
CORRECT
3. Hypermetropia
4. Diplopia
SOLUTION
The above image shows a pterygium. Defective vision occurs when it encroaches the pupillary
area or due to corneal astigmatism induced by fibrosis in the regressive stage.
Q. 71
This test is used in:-
1. Squint
2. Heterophoria
3. Esotropia
4. All of the above
CORRECT
SOLUTION
The above diagram shows cover uncover test used for squint diagnosis(both tropias and
phorias).
Interpretation of cover, cover–uncover and alternate cover tests.
REFERENCE
AK Khurana Pg 315
Q. 72
What is your diagnosis:-
1. Orbital blow out fracture
CORRECT
2. Orbital roof fracture
3. Oculomotor nerve paralysis
4. Retrobulbar hemorrhage
SOLUTION
Following image shows a defect in the orbital floor and the ‘tear drop’ sign in the antrum
characteristic of blow out fractures of orbit.
WATER'S view on X-RAY is helpful in eliciting orbital blow out fracture
REFERENCE
Comprehensive ophthalmology A K Khurana 6th edition page 423
Q. 73
Which one is against the rule astigmatism?
1. +2.0 DC x 90
2. -2.0 DC x 90
CORRECT
3. +1.5 DC x 180
4. -1.5 DC x 180
SOLUTION
Against-the-rule astigmatism (ATR) refers to an astigmatic condition in which the horizontal
meridians is more curved than the vertical meridian. Therefore, correction of this astigmatism
will require the prescription of convex cylindrical lens at 180° ± 7 20° or concave cylindrical
lens at 90° ± 20° axis.
Therefore, both 2 and 3 are the correct options.
Q. 74
Intravenous mannitol is indicated in:-
1. Primary open angle glaucoma
2. Acute angle closure attack
CORRECT
3. Normal tension glaucoma
4. Sympathetic ophthalmitis
SOLUTION
Mannitol is a hyperosmotic agent. Hyperosmotic agents increase the plasma tonicity. Thus, the
osmotic pressure gradient created between the blood and vitreous draws sufficient water out of
the eyeball, thereby significantly lowering the IOP. Indications These are used as additive
therapy for rapidly lowering the IOP in emergency situations, such as acute angle-closure
glaucoma or secondary glaucomas with very high IOP. They are also used as a prophylactic
measure prior to intraocular surgery.
Q. 75
The ocular hypotensive agent causing apnoea in infants is:-
1. Latanoprost
2. Timolol
3. Betaxolol
4. Brimonidine
CORRECT
SOLUTION
Brimonidine is selective alpha 2 receptor agonist.
It can cause drowsiness and respiratory depression in children and infants and therefore
contraindicated in them.
REFERENCE
AK khurana 7th edition pg no. 243,469,473
Q. 76
The dosage of vitamin A in keratomalacia in a 2 year old boy who is 12 kg weight is:-
1. Vitamin A: 2 lakh in oral, 1st, 2nd, 14th day
CORRECT
2. Vitamin A: 1 lakh in oral, 1st, 2nd, 14th
3. Vitamin A: 2 lakh in oral, 1st, 2nd, 3rd
4. Vitamin A: 1 lakh in oral, 1st, 2nd, 3rd
SOLUTION
1. All patients above the age of 1 year (except women of reproductive age): 200,000 IU of
vitamin A orally or 100,000 IU by intramuscular injection should be given immediately
on diagnosis and repeated the following day and 4 weeks later.
2. Children under the age of 1 year and children of any age who weigh less than 8 kg should
be treated with half the doses for patients of more than 1 year of age. iii.Women of
reproductive age, pregnant or not: (a) Those having night blindness (XN), conjunctival
xerosis (X1A) and Bitot’s spots (X1B) should be treated with a daily dose of 10,000 IU
of vitamin A orally (1 sugar coated tablet) for 2 weeks. (b) For corneal xerophthalmia,
administration of full dosage schedule (described for patients above 1 year of age) is
recommended.
Q. 77
Which of the parameters is decreased in Retinitis pigmentosa?
1. Arachidonic acid
2. Docosahexanoic acid
CORRECT
3. Thromboxane
4. Trielonic acid
SOLUTION
Mean plasma DHA levels (as percentages of total plasma fatty acids in plasma) were reduced
below the mean normal levels in patients with X-linked and isolate forms of RP.
Q. 78
The most common cause of proptosis in adults:-
1. Orbital cellulitis
2. Preseptal cellulitis
3. Thyroid eye disease
CORRECT
4. Capillary hemangioma
SOLUTION
Most common cause of unilateral proptosis in children is orbital cellulitis and in adults is
thyroid ophthalmopathy.
Most common cause of bilateral proptosis in children is neuroblastoma and leukemia (chloroma)
and in adults is thyroid ophthalmopathy.
REFERENCE
Ak khurana 7th edition 421
Q. 79
Which one of the procedure involves using glaucoma drainage device?
1. Seton operation
CORRECT
2. Deep sclerectomy
3. Viscocanalostomy
4. Trabeculectomy
SOLUTION
1. Seton operation- Glaucoma drainage device or the so called glaucoma valve
implants are plastic devices which allow aqueous outflow by creating a
communication between the anterior chamber and sub-Tenon’s space. The
operation using glaucoma valve implant is also known as Seton operation 2.
2. Deep sclerectomy-A Descemet window is created to allow aqueous migration from the
AC. Subsequent egress is subconjunctival, resulting in a shallow filtration bleb, as well as
along deeper suprachoroidal routes.
3. Viscocanalostomy-Viscocanalostomy involves the creation of a filtering window,
with identification and dilatation of the Schlemm canal with high density
viscoelastic. The superficial scleral flap is sutured tightly so that subconjunctival
fluid outflow and bleb formation are minimized
4. Trabeculectomy-Trabeculectomy is glaucoma filtration surgery that lowers IOP by
creating a fistula, protected by a superficial scleral flap, to allow aqueous outflow from
the anterior chamber to the sub-Tenon space.
Q. 80
Oral factor Xa inhibitor is:-
1. Dabigatran etexilate
2. Rivaroxaban
CORRECT
3. Fondaparinux
4. Bivalirudin
SOLUTION
 Any drug ending with XABAN is X a blocker,
 Rivaroxaban
 Apixaban
 Edxaban
 Betrixaban
Dabigatran: Direct thrombin inhibitor
Only oral agent in class(DTI)
Can reverse with Idarucizumab.
Fondaparinux: LMWH which acts only on Factor X a
Bivalirudin : Direct Thrombin Inhibitor.
REFERENCE
Lippincott pharmacology 6th edition page 303
Q. 81
A nursing mother presented with fever and breast tenderness after two weeks postpartum. Which
oral antibiotics is ideal for her condition?
1. Ampicillin
2. Dicloxacillin
CORRECT
3. Ceftazidime
4. Ciprofloxacin
SOLUTION
 Dicloxacillin:- Anti - staphylococcal penicillin :- B lactamase resistant penicillin. Their
use is restricted to the treatment of infection cause by penicillinase producing
staphylococci including MSSA. Other drug of this class:- Methicillin Oxacillin
 Methicillin because of its toxicity (interstitial Nephritis), Methicillin is not used.
 Ampicillin/Amoxicillin:-Extended- Spectrum penicillin. More effective against gram -
ve bacilli.
 Ampicillin is a drug of choice for gram positive bacilli : listeria monocytogenes
 Ceftazidime:-3rd
gen cephalosporin, has activity against P. aeruginosa
 Ciprofloxacin: of the fluoroquinolone it has best activity against P. Aeruginosa and is
commonly used in cystic fibrosis patient.
REFERENCE
KD Tripathi Essentials of Medical Pharmacology 8th edition/page 771
Q. 82
High plasma protein binding of a drug results in:-
1. Decreased glomerular filtration
CORRECT
2. High volume of distribution
3. Lowers duration of action
4. Less drug interaction
SOLUTION
• High plasma protein binding (PPB) of a drug decreases its distribution into tissue and thus
lower the volume of distribution.
• As only free drug can be metabolized or excreted, high PPB usually increases the duration of
action.
• With higher PPB, there is more chance of getting displaced by other drugs leading to more
drug interactions.
• Drugs with high plasma protein binding have lesser glomerular filteration as proteins cannot be
filtered across the glomerulus.
REFERENCE
KD Tripathi Essentials of Medical Pharmacology 8th edition/page 26
Q. 83
DOC for prophylaxis against Diphtheria is:-
1. Erythromycin
CORRECT
2. Rifampicin
3. Cloxacillin
4. Ciprofloxacin
SOLUTION
 Erythromycin is macrolide group of drug which binds irreversibly to a site on the 50s
subunit of bacterial ribosome thus inhibiting translocation steps of protein synthesis.
 It is used to eliminate the carrier state of Corynebacterium Diphtheriae.
 Rifampicin: Blocks RNA transcription by interacting with B subunit of mycobacterial
DNA dependent RNA polymerase.
REFERENCE
Lippincott pharmacology 6th edition page 431
Q. 84
Variation in drug responsiveness to a particular dose of the drug in different individuals can be
obtained from:-
1. Graded Dose Response Curve
2. Efficacy
3. Potency
4. Quantal Dose response Curve
CORRECT
SOLUTION
 ED50 : dose at which 50% of subject will responds to drug.
 Potency :- Measure of amount of drug necessary to produce an effect of a given
magnitude.
 Efficacy: Magnitude of response a drug can cause when it interacts with a receptor.
Depends on of drug receptor complex formed.
• In this Quantal DRC, x-axis represents the log dose and y-axis represents the percentage of
population responding.
• It is used when responses are like all or none. The middle point of curve when projected to x-
axis gives effective dose and lethal dose in 50% of population.
REFERENCE
KD Tripathi Essentials of Medical Pharmacology 8th edition/page 64
Q. 85
Most specific antiemetic for chemotherapy induced vomiting is:-
1. Doxylamine
2. Tegaserod
3. Granisetron
CORRECT
4. Domperidone
SOLUTION
DOC for chemotherapy induced vomiting are 5HT3 antagonist which are drugs ending with
setron.
 Granisetron
 Ondansetron
 Dolasetron
These agent selectively block 5HT3 receptor in periphery (Visceral Vagal afferent fibers) and
in brain(CTZ)
Doxylamine are somnifacient (have strong sedative properties and are used in treatment of
insomnia) these are 1st generation antihistamine like diphenhydramine .
REFERENCE
ESSENTIALS OF MEDICAL PHARMACOLOGY -8TH EDITION BY KD TRIPATHI (Pg-711)
Q. 86
Identify the true statement regarding clinical trials:-
1. Phase 1 is done to determine efficacy in patients
2. Healthy volunteers are recruited for the first time in phase II
3. Randomized controlled trials in patients is done in phase III
CORRECT
4. Phase IV is pharmacokinetics study in animals
SOLUTION
Clinical Trials
Phase Done On Aim
Cannot
find
I Healthy individual To know MTD Efficacy
II Patient (20-200) Efficacy indication
III Patient (upto 5000) Confirm efficacy
IV (post
marketing)
Large number of
patients
To know rare side effect and chronic
s/e.
 MTD :- Maximum tolerable dose
 1st phase in human: Phase I
 1st phase in patient : phase II
 Indication of efficacy of drug :- Phase II
 Confirmation of efficacy of drug :- Phase III
 Maximum no. of patient :- phase IV
 Double Blinding and Randomized control Trial is done in phase III
REFERENCE
KD Tripathi Essentials of Medical Pharmacology 8th edition/page 90
Q. 87
Fluoroquinolone with highest oral bioavilability:-
1. Levofloxacin
CORRECT
2. Gemifloxacin
3. Ciprofloxacin
4. Norfloxacin
SOLUTION
Bioavailability : rate and extend to which an administered drug reaches the systemic circulation .
Of the above mentioned drugs:-
 Levofloxacin has maximum bioavailability
 Norfloxacin has minimum bioavailability
REFERENCE
KD Tripathi Essentials of Medical Pharmacology 8th edition/page 764
Q. 88
Which of the following statement is true regarding fixed dose combination of drugs?
1. Adverse effect of one drug may be reduced by the other drug
CORRECT
2. Two drugs with different pharmacokinetics can be combined easily
3. Dose of one drug can be altered independently as per requirement
4. Adverse effect can be ascribed to a single drug
SOLUTION
Fixed dose combination (FDC):- Combination of two or more active drugs in single dose form.
Rationality of FDC’s should be based on certain aspect such as:-
 The drugs in combination should act by different mechanism.
 Pharmacokinetic must not be widely different.
 Combination should not have supraadditive toxicity of the ingredients.
Demerits:-
 Dosage alteration of one drug is not possible without alteration of other drugs.
 Differing pharmacokinetic of constituent drugs poses the problem of frequency of
administration of the formula.
 Increased chance of adverse drug effect and interaction compared with both drugs given
individually.
REFERENCE
Lippincott pharmacology 6th edition page 598
Q. 89
Which of the following is a K+
channel opener?
1. Ranolazine
2. Nicorandil
CORRECT
3. Verapamil
4. Lignocaine
SOLUTION
NICORANDIL:-
 K+
channel opener
 NO releaser
 Given for Angina
 Drug activates ATP sensitive K+
channels --> Hyperpolarizing vascular smooth muscle.
 Nicorandil is believed to exert cardioprotective action by simulating ‘ischaemic
preconditioning’ as a result of activation of mitochondrial K-ATP channels.
Ranolazine - Inhibits late phase of sodium current (Late I Na) improving oxygen supply and
demand equation. It has antianginal as well as antiarrhythmic properties.
Verapamil :- calcium channel blocker, slows AV conduction directly and decrease HR,
contractility, BP and oxygen demand.
Lidocaine : class IB antiarrhythmic drug, shortens phase 3 repolarization and decrease the
duration of action
REFERENCE
KD Tripathi Essentials of Medical Pharmacology 8th edition/page- 552
Q. 90
A rheumatoid Arthritis patient on Methotrexate, steroids and NSAIDs for past 4 months has had
no retardation of disease progression. What is the next rational step in management?
1. Start monotherapy with anti-TNF alpha drugs
2. Continue Methotrexate and steroids
3. Stop oral Methotrexate and start parenteral Methotrexate
4. Add Sulfasalazine
CORRECT
SOLUTION
 For patient with RA, Steroid and NSAIDS one given for symptomatic relief.
 Methotrexate used alone or in combination therapy has become a mainstay of treatment
in patient with RA or psoriatic arthritis.
 Methotrexate is a folic acid antagonist that inhibits cytokine production and purine
nucleotide biosynthesis leading to immunosuppressive and anti-inflammatory effects.
 If methotrexate alone is not working, We add sulfasalazine, If it still doesn’t work, we
give triple therapy which are:-
 Methotrexate
 Sulfasalazine
 Hydroxychloroquine/ Leflunomide
 If triple therapy doesn’t work, we Start INF inhibitor.
REFERENCE
KD Tripathi Essentials of Medical Pharmacology 8th edition/page : 734
Q. 91
Tolvaptan is used for:-
1. SIADH
CORRECT
2. Central DI
3. Von willebrand disease
4. Catecholamine resistant Shock
SOLUTION
VAPTAN are Vasopressin antagonist it is given to patient who have excess ADH (SIADH).
For patient with central DI and von Willebrand disease- Desmopressin.
New indication of Tolvaptan is Autosomal dominant polycystic kidney Disease (ADPKD)
REFERENCE
KD Tripathi Essentials of Medical Pharmacology 8th edition/page: 639
Q. 92
Mechanism of action of Oseltamivir and Zanamivir is:-
1. DNA polymerase inhibition
2. Inhibition of viral mRNA
3. Blocking viral uncoating
4. Neuraminidase inhibition
CORRECT
SOLUTION
Oseltamivir acts by inhibiting influenza virus neuraminidase enzyme which is needed for release
of progeny virions from the infected cell.
Other neuraminidase inhibitor includes Zanamivir & Peramivir.
These are used against Flu Virus (Bird flu/swine Flu).
Drugs like Amantadine and Rimantadine are viral uncoating inhibitors.
Route of administration of neuraminidase inhibitors in influenza:
Zanamivir: Inhalational
Oseltamivir: Oral
Peramivir: Intravenous
REFERENCE
KD Tripathi Essentials of Medical Pharmacology - 8th Edition, Page No. 853
Q. 93
Buspirone acts on:-
1. 5HT1A
CORRECT
2. 5HT1B
3. 5HT2
4. 5HT3
SOLUTION
Drugs like :
Buspirone
Azapirone
Ipsapirone
Gepirone
Are used for anxiety and it acts on 5HT 1A
Clozapine has high affinity for D1, D4, 5HT2, X adrenergic receptor.
Ondansetron
Granisetron
Palonosetron
Dolasetron
5HT3 receptor Antagonist
Palonsetron - Maximum potency
REFERENCE
KD Tripathi Essentials of Medical Pharmacology 8th edition/page 185
Q. 94
Carbapenem with maximum seizure risk is:-
1. Imipenem
CORRECT
2. Meropenem
3. Ertapenem
4. Doripenem
SOLUTION
Carbapenems are synthetic Beta lactam antibiotic. Imipenem is compounded with cilastatin to
protect it from metabolism by renal dehydropeptidase. Cilastatin is a dehydropeptidase inhibitor.
High level of imipenem may provoke seizure, however other carbapenem like:-
 Meropenem
 Ertapenem
 Doripenem
Are less likely to do so.
REFERENCE
Lippincott pharmacology 6th edition page 504
Q. 95
Cilastatin is given in combination with Imipenem because:-
1. Cilastatin prevents degradation of Imipenem in kidney
CORRECT
2. Cilastatin increase absorption of Imipenem
3. Inhibits the enzymes that digest Imipenem in stomach
4. Reduces side effects of Imipenem
SOLUTION
Carbapenems are synthetic Beta lactam antibiotic. Imipenem is compounded with cilastatin to
protect it from metabolism by renal dehydropeptidase. Cilastatin is a dehydropeptidase
inhibitor. High level of imipenem may provoke seizure, however other carbapenem like:-
 Meropenem
 Ertapenem
 Doripenem
Are less likely to do
REFERENCE
KD Tripathi Essentials of Medical Pharmacology 8th edition/page 43
Q. 96
Mechanism of action of Triazoles:-
1. Inhibits ergosterol biosynthesis
CORRECT
2. Inhibits tubulin
3. Inhibits glucan synthesis
4. Inhibits cell wall synthesis
SOLUTION
 The azoles act by inhibiting fungal cytochrome P450 enzyme 'lanosterol 14-demethylase'
and thus impair ergosterol synthesis.
 These are presently the most extensively used antifungal drugs.
 Mechanism of action of other antifungal drugs:
o Polyene (Amphotericin B/Nystatin): Binds to ergosterol and creates pores.
o 5 Flucytosine : DNA polymerase inhibitor.
o Allylamine (Terbinafine, Butenafine, Naftifine): Inhibit enzyme squalene
epoxidase.
o Griseofulvin : Act mitotic spindle
o Echinocandin (Caspofungin): Inhibit Beta 1-3
REFERENCE
KD Tripathi Essentials of Medical Pharmacology - 8th Edition, Page No. 843
Q. 97
Identify the false statement regarding Teduglutide is:-
1. It is Used for short-Bowel Syndrome
2. It is a GLP-2 agonist
3. It it a pancreatic enzyme
CORRECT
4. It is a gut Hormone
SOLUTION
Drug ending with GLUTIDE :- Glucagon like peptide (GLP) receptor agonists.
 GLP-1 agonist --> Exenatide/ Liraglutide
 GLP-2 agonist --> Teduglutide
GLP-1:-
 Increase Insulin release
 Decrease gastric motility.
GLP-2 :-
 Used for Short Bowel Syndrome to increase gastric emptying time.
 It is the hormone released by GI Tract not pancreas.
REFERENCE
Goodman & Gilman’s the pharmacological basis of therapeutics- 13th
edition/page- 939
Q. 98
Hypertension and pulmonary edema associated with scorpion sting is managed by :-
1. Carvedilol
2. Prazosin
CORRECT
3. Spironolactone
4. Phentolamine
SOLUTION
 Scorpion sting cause excessive stimulation of alpha 1 receptor. Of the above
mentioned drug:- Prazosin acts by blocking alpha 1 receptor. Therefore drug of
choice for Scorpion sting is Prazosin.
 Carvedilol : Non selective β antagonist that also blocks alpha adrenoreceptor
 Spironolactone: Direct antagonist of aldosterone, thereby preventing salt retention,
myocardial hypertrophy and Hypokalemia.
 Phentolamine : produce a competitive block of alpha1 and alpha 2 receptor.
REFERENCE
Lippincott pharmacology 6th edition page 96/143
Q. 99
Mannitol is used in the management of:-
1. Congestive cardiac failure
2. Acute congestive glaucoma
CORRECT
3. Acute renal failure
4. Pulmonary edema
SOLUTION
Mannitol:-
Uses:- Cerebral edema
Acute Congestive Glaucoma
CI:- Cerebral Edema due to Haemorrhage
Pulmonary Edema
Acute Renal Failure
REFERENCE
Lippincott pharmacology 6th edition page 265
Q. 100
Drug inhibiting granulocyte migration is:-
1. Montelukast
2. Cromoglycate
3. Colchicine
CORRECT
4. Felbamate
SOLUTION
Colchicine : used for the treatment of acute gouty attack. Binds to tubulin, a microtubular
protein, causing it’s depolymerization. This disrupts cellular function such as inability of
granulocyte, thus deceasing their migration into affected area. Furthermore, colchicine block cell
division by binding to mitotic spindle.
Montelukast/ Zafirlukast : Blocks leukotriene receptor.
Felbamate : Broad spectrum of anticonvulsant action with multiple mechanism including
blocking of voltage depended sodium channel, blocking calcium channel and potentiating GABA
action. It is reserved for use in refractory epilepsies particularly Lennox- Gastaut Syndrome
because of the risk of aplastic anemia and hepatic failure.
Sodium cromoglycate :- Inhibit degranulation of mast cell (Mast cell Stabilizer).
REFERENCE
Lippincott pharmacology 6th edition page 477
Q. 101
Drug capable of causing ocular hypotension with apnoea in infants is:-
1. Acetazolamide
2. Latanoprost
3. Brimonidine
CORRECT
4. Apraclonidine
SOLUTION
Brimonidine is contraindicated in infant <2 yo because it cause brain suppression leading to
apnea.
Apraclonidine and Brimonidine both are topical alpha adrenergic agonist which decrease
aqueous humor production and increase aqeous outflow. Apraclonidine cause lid retraction.
Latanoprost :- PGF2alpha
 P- Pigmentation of Iris
 G-Growth of eyelash (Hypertrichosis)
 F2x-fluid in macula (Macular edema)
REFERENCE
Lippincott pharmacology 6th edition page 101
Q. 102
Inhalational anesthetic with highest respiratory irritation is:-
1. Desflurane
CORRECT
2. Nitrous oxide
3. Sevoflurane
4. Halothane
SOLUTION
 Max respiratory irritation:- Desflurane
 MAC of Nitrous Oxide: >104%
 Smooth inducing agent, preferred in children : Sevoflurane
 Halothane:- Sensitize heart to arrythmogenic agent( Eg:- Ardenaline). Therefore
contraindicated in pheochromocytoma
REFERENCE
KD Tripathi Essentials of Medical Pharmacology 8th edition(page-413)
Q. 103
Which of the following drug is used for smoking cessation?
1. Varenicline
CORRECT
2. Acamprostate
3. Nalmefene
4. Gabapentim
SOLUTION
DOC for smoking cessation : Varenicline
Nicotine patch and gum are also used. Bupropion is also used in smoking cessation.
Varenicline: is partial agonist at neuronal nicotinic acetylcholine receptor in CNS. Patient taking
varenicline should be monitored for suicidal thoughts, vivid nightmares and mood changes.
Acamprosate : agent used in alcohol dependent treatment programs with an as yet poorly
understood mechanism of action.
Gabapentin: analog of GABA: Does not act at GABA receptor. It’s precise mechanism is not
known. It is approved as adjunct therapy for focal seizure and treatment of post herpetic
neuralgic.
Nalmefene :- is opioid antagonist use for opioid poisoning
REFERENCE
Lippincott pharmacology 6th edition page 215/218
Q. 104
Which of the following statements about pegfilgrastim is true?
1. It decrease the risk of neutropenia in patients on anti cancer therapy
CORRECT
2. It is a short acting drug
3. It can be given orally
4. It is recombinant GM CSF
SOLUTION
PEGFILGRASTIM:-
 Long acting granulocyte colony stimulating factor (GCSF).
 It is used along with anticancer drug to decrease neutropenia.
 Commonly given by SC route.
Drug ending with GRASTIM are granulocytes stimulator. Drug ending with GRAMOSTIM are
granulocyte monocyte stimulator. Eg:- SARGRAMOSTIM.
REFERENCE
Katzung's Basic & Clinical Pharmacology- 14th edition/page 603
Q. 105
Agent used for eliciting diagnostic differentiation of Myasthenia Gravis from Cholinergic crisis
is:-
1. Ecothiophate
2. Edrophonium
CORRECT
3. Neostigmine
4. Ambenonium
SOLUTION
- Myasthenia Gravis
 In Myasthenia Gravis there are auto antibodies directed against the Nm receptors.
 This causes weakness and easy fatigability , with recovery after rest.
- Edrophonium which is Acetyl cholinesterase inhibitor if given to patient with MG , a short-
lasting improvement in the strength of affected muscles occurs only in myasthenia gravis and not
in other muscular dystrophies. This is known as Ameliorative test.
- Whereas, if the patient is of Cholinergic crisis edrophonium will further worsens the symptoms.
So it can be used to differentiate MG and cholinergic crisis.
REFERENCE
KD Tripathi Essentials of Medical Pharmacology 8th edition/page 121
Q. 106
Which of the following anesthetic agent is not painful on intravenous administration?
1. Ketamine
CORRECT
2. Propofol
3. Etomidate
4. Methohexital
SOLUTION
Ketamine: Short acting, non barbiturate anaesthetic induces a dissociated state in which the
patient is unconcious and does not feel pain. This dissociative anaesthesia provide sedation,
amnesia and immobility. Not painful on IV administration
Propofol: potent anaesthetic but a weak analgesic . Maximum pain on injection.
Etomidate:- Hypnotic agent used to induce anaesthetic but lacks analgesic activity. Usually only
used for patient with coronary artery diseases or cardiovascular dysfunction.
Methohexital:- Barbiturate derivative, short acting and has rapid onset of action.
REFERENCE
Lippincott pharmacology 6th edition page 171/178
Q. 107
Which IV anesthetic not cause cardiac depression?
1. Etomidate
CORRECT
2. Propofol
3. Methohexital
4. Thiopentone
SOLUTION
Etomidate:-
 Preferred inducing agent in cardiovascular surgery
 Major disadvantage is adrenal suppression
Thiopental:-
 Ultrashort acting barbiturate with high lipid solubility.
 Potent anaesthetic but weak analgesic.
Methohexital & Thiopental - Histamine release
REFERENCE
KD Tripathi Essentials of Medical Pharmacology 8th edition(page409)
Q. 108
Mechanism of action of curare like drugs is:-
1. Competitive, Non-depolarizing block at the Nm cholinergic receptors
CORRECT
2. Noncompetitive, Non depolarizing block at the Nm cholinergic receptors
3. Competitive, depolarizing block at the Nm cholinergic receptors
4. Noncompetitive, depolarizing block at the Nm cholinergic receptors
SOLUTION
Neuromuscular Blocker
Deploarising Non Competitive Non - Dep/Competitive
Succinyl Choline
Atracurium
Cis-atracurium
Mivacurium
Pancuronium
Pipecuronium
Vacuronium
REFERENCE
Lippincott pharmacology 6th edition page 70
Q. 109
Reason for preferring cis-Atracurium over Atracurium is:-
1. Faster acting than Atracurium
2. Lesser provocation of histamine release
CORRECT
3. Shorter action than atracurium
4. Does not undergo Hoffman elimination
SOLUTION
Cis-atracurium and Atracurium both are NMJ blocking agent. Cis atracurium has same
pharmacokinetic properties as atracurium. Atracurium release histamine and is metabolized to
Laudonasine, which can provoke seizures. Both undergo Hoffman elimination.
REFERENCE
Lippincott pharmacology 6th edition
Q. 110
Drug of choice for Digoxin induced Ventricular Tachycardia:-
1. Propranolol
2. Dilitiazem
3. Verapamil
4. Lignocaine
CORRECT
SOLUTION
Lignocaine:- DOC in Digoxin Induced Ventricular Tachycardia.
DOC in post Mi Ventricular Tachycardia.
It is class IB Ant arrythmia drug.
Propranolol:- Non selective antagonist, having both negative inotropic and chronotropic effect It
doesnot reduce blood pressure in people with (N) Blood pressure. It lower BP in Hypertension.
Verapamil and diltiazem should be avoided in patient with heart failure or with AV block due to
their negative inotropic and dromotropic
REFERENCE
Lippincott pharmacology 6th edition page 69
Q. 111
Which of the following is a contraindication to the use of Beta Blockers:-
1. Glaucoma
2. Tachycardia
3. Bronchial asthma
CORRECT
4. Hypertension
SOLUTION
• Contraindication of Blocker:-
1. Asthma
2. AV Block
3. Acute CHF
4. Diabetes Mellitus
• ß Blocker uses:-
 Hypertension
 Migraine
 Hyperthyroidism
 Angina pretoirs
 MI
REFERENCE
Lippincott pharmacology 6th edition page 99
Q. 112
Screening test, following genetic counselling, in a family with familial adenomatous polyposis
is:-
1. APC gene testing
CORRECT
2. Occult blood in stool
3. Flexible sigmoidoscopy
4. Colonoscopy
Q. 113
Identify the nerve supply of the marked muscle:-
1. Radial nerve
2. Median nerve
CORRECT
3. Ulnar nerve
4. Anterior interosseous nerve
SOLUTION
• Radial nerve → extensor compartment
• Median nerve → 1st and 2nd lumbricals
• Ulnar nerve → 3rd and 4th lumbricals
• Anterior interosseous nerve → Flexor digitorum profundus (lateral half), Flexor pollicis longus,
Pronator quadratus
REFERENCE
Gray’s anatomy,2nd south Asian edition, pg. 204-205
Q. 114
Tenosynovitis of flexor tendon. What is the correct option?
1. The affected finger is extended at all joints
2. Every case is surgically managed .
3. Tenderness along tendon sheath .
CORRECT
4. Patient present with minimal pain
SOLUTION
Tenosynovitis
 Very painful condition
 The cardinal signs of flexor sheath infection are:
o Finger held in slight flexion
o Fusiform swelling of the affected digit
o Tenderness along the flexor tendon sheath
o Pain with passive extension of the digit
 Treatment-
o Conservative management
o Surgery (closed method)
REFERENCE
Apley & Solomon’s, 10th edition: pg.449
Q. 115
12 years male came with swelling of lower end tibia which is surrounded by rim of reactive
bone. What is most likely diagnosis?
1. GCT
2. Brodie's Abscess
CORRECT
3. Hyper PTH
4. Osteomyelitis
SOLUTION
• GCT – occurs around 20-40 years of age
• Hyperparathyroidism – typically in a female around 30 years of age; presents with lytic lesion
• Brodie’s abscess – swelling in the lower end of tibia and radiological evidence of cavity
surrounded by a halo of sclerosis (rim of a reactive bone) point towards brodie’s abscess.
REFERENCE
Apley & Solomon’s, 10th edition: pg.39
Q. 116
A patient with GCT which of the following is false:-
1. Epiphyseo-metaphyseal location
2. Eccentric
3. Defined margins
4. Chemotherapy is the mainstay of treatment
CORRECT
SOLUTION
• GCT occurs in epiphyseao-metaphyseal area.
• It is eccentric in location and goes till the joint.
• It is a locally aggressive tumor with defined margins.
• The treatment of choice is extended curettage with autograft.
REFERENCE
Apley & Solomon’s, 10th edition: pg.198-199
Q. 117
75 yrs female has chronic backache, x-ray spine is shown. What is the most likely diagnosis:-
1. Pott's spine
2. Osteoporosis
CORRECT
3. Spondylolisthesis
4. Spondylodiscitis
SOLUTION
• Spondylodiscitis – Inflammation of disc along with paradiscal
margin and shows slight narrowing the disc space at first in X-rays. It
is a very painful condition.
• Pott’s spine – occurs in paradiscal area. X-ray shows narrowing of
the disc space and erosion of adjacent vertebral bodies.
• Spondylolisthesis – Slipping of one vertebra into other. Oblique X-
ray view demonstrates “Scotty Dog Neck” which is pathognomonic
Osteoporosis – Patient’s age and the chronic condition points towards
osteoporosis. The given X-ray shows Cod fish / fish mouth vertebrae
which is characteristic of osteoporosis.
REFERENCE
Apley & Solomon’s, 10th edition: pg.480-481,522
Q. 118
A 70 yrs male patient has single well defined lytic lesion of skull. The patient had no other
complaint and urine examination had no abnormality. What is the most likely diagnosis?
1. L.C.H
CORRECT
2. Localized myeloproliferative disorder
3. Generalized myeloproliferative disorder
4. Tumor of osteoblasts
SOLUTION
Langerhans’s cell histiocytosis
 X-ray- well-demarcated lytic lesion, “hole within a hole”.
 M/E - “Tennis-racket” inclusion bodies and Birbeck granules
Plasmacytoma
 Localized myeloproliferative disorder, does not present with lytic lesions in the skull.
Generalized myeloproliferative disorder is multiple myeloma which presents with characteristic
“punched-out” lytic lesions in the skull. Serum or urinary electrophoresis measures
immunoglobulins over-produced by the plasma cells.
Lytic lesions of skull:
 M- Metastasis
 E- EG(eosinophilic granuloma)
 L- LCH/Lymphoma
 T-T.B
 H-Hyperparathyroidism
 O- Osteomyelitis
 R- RT(Radiotherapy)
 M- MM(Multiple myeloma)
 E- Epidermoid
REFERENCE
Apley & Solomon’s, 10th edition: pg.250,214
Q. 119
Foot drop is caused due to injury to?
1. Common peroneal nerve
CORRECT
2. Tibial Nerve
3. Femoral Nerve
4. Obturator Nerve
SOLUTION
Foot drop = it occurs due to injury to common peroneal nerve which usually gets injured at the
neck of fibula.
Femoral nerve injury – unable to extend the knee actively.
Tibial nerve injury – unable to plantar flex the ankle or foot and no sensation over the sole and
part of calf.
REFERENCE
Apley & Solomon’s, 10th edition: pg.299-302
Q. 120
Image of dorsum of hand, which nerve gives sensory supply to this region:-
1. PIN
2. Radial
CORRECT
3. Median
4. Ulnar
SOLUTION
REFERENCE
Gray’s anatomy,2nd south Asian edition, pg.44
Q. 121
Scaphoid fracture which area has maximum chances of AVN/Non-union/Malunion:-
1. Proximal 1/3
CORRECT
2. Middle 1/3
3. Distal 1/3
4. Scaphoid Tubercle fractrue
SOLUTION
• Scaphoid has retrograded blood supply (distal to proximal) hence the distal pole is highly
vascularized.
• Due to fracture at the waist of scaphoid, blood supply to proximal pole is compromised. Hence
proximal 1/3rd has the maximum chance to undergo AVN/non-union or malunion.
REFERENCE
Apley & Solomon’s, 10th edition: pg.806
Q. 122
Painful arc syndrome which movement is painful:-
1. Initial abduction
2. Terminal abduction
3. Mid abduction
CORRECT
4. Full range of abduction
SOLUTION
• Painful arc syndrome is subacute tendinitis of the rotator cuff tendons due to vascular
congestion.
• Treatment is usually rest +physiotherapy + NSAIDS. If not treated, local steroids and surgery
can be done.
• Pain is aggravated as arm transverses an arc between 60 and 120 degrees.
REFERENCE
Apley & Solomon’s, 10th edition: pg.357-358
Q. 123
The most likely diagnosis for the tumor at upper end of tibia is:-
1. GCT
CORRECT
2. UBC
3. ABC
4. CB
SOLUTION
• Giant cell tumor – It is the only tumor to involve the joint. It is seen in 20-40 years of age
(after skeletal maturity). It occurs in epiphysis (mainly) and extends into metaphysis, so called as
epiphyseal-metaphysis tumor
• Unicameral bone cyst –Unilocular cystic bone cavity filled with serosanguinous fluid. X-ray
shows fallen leaf sign. It often abuts but does not cross the physis.
• Aneurysmal bone cyst –Blood-filled cystic spaces. X-ray shows expansile, lytic cystic lesions
and MRI shows fluid-fluid levels within the lesion.
• Chondroblastoma – presence of “chicken-wire” calcification is pathognomonic.
REFERENCE
Apley & Solomon’s, 10th edition: pg.195-202
Q. 124
Left-Right movement of skull occurs at:-
1. Atlento occipital joint
2. Atlanto-axial joint
CORRECT
3. C2-C3
4. C6-C7
SOLUTION
 Atlanto-occipital joint – flexion and extension of neck."yes joint"
 Atlanto-axial joint (C1 -C2) movements -side to side movements of head (“no joint”) i.e
left and right movements of skull.
 Atlanto-occipital joint Atlanto-axial joint (C1 -C2) movements
REFERENCE
Apley & Solomon’s, 10th edition: pg.456
Q. 125
Degloving injury occurs at:-
1. Skin
2. Skin + Subcutaneous tissue
CORRECT
3. Skin + Fascia + Subcutaneous tissue
4. Everything above from bone
SOLUTION
Degloving injury – it is the separation of skin and subcutaneous tissues from the underlying
bone, muscle or vasculature.
Degloving occurs when the skin and subcutaneous fat are stripped by avulsion from the
underlying fascia, leaving neuro vascular structures, tendon or bone exposed.
REFERENCE
Bailey & Love,27th edition, pg.27
Q. 126
14 years old child with Naive Rheumatoid Arthritis patient (image of both hands give) what is
treatment?
1. Dmards after initial 3 months of NSAIDs
2. Only NSAID
3. Dmards with short course of steroids
CORRECT
4. Monotherapy with TNF drugs
SOLUTION
• Treatment should be aimed at controlling the inflammation as rapidly as possible. This is likely
to require the use of corticosteroids for their rapid onset. In addition, disease-modifying
antirheumatic drugs (DMARDS) should be started at this time.
• Control of pain and stiffness with NSAIDS may be needed.
• If there is no satisfactory response to DMARDS, it’s wise to progress to TNF therapies such as
Infliximab.
REFERENCE
Apley & Solomon’s, 10th edition: pg.71
Q. 127
Caspase involved in activation of IL_1 is which of the following?
1. Caspase 1
CORRECT
2. Caspase 3
3. Caspase 5
4. Caspase 8
SOLUTION
Pyroptosis :-
 Example of process in which injury is initiated because of exposure to microbes.
Whenever there is exposure to microbes there is activation of enzyme caspase 1 and
caspase 11.
 Caspase 1 convert inactive form of IL 1 to active form of IL1. As IL1 has central action,
it enters CNS and results in development of fever.
 Caspase-1 along with closely related caspase-11 also cause death of the infected cell.
→Caspase 3, a typical executioner caspase that cleaves DNA and other substrates to cause cell
death.
REFERENCE
Robbins and Cotran Pathology 10th E Page 302
Q. 128
Which of the following is positive in Follicular lymphoma?
1. Bcl 2
CORRECT
2. Bcl 6
3. Bcl 1
4. Bcl 10
SOLUTION
Follicular lymphoma likely arises from germinal center B cells and is strongly associated
with chromosomal translocations involving BCL2. Its hallmark is a (14;18) translocation that
juxtaposes the IGH locus on chromosome 14 and the BCL2 locus on chromosome 18. The
t(14;18) is seen in up to 90% of follicular lymphomas, and leads to overexpression of BCL2.
BCL2 antagonizes apoptosis and promotes the survival of follicular lymphoma cells. Notably,
while normal germinal centers contain numerous B cells undergoing apoptosis, follicular
lymphoma is characteristically devoid of apoptotic cells.
BCL 10 : associated with extra nodal marginal zone lymphoma The t(14;18)(q32;q21) and
t(1;14) (p22;q32) translocations cause increased expression of intact MALT1 and BCL-10
protein respectively. This translocation is associated with gastric MALToma. Each of the three
translocations has the same net effect, the constitutive activation of NF-κB, a transcription factor
that promotes B-cell growth and survival. Antigen dependent activation of NF-κB in normal B
and T cells. requires both BCL-10 and MLT, which work together in a pathway downstream of
the B- and T-cell antigen receptors. Thus, H. pylori–induced inflammation may trigger NF-κB
activation through the MLT/BCL-10 pathway in MALTomas that lack these translocations.
Removal of this stimulus may explain why these tumors tend to respond to H. pylori eradication.
In contrast, NF-κB is constitutively active in tumors bearing translocations involving MLT or
BCL10, and H. pylori treatment is ineffective.
BCL 6 : associated with diffuse large b cell lymphoma(DLBCL). DLBCL is most common
subtype of non Hodgkin lymphoma globally.
BCL 1 : also called cyclin D1 and is associated with Mantle Cell Lymphoma
REFERENCE
Robbins and Cotran Pathology : Page 773
Q. 129
Vitamin A is stored in:-
1. Hepatocyte
2. Ito cell
CORRECT
3. Hepatic endothelial cell
4. Kupffer cell
SOLUTION
Vitamin A is stored in Ito cell
The principal cell type involved in scar deposition is the hepatic stellate cell.
 In its quiescent form, it is a lipid (vitamin A) storing cell. However, in several forms of
acute and chronic injury, the stellate cells( ito) can become activated and are converted
into highly fibrogenic myofibroblasts.
REFERENCE
Robbins and Cotran Pathology 9th ed: Pg 823
Q. 130
Glanzmann thromasthenia is due to defect in:-
1. Gp IIb/IIIa
CORRECT
2. Gp Ib-IX
3. CD68
4. Von Willebrand factor
SOLUTION
The conformational change in glycoprotein IIb/IIIa that occurs with platelet activation allows
binding of fibrinogen, a large bivalent plasma polypeptide that forms bridges between adjacent
platelets, leading to their aggregation. Predictably, inherited deficiency of GpIIb-IIIa results in a
bleeding disorder called Glanzmann thrombasthenia.
Bernard-Soulier syndrome illustrates the consequences of defective adhesion of platelets to
subendothelial matrix. Bernard-Soulier syndrome is caused by an inherited deficiency of the
platelet membrane glycoprotein Complex Ib-IX. This glycoprotein is a receptor for vWF and is
essential for normal platelet adhesion to the subendothelial extracellular matrix. Affected patients
have a variable, often severe, bleeding tendency.
Factor VIII and vWF are encoded by separate genes and are synthesized in different cells. Factor
VIII is an essential cofactor of factor IX, which converts factor X to factor Xa. It is made in
several tissues; sinusoidal endothelial cells and Kupffer cells in the liver seem to be major
sources.
Type 1 and type 3 von Willebrand disease are associated with quantitative defects in vWF. Type
1, an autosomal dominant disorder characterized by a mild to moderate vWF deficiency,
accounts for about 70% of all cases. Type 2 von Willebrand disease is characterized by
qualitative defects in vWF.
REFERENCE
Robbins and Cotran Pathology Page : 662
Q. 131
Which of the following is an antiapoptotic gene?
1. Bcl2
CORRECT
2. Bcl- XS
3. BAX
4. BAD
SOLUTION
Antiapoptotic genes are:
 BCL-2
 BCL-XL
 MCL-1

o These genes possess four BH domains (called BH1-4).
o These proteins reside in the outer mitochondrial membranes as well as the cytosol
and ER membranes.
Proapoptotic genes are:
 BAX
 BAK

o They also have four BH domains.
o Upon activation, BAX and BAK oligomerize within the outer mitochondrial
protein and promote mitochondrial outer membrane permeability.
Sensor genes are:
 BAD
 BIM
 BID
 Puma
 Noxa

o They contain only one BH domain, the third of the four BH domains, and hence
are sometimes called BH3-only proteins.
o These proteins act as sensors of cellular stress and damage, and regulate the
balance between the other two groups, thus acting as arbiters of apoptosis.
REFERENCE
Robbins and Cotran Pathology 9th edition pg 52
Q. 132
Which of the following is true about alpha 1 antitrypsin?
1. Inhibits elastase
CORRECT
2. Inhibits trypsinogen activation in pancreas
3. Inhibits trypsin activating protease
4. Inhibits chymotrypsin
SOLUTION
Alpha 1 antitrypsin : anti elastase protein coded by gene PiMM gene.
Any mutation in this gene lead to abnormal formation of alpha 1 antitrypsin protein which leads
to
1. Storage of this abnormal enzyme inside hepatocyte instead of it’s secretion resulting in
manifestation of micronodular cirrhosis of liver and
2. Pan acinar emphysema in basal part of lung.
REFERENCE
Robbin 10th edition Page : 498
Q. 133
Which of the following is autosomal recessive disorder?
1. Huntington's chorea
2. Neurofibromatosis 1
3. Marfan's syndrome
4. Sickle cell anemia
CORRECT
SOLUTION
Autosomal Dominant (A Very DOMINANT
Hereditary Family )
 Achondroplasia/ADPKD
 Von Hipple Lindau disease
 Dystrophia myotonica
 Osteogenesis imperfecta
 Marfan Syndrome
 Intermittent porphyria
 NF 1(von reckinghausen disease)
 Achondroplasia
 NF 2
 Tuberous Sclerosis
 Huntington Disease
 Familial hypercholesterolemia
Autosomal Recessive (ABCDEFGH’S)
 Albinism, Ataxia Telengiactasia,
Alkaptonuria
 Beta thalassemia
 Cystic Fibrosis
 Dubin Johnson
 Emphysema(alpha 1 antitrypsin
deficiency)
 Friedreich Ataxia
 Galactosemia
 Hemochromatosis
 Sickle cell anemia
REFERENCE
ROBBINS AND CORTON 10e/172
Q. 134
Fish mouth stenosis in rheumatic heart disease is due to which of the following mechanisms?
1. Calcification and fibrosis bridging across valvular commissures
CORRECT
2. Fibrinoid necrosis
3. Acute inflammation leading to valvular damage
4. Myxomatous degeneration of the valve
SOLUTION
Fish mouth stenosis in RHD is manifestation of Chronic RHD involving mitral valve which is
due to calcification and fibrosis bridging across valvular commissures. RHD is virtually the only
cause of mitral stenosis.
Acute and chronic rheumatic heart disease.
A) Acute rheumatic mitral valvulitis superimposed on chronic rheumatic heart disease. Small
vegetations (verrucae) are visible along the line of closure of the mitral valve leaflet (arrows).
Previous episodes of rheumatic valvulitis have caused fibrous thickening and fusion of the
chordae tendineae.
B) Microscopic appearance of an Aschoff body in a patient with acute rheumatic carditis. The
myocardium exhibits a circumscribed nodule of mixed mononuclear inflammatory cells with
associated necrosis; within the inflammation, large activated macrophages show prominent
nucleoli, as well as chromatin condensed into long, wavy ribbons (caterpillar cells; arrows).
C/D). Mitral stenosis with diffuse fibrous thickening and distortion of the valve leaflets and
commissural fusion (arrows, C), and thickening of the chordae tendineae (D). Note
neovascularization of anterior mitral leaflet (arrow, D).
E). Surgically resected specimen of rheumatic aortic stenosis, demonstrating thickening and
distortion of the cusps with commissural fusion.
Comparison of the four major forms of vegetative endocarditis. The rheumatic fever phase of
rheumatic heart disease (RHD) is marked by small, warty vegetations along the lines of closure
of the valve leaflets. Infective endocarditis (IE) is characterized by large, irregular masses on the
valve cusps that can extend onto the chordae. Nonbacterial thrombotic endocarditis (NBTE)
typically exhibits small, bland vegetations, usually attached at the line of closure. One or many
may be present. Libman-Sacks endocarditis (LSE) has small or medium-sized vegetations on
either or both sides of the valve leaflets.
REFERENCE
Robbins and Cotran Pathology 9th edition Page : 558
Q. 135
Which of the following feature distinguishes Crohn's disease from Ulcerative colitis ?
1. Transmural involvement
CORRECT
2. Presence of polyps
3. Mucosal edema
4. Lymphocyte infiltrate
SOLUTION
→ Crohn's disease is associated with transmural inflammation whereas only superficial layers
[mucosa, submucosa] are involved in ulcerative colitis.
Feature Crohn Disease Ulcerative Colitis
Macroscopic
Bowel region involved Any site of GIT Colon only
Distribution Skip lesions Diffuse
Stricture Yes Rare
Wall appearance Thick Thin
Microscopic
Inflammation Transmural Limited to mucosa, submucosa
Pseudopolyps may/ may not be present Markedly present
Ulcers Deep, knife-like Superficial, broad-based
Lymphoid reaction Marked Moderate
Fibrosis Marked Mild to none
Serositis Marked Mild to none
Granulomas Yes ( 35%) [non-caseating] No
Fistulae/sinuses Yes No
Clinical
Perianal fistula Yes (in colonic disease) No
Fat/vitamin malabsorption Yes No
Malignant potential With colonic involvment yes Yes
Recurrence after surgery Common No
Toxic megacolon No Yes
All feature may not be present in a single case.
REFERENCE
Robbins 10th edition Page : 621-625
Q. 136
Which of the following is a cause of intravascular hemolysis?
1. Warm type autoimmune hemolytic anemia
2. PNH
CORRECT
3. Cold agglutinin disease
4. Hereditary spherocytosis
SOLUTION
→ Causes of intravascular hemolysis are:
NEET PG 2019.docx
NEET PG 2019.docx
NEET PG 2019.docx
NEET PG 2019.docx
NEET PG 2019.docx
NEET PG 2019.docx
NEET PG 2019.docx
NEET PG 2019.docx
NEET PG 2019.docx
NEET PG 2019.docx
NEET PG 2019.docx
NEET PG 2019.docx
NEET PG 2019.docx
NEET PG 2019.docx
NEET PG 2019.docx
NEET PG 2019.docx
NEET PG 2019.docx
NEET PG 2019.docx
NEET PG 2019.docx
NEET PG 2019.docx
NEET PG 2019.docx
NEET PG 2019.docx
NEET PG 2019.docx
NEET PG 2019.docx
NEET PG 2019.docx
NEET PG 2019.docx
NEET PG 2019.docx
NEET PG 2019.docx
NEET PG 2019.docx
NEET PG 2019.docx
NEET PG 2019.docx
NEET PG 2019.docx
NEET PG 2019.docx
NEET PG 2019.docx
NEET PG 2019.docx
NEET PG 2019.docx
NEET PG 2019.docx
NEET PG 2019.docx
NEET PG 2019.docx
NEET PG 2019.docx
NEET PG 2019.docx
NEET PG 2019.docx
NEET PG 2019.docx
NEET PG 2019.docx
NEET PG 2019.docx
NEET PG 2019.docx
NEET PG 2019.docx
NEET PG 2019.docx
NEET PG 2019.docx
NEET PG 2019.docx
NEET PG 2019.docx
NEET PG 2019.docx
NEET PG 2019.docx
NEET PG 2019.docx
NEET PG 2019.docx
NEET PG 2019.docx
NEET PG 2019.docx
NEET PG 2019.docx
NEET PG 2019.docx
NEET PG 2019.docx
NEET PG 2019.docx
NEET PG 2019.docx
NEET PG 2019.docx
NEET PG 2019.docx
NEET PG 2019.docx
NEET PG 2019.docx
NEET PG 2019.docx
NEET PG 2019.docx
NEET PG 2019.docx
NEET PG 2019.docx
NEET PG 2019.docx
NEET PG 2019.docx
NEET PG 2019.docx
NEET PG 2019.docx
NEET PG 2019.docx
NEET PG 2019.docx
NEET PG 2019.docx
NEET PG 2019.docx
NEET PG 2019.docx
NEET PG 2019.docx
NEET PG 2019.docx
NEET PG 2019.docx
NEET PG 2019.docx
NEET PG 2019.docx
NEET PG 2019.docx
NEET PG 2019.docx
NEET PG 2019.docx
NEET PG 2019.docx
NEET PG 2019.docx
NEET PG 2019.docx
NEET PG 2019.docx
NEET PG 2019.docx
NEET PG 2019.docx
NEET PG 2019.docx
NEET PG 2019.docx
NEET PG 2019.docx
NEET PG 2019.docx
NEET PG 2019.docx
NEET PG 2019.docx
NEET PG 2019.docx
NEET PG 2019.docx
NEET PG 2019.docx
NEET PG 2019.docx
NEET PG 2019.docx
NEET PG 2019.docx
NEET PG 2019.docx
NEET PG 2019.docx
NEET PG 2019.docx
NEET PG 2019.docx
NEET PG 2019.docx
NEET PG 2019.docx
NEET PG 2019.docx
NEET PG 2019.docx
NEET PG 2019.docx
NEET PG 2019.docx
NEET PG 2019.docx
NEET PG 2019.docx
NEET PG 2019.docx
NEET PG 2019.docx
NEET PG 2019.docx
NEET PG 2019.docx
NEET PG 2019.docx
NEET PG 2019.docx
NEET PG 2019.docx
NEET PG 2019.docx
NEET PG 2019.docx
NEET PG 2019.docx
NEET PG 2019.docx
NEET PG 2019.docx
NEET PG 2019.docx
NEET PG 2019.docx
NEET PG 2019.docx
NEET PG 2019.docx
NEET PG 2019.docx
NEET PG 2019.docx
NEET PG 2019.docx
NEET PG 2019.docx
NEET PG 2019.docx
NEET PG 2019.docx
NEET PG 2019.docx
NEET PG 2019.docx
NEET PG 2019.docx
NEET PG 2019.docx
NEET PG 2019.docx
NEET PG 2019.docx
NEET PG 2019.docx
NEET PG 2019.docx
NEET PG 2019.docx
NEET PG 2019.docx
NEET PG 2019.docx
NEET PG 2019.docx
NEET PG 2019.docx
NEET PG 2019.docx
NEET PG 2019.docx
NEET PG 2019.docx
NEET PG 2019.docx
NEET PG 2019.docx
NEET PG 2019.docx
NEET PG 2019.docx
NEET PG 2019.docx
NEET PG 2019.docx
NEET PG 2019.docx
NEET PG 2019.docx
NEET PG 2019.docx
NEET PG 2019.docx
NEET PG 2019.docx
NEET PG 2019.docx
NEET PG 2019.docx
NEET PG 2019.docx
NEET PG 2019.docx
NEET PG 2019.docx
NEET PG 2019.docx
NEET PG 2019.docx
NEET PG 2019.docx
NEET PG 2019.docx
NEET PG 2019.docx
NEET PG 2019.docx
NEET PG 2019.docx
NEET PG 2019.docx
NEET PG 2019.docx
NEET PG 2019.docx
NEET PG 2019.docx
NEET PG 2019.docx
NEET PG 2019.docx
NEET PG 2019.docx
NEET PG 2019.docx
NEET PG 2019.docx
NEET PG 2019.docx
NEET PG 2019.docx
NEET PG 2019.docx
NEET PG 2019.docx
NEET PG 2019.docx
NEET PG 2019.docx
NEET PG 2019.docx
NEET PG 2019.docx
NEET PG 2019.docx
NEET PG 2019.docx
NEET PG 2019.docx
NEET PG 2019.docx
NEET PG 2019.docx
NEET PG 2019.docx
NEET PG 2019.docx
NEET PG 2019.docx
NEET PG 2019.docx
NEET PG 2019.docx
NEET PG 2019.docx
NEET PG 2019.docx

More Related Content

Similar to NEET PG 2019.docx

Amino acids degradation II
Amino acids degradation IIAmino acids degradation II
Amino acids degradation IIKshema Thakur
 
Principles of Hormonal regulations
Principles of Hormonal regulationsPrinciples of Hormonal regulations
Principles of Hormonal regulationsEneutron
 
Treatment of other anemias
Treatment  of other anemiasTreatment  of other anemias
Treatment of other anemiasRudhra Prabhakar
 
Plasmaproteins 120711133728-phpapp01
Plasmaproteins 120711133728-phpapp01Plasmaproteins 120711133728-phpapp01
Plasmaproteins 120711133728-phpapp01DrShamimAkram
 
Lec,7-plasma proteins
Lec,7-plasma proteinsLec,7-plasma proteins
Lec,7-plasma proteinsShamim Akram
 
Carbohydrate Metabolism Part 1
Carbohydrate Metabolism Part 1Carbohydrate Metabolism Part 1
Carbohydrate Metabolism Part 1ShibleeZaman
 
Inborn error of metabolism
Inborn error of metabolism Inborn error of metabolism
Inborn error of metabolism Vishakha Sharma
 
PLASMA PROTEINS
PLASMA PROTEINSPLASMA PROTEINS
PLASMA PROTEINSYESANNA
 
Hepatocellular carcinoma
Hepatocellular carcinomaHepatocellular carcinoma
Hepatocellular carcinomaneralagundi
 
ENZYMES INHERITED ENZYMOPATHIES. APPLICATION OF ENZYME IN THE TREATMENT OF DI...
ENZYMES INHERITED ENZYMOPATHIES. APPLICATION OF ENZYME IN THE TREATMENT OF DI...ENZYMES INHERITED ENZYMOPATHIES. APPLICATION OF ENZYME IN THE TREATMENT OF DI...
ENZYMES INHERITED ENZYMOPATHIES. APPLICATION OF ENZYME IN THE TREATMENT OF DI...Dr. Hament Sharma
 
Plasma proteins
Plasma proteinsPlasma proteins
Plasma proteinsYESANNA
 
2nd peroxisome metabolism 20760414
2nd peroxisome metabolism 207604142nd peroxisome metabolism 20760414
2nd peroxisome metabolism 20760414Raveena Ramtel
 
Carbohydrate metabolism
Carbohydrate metabolismCarbohydrate metabolism
Carbohydrate metabolismHaseeb Quadri
 

Similar to NEET PG 2019.docx (20)

Sodium metabolism
Sodium metabolismSodium metabolism
Sodium metabolism
 
Amino acids degradation II
Amino acids degradation IIAmino acids degradation II
Amino acids degradation II
 
Untitled
 Untitled Untitled
Untitled
 
Principles of Hormonal regulations
Principles of Hormonal regulationsPrinciples of Hormonal regulations
Principles of Hormonal regulations
 
Treatment of other anemias
Treatment  of other anemiasTreatment  of other anemias
Treatment of other anemias
 
Plasmaproteins 120711133728-phpapp01
Plasmaproteins 120711133728-phpapp01Plasmaproteins 120711133728-phpapp01
Plasmaproteins 120711133728-phpapp01
 
Lec,7-plasma proteins
Lec,7-plasma proteinsLec,7-plasma proteins
Lec,7-plasma proteins
 
Carbohydrate Metabolism Part 1
Carbohydrate Metabolism Part 1Carbohydrate Metabolism Part 1
Carbohydrate Metabolism Part 1
 
Biochemistry high yield part 2
Biochemistry high yield part 2Biochemistry high yield part 2
Biochemistry high yield part 2
 
Inborn error of metabolism
Inborn error of metabolism Inborn error of metabolism
Inborn error of metabolism
 
Metabolism
MetabolismMetabolism
Metabolism
 
PLASMA PROTEINS
PLASMA PROTEINSPLASMA PROTEINS
PLASMA PROTEINS
 
Hepatocellular carcinoma
Hepatocellular carcinomaHepatocellular carcinoma
Hepatocellular carcinoma
 
Plasma proteins
Plasma proteinsPlasma proteins
Plasma proteins
 
ENZYMES INHERITED ENZYMOPATHIES. APPLICATION OF ENZYME IN THE TREATMENT OF DI...
ENZYMES INHERITED ENZYMOPATHIES. APPLICATION OF ENZYME IN THE TREATMENT OF DI...ENZYMES INHERITED ENZYMOPATHIES. APPLICATION OF ENZYME IN THE TREATMENT OF DI...
ENZYMES INHERITED ENZYMOPATHIES. APPLICATION OF ENZYME IN THE TREATMENT OF DI...
 
Haematinics
HaematinicsHaematinics
Haematinics
 
Plasma proteins
Plasma proteinsPlasma proteins
Plasma proteins
 
2nd peroxisome metabolism 20760414
2nd peroxisome metabolism 207604142nd peroxisome metabolism 20760414
2nd peroxisome metabolism 20760414
 
purine metabolism.ppt
purine metabolism.pptpurine metabolism.ppt
purine metabolism.ppt
 
Carbohydrate metabolism
Carbohydrate metabolismCarbohydrate metabolism
Carbohydrate metabolism
 

Recently uploaded

Fair Trash Reduction - West Hartford, CT
Fair Trash Reduction - West Hartford, CTFair Trash Reduction - West Hartford, CT
Fair Trash Reduction - West Hartford, CTaccounts329278
 
##9711199012 Call Girls Delhi Rs-5000 UpTo 10 K Hauz Khas Whats Up Number
##9711199012 Call Girls Delhi Rs-5000 UpTo 10 K Hauz Khas  Whats Up Number##9711199012 Call Girls Delhi Rs-5000 UpTo 10 K Hauz Khas  Whats Up Number
##9711199012 Call Girls Delhi Rs-5000 UpTo 10 K Hauz Khas Whats Up NumberMs Riya
 
2024: The FAR, Federal Acquisition Regulations - Part 27
2024: The FAR, Federal Acquisition Regulations - Part 272024: The FAR, Federal Acquisition Regulations - Part 27
2024: The FAR, Federal Acquisition Regulations - Part 27JSchaus & Associates
 
Global debate on climate change and occupational safety and health.
Global debate on climate change and occupational safety and health.Global debate on climate change and occupational safety and health.
Global debate on climate change and occupational safety and health.Christina Parmionova
 
EDUROOT SME_ Performance upto March-2024.pptx
EDUROOT SME_ Performance upto March-2024.pptxEDUROOT SME_ Performance upto March-2024.pptx
EDUROOT SME_ Performance upto March-2024.pptxaaryamanorathofficia
 
(DIVYA) Call Girls Wakad ( 7001035870 ) HI-Fi Pune Escorts Service
(DIVYA) Call Girls Wakad ( 7001035870 ) HI-Fi Pune Escorts Service(DIVYA) Call Girls Wakad ( 7001035870 ) HI-Fi Pune Escorts Service
(DIVYA) Call Girls Wakad ( 7001035870 ) HI-Fi Pune Escorts Serviceranjana rawat
 
↑VVIP celebrity ( Pune ) Serampore Call Girls 8250192130 unlimited shot and a...
↑VVIP celebrity ( Pune ) Serampore Call Girls 8250192130 unlimited shot and a...↑VVIP celebrity ( Pune ) Serampore Call Girls 8250192130 unlimited shot and a...
↑VVIP celebrity ( Pune ) Serampore Call Girls 8250192130 unlimited shot and a...ranjana rawat
 
VIP Kolkata Call Girl Jatin Das Park 👉 8250192130 Available With Room
VIP Kolkata Call Girl Jatin Das Park 👉 8250192130  Available With RoomVIP Kolkata Call Girl Jatin Das Park 👉 8250192130  Available With Room
VIP Kolkata Call Girl Jatin Das Park 👉 8250192130 Available With Roomishabajaj13
 
2024: The FAR, Federal Acquisition Regulations - Part 28
2024: The FAR, Federal Acquisition Regulations - Part 282024: The FAR, Federal Acquisition Regulations - Part 28
2024: The FAR, Federal Acquisition Regulations - Part 28JSchaus & Associates
 
Goa Escorts WhatsApp Number South Goa Call Girl … 8588052666…
Goa Escorts WhatsApp Number South Goa Call Girl … 8588052666…Goa Escorts WhatsApp Number South Goa Call Girl … 8588052666…
Goa Escorts WhatsApp Number South Goa Call Girl … 8588052666…nishakur201
 
PPT Item # 4 - 231 Encino Ave (Significance Only)
PPT Item # 4 - 231 Encino Ave (Significance Only)PPT Item # 4 - 231 Encino Ave (Significance Only)
PPT Item # 4 - 231 Encino Ave (Significance Only)ahcitycouncil
 
VIP High Profile Call Girls Gorakhpur Aarushi 8250192130 Independent Escort S...
VIP High Profile Call Girls Gorakhpur Aarushi 8250192130 Independent Escort S...VIP High Profile Call Girls Gorakhpur Aarushi 8250192130 Independent Escort S...
VIP High Profile Call Girls Gorakhpur Aarushi 8250192130 Independent Escort S...Suhani Kapoor
 
Climate change and occupational safety and health.
Climate change and occupational safety and health.Climate change and occupational safety and health.
Climate change and occupational safety and health.Christina Parmionova
 
(NEHA) Bhosari Call Girls Just Call 7001035870 [ Cash on Delivery ] Pune Escorts
(NEHA) Bhosari Call Girls Just Call 7001035870 [ Cash on Delivery ] Pune Escorts(NEHA) Bhosari Call Girls Just Call 7001035870 [ Cash on Delivery ] Pune Escorts
(NEHA) Bhosari Call Girls Just Call 7001035870 [ Cash on Delivery ] Pune Escortsranjana rawat
 
Human-AI Collaboration for Virtual Capacity in Emergency Operation Centers (E...
Human-AI Collaborationfor Virtual Capacity in Emergency Operation Centers (E...Human-AI Collaborationfor Virtual Capacity in Emergency Operation Centers (E...
Human-AI Collaboration for Virtual Capacity in Emergency Operation Centers (E...Hemant Purohit
 

Recently uploaded (20)

Fair Trash Reduction - West Hartford, CT
Fair Trash Reduction - West Hartford, CTFair Trash Reduction - West Hartford, CT
Fair Trash Reduction - West Hartford, CT
 
##9711199012 Call Girls Delhi Rs-5000 UpTo 10 K Hauz Khas Whats Up Number
##9711199012 Call Girls Delhi Rs-5000 UpTo 10 K Hauz Khas  Whats Up Number##9711199012 Call Girls Delhi Rs-5000 UpTo 10 K Hauz Khas  Whats Up Number
##9711199012 Call Girls Delhi Rs-5000 UpTo 10 K Hauz Khas Whats Up Number
 
Delhi Russian Call Girls In Connaught Place ➡️9999965857 India's Finest Model...
Delhi Russian Call Girls In Connaught Place ➡️9999965857 India's Finest Model...Delhi Russian Call Girls In Connaught Place ➡️9999965857 India's Finest Model...
Delhi Russian Call Girls In Connaught Place ➡️9999965857 India's Finest Model...
 
2024: The FAR, Federal Acquisition Regulations - Part 27
2024: The FAR, Federal Acquisition Regulations - Part 272024: The FAR, Federal Acquisition Regulations - Part 27
2024: The FAR, Federal Acquisition Regulations - Part 27
 
Russian Call Girls Service Ashiyana Colony { Lucknow Call Girls Service 95482...
Russian Call Girls Service Ashiyana Colony { Lucknow Call Girls Service 95482...Russian Call Girls Service Ashiyana Colony { Lucknow Call Girls Service 95482...
Russian Call Girls Service Ashiyana Colony { Lucknow Call Girls Service 95482...
 
Global debate on climate change and occupational safety and health.
Global debate on climate change and occupational safety and health.Global debate on climate change and occupational safety and health.
Global debate on climate change and occupational safety and health.
 
EDUROOT SME_ Performance upto March-2024.pptx
EDUROOT SME_ Performance upto March-2024.pptxEDUROOT SME_ Performance upto March-2024.pptx
EDUROOT SME_ Performance upto March-2024.pptx
 
(DIVYA) Call Girls Wakad ( 7001035870 ) HI-Fi Pune Escorts Service
(DIVYA) Call Girls Wakad ( 7001035870 ) HI-Fi Pune Escorts Service(DIVYA) Call Girls Wakad ( 7001035870 ) HI-Fi Pune Escorts Service
(DIVYA) Call Girls Wakad ( 7001035870 ) HI-Fi Pune Escorts Service
 
↑VVIP celebrity ( Pune ) Serampore Call Girls 8250192130 unlimited shot and a...
↑VVIP celebrity ( Pune ) Serampore Call Girls 8250192130 unlimited shot and a...↑VVIP celebrity ( Pune ) Serampore Call Girls 8250192130 unlimited shot and a...
↑VVIP celebrity ( Pune ) Serampore Call Girls 8250192130 unlimited shot and a...
 
VIP Kolkata Call Girl Jatin Das Park 👉 8250192130 Available With Room
VIP Kolkata Call Girl Jatin Das Park 👉 8250192130  Available With RoomVIP Kolkata Call Girl Jatin Das Park 👉 8250192130  Available With Room
VIP Kolkata Call Girl Jatin Das Park 👉 8250192130 Available With Room
 
2024: The FAR, Federal Acquisition Regulations - Part 28
2024: The FAR, Federal Acquisition Regulations - Part 282024: The FAR, Federal Acquisition Regulations - Part 28
2024: The FAR, Federal Acquisition Regulations - Part 28
 
Goa Escorts WhatsApp Number South Goa Call Girl … 8588052666…
Goa Escorts WhatsApp Number South Goa Call Girl … 8588052666…Goa Escorts WhatsApp Number South Goa Call Girl … 8588052666…
Goa Escorts WhatsApp Number South Goa Call Girl … 8588052666…
 
Rohini Sector 37 Call Girls Delhi 9999965857 @Sabina Saikh No Advance
Rohini Sector 37 Call Girls Delhi 9999965857 @Sabina Saikh No AdvanceRohini Sector 37 Call Girls Delhi 9999965857 @Sabina Saikh No Advance
Rohini Sector 37 Call Girls Delhi 9999965857 @Sabina Saikh No Advance
 
PPT Item # 4 - 231 Encino Ave (Significance Only)
PPT Item # 4 - 231 Encino Ave (Significance Only)PPT Item # 4 - 231 Encino Ave (Significance Only)
PPT Item # 4 - 231 Encino Ave (Significance Only)
 
VIP High Profile Call Girls Gorakhpur Aarushi 8250192130 Independent Escort S...
VIP High Profile Call Girls Gorakhpur Aarushi 8250192130 Independent Escort S...VIP High Profile Call Girls Gorakhpur Aarushi 8250192130 Independent Escort S...
VIP High Profile Call Girls Gorakhpur Aarushi 8250192130 Independent Escort S...
 
Climate change and occupational safety and health.
Climate change and occupational safety and health.Climate change and occupational safety and health.
Climate change and occupational safety and health.
 
(NEHA) Bhosari Call Girls Just Call 7001035870 [ Cash on Delivery ] Pune Escorts
(NEHA) Bhosari Call Girls Just Call 7001035870 [ Cash on Delivery ] Pune Escorts(NEHA) Bhosari Call Girls Just Call 7001035870 [ Cash on Delivery ] Pune Escorts
(NEHA) Bhosari Call Girls Just Call 7001035870 [ Cash on Delivery ] Pune Escorts
 
Human-AI Collaboration for Virtual Capacity in Emergency Operation Centers (E...
Human-AI Collaborationfor Virtual Capacity in Emergency Operation Centers (E...Human-AI Collaborationfor Virtual Capacity in Emergency Operation Centers (E...
Human-AI Collaboration for Virtual Capacity in Emergency Operation Centers (E...
 
Call Girls In Rohini ꧁❤ 🔝 9953056974🔝❤꧂ Escort ServiCe
Call Girls In  Rohini ꧁❤ 🔝 9953056974🔝❤꧂ Escort ServiCeCall Girls In  Rohini ꧁❤ 🔝 9953056974🔝❤꧂ Escort ServiCe
Call Girls In Rohini ꧁❤ 🔝 9953056974🔝❤꧂ Escort ServiCe
 
Call Girls Service Connaught Place @9999965857 Delhi 🫦 No Advance VVIP 🍎 SER...
Call Girls Service Connaught Place @9999965857 Delhi 🫦 No Advance  VVIP 🍎 SER...Call Girls Service Connaught Place @9999965857 Delhi 🫦 No Advance  VVIP 🍎 SER...
Call Girls Service Connaught Place @9999965857 Delhi 🫦 No Advance VVIP 🍎 SER...
 

NEET PG 2019.docx

  • 1. Q. 1 In Zellweger syndrome, which of the following is absent? 1. ER 2. Golgi apparatus 3. Mitochondria 4. Peroxisomes CORRECT SOLUTION Zellweger syndrome is a rare disease characterized by the absence of functional peroxisomes.  the long chain fatty acids (C26-C36) are not oxidized. They accumulate in tissues, particularly in brain, liver and kidney. Hence the disorder is also known as cerebrohepatorenal syndrome.  This condition is apparent at birth and is characterized by profound neurologic impairment, victims often dying within a year.  Two closely related conditions are neonatal adrenoleukodystrophy and infantile Refsum disease. Zellweger syndrome and these two conditions represent a spectrum of overlapping features, with Zellweger syndrome being the most severe (many proteins affected) and infantile Refsum disease the least severe REFERENCE Lippincotts-biochemistry-6th-edition Q NO 16.5 Q. 2 Ammonia causes depletion of which of the following in TCA cycle? 1. Oxaloacetate 2. Alpha-Keto glutarate CORRECT 3. Fumarate 4. Malate SOLUTION In Hyperammonemia (i.e. excess amino groups in cells), a large amount of alpha keto glutarate
  • 2. (intermediate of TCA) combine with an excess of ammonia in cells to form glutamate which further give rise to glutamine by taking up more ammonia (see fig). This leads to depletion of alpha-KG. As alpha-KG is an intermediate of TCA cycle, so TCA cycle stops and the brain is affected as brain is highly dependent on this aerobic pathway for energy. So in case of hyper ammonia alpha-ketoglutarate & glutamate will decrease and glutamine will increase. REFERENCE SATYANARAYANA BIOCHEMISTRY PG NO 340 Q. 3 Defect in Menke disease: 1. Lysyl hydroxylase 2. Lysyl oxidase CORRECT 3. Prolyl hydroxylase 4. Prolyl oxidase SOLUTION MENKE DISEASE / kinky OR STEEL HAIR SYNDROME- deficiency of copper results in defective crosslinking of collagen and elastin by the copper-dependent enzyme lysyl oxidase.
  • 3.  Defect in dietary absorption of copper  Die with in 5 years of age  Kinky hair and growth retardation.  menke disease is X linked disease due to mutation in ATP 7 A gene. This gene is present on Xp 12-13. REFERENCE Harper Illustrated biochemistry 31st edition PG NO 1459 Q. 4 Glutamine is increased in CSF, blood & urine in which defect: 1. Arginosuccinate synthetase 2. OTC 3. CPS-I CORRECT 4. Arginase SOLUTION Defect in urea cycle Defect Enzyme involved Hyperammonemia type I Carbamoyl phosphate synthase I Hyperammonemia type II Ornithine transcarbamoylase Citrullinemia Arginosuccinate synthase Arginosuccinic aciduria Arginosuccinase Hyperarginemia Arginase In all enzyme deficiency ammonia will increase but cps 1 is rate limiting enzyme so it will increase ammonia most & and increase in ammonia lead to increase in glutamine. NOTE: The only urea cycle disorder that is inherited X-linked recessively is hyperammonemia type II. REFERENCE LIPPINCOTT 7TH /258
  • 4. Q. 5 Amino acid required for conversion of norepinephrine to epinephrine:- 1. Lysine 2. Phenyl-alanine 3. Methionine CORRECT 4. Tryptophan SOLUTION Norepinephrine to Epinephrine conversion requires SAM (S-Adenosyl Methionine). Enzyme involved is Phenylethanolamine N-methyl transferase. Cortisol is the activator.
  • 5. Synthesis of Catecholamines (Dopamine, NE and E) from Tyrosine. SAM (S - adenosyl methionine) is synthesized from dietary methionine with the help of enzyme METHIONINE ADENOSYL TRANSFERASE (MAT). REFERENCE SATYANARAYANA BIOCHEMISTRY PG NO350 Q. 6 Creatinine, NO & urea are synthesized from which amino acid?
  • 6. 1. Arginine CORRECT 2. Citrulline 3. Aspartate 4. Glycine SOLUTION 1. Creatinine – Three amino acid -glycine, arginine and methionine-are required for creatine formation FORMATION OF CREATININE 2. NO is synthesized from arginine by enzyme NOS (Nitric Oxide Synthase).
  • 7. FORMATION OF NO 3. Urea is formed from arginine by enzyme arginase
  • 9. Which of the following is not a dietary fiber? 1. Inulin 2. Cellulose 3. Pectin 4. Gum CORRECT SOLUTION Dietary Fibre Insoluble Dietary Fibres Soluble Dietary Fibres Excreted unchanged from the intestine. get converted to gel form, and then they are excreted  Cellulose  Hemicellulose  Lignin  Resistant starch  Pectins  Beta – glucans (β-D-glucose)  Psyllium husk  Inulin  Resistant starch  Most common dietary fibre is cellulose. REFERENCE Lipponcott 7th ed. pg. 365 Q. 8 Which of the following is not the source of cytosolic NADPH:- 1. Isocitrate dehydrogenase 2. ATP citrate lyase CORRECT 3. Malic enzyme 4. G6PD SOLUTION SOURCE of Cytosolic NADPH
  • 10. HMP pathway Malic enzyme Isocitrate dehydrogenase  Main enzyme - G6PD  alternative pathway to glycolysis and TCA cycle for the oxidation of glucose  the main pathway for biosynthesis of NADPH and pentoses.  Converts malate to pyruvate  NADPH and CO2 are generated  NADPH mainly utilized for fatty acid synthesis  3 isoenzymes  One, which uses (NAD+) in Mitochondria  other two use NADP+ in Mitochondria and cytosol REFERENCE SATYANARAYANA BIOCHEMISTRY PG NO 274 Q. 9 Which of the following is increased in LCAT deficiency? 1. HDL CORRECT 2. LDL 3. VLDL 4. Chylomicron SOLUTION  LCAT(Lecithin Cholesterol Acyl Transferase,) o Converts HDL cholesterol to HDL cholesterol ester by adding one fatty acid in cholesterol.  LCAT deficiency o Proper HDL is not formed o Spherical HDL ↓ o Nascent or discoidal HDL ↑ o Free cholesterol ↑ ( not getting incorporated in HDL) o RBCs have increased cholesterol content and thus hemolysis occurs Disease Norum’s disease Fish eye disease LCAT Complete deficiency Partial deficiency
  • 11. Hemolytic Anemia Severe Absent End stage renal disease Present Absent REFERENCE harper's 31st edition PG NO 254 Q. 10 True regarding mitochondrial DNA is: 1. Linear double stranded 2. Inherited from mother CORRECT 3. Low mutation rate 4. All respiratory proteins are synthesized within mitochondria itself SOLUTION The mitochondrial DNA (mtDNA) has structural and functional resemblances with prokaryotic DNA  mtDNA is circular in nature and contains about 16,000 nucleotide bases  A vast majority of structural and functional proteins of the mitochondria are synthesized in the cvtosol under the influence of nuclear DNA. However, certain proteins (around 13), most of them being the components of electron transport chain, are synthesized in the mitochondria (e.g. cytochrome b of complex lll, two subunits of ATP synthase.
  • 12.  mtDNA is inherited from the mother  Mitochondrial DNA is subjected to high rate of mutations (about 10 times more than nuclear DNA)  Diseases related to mitochondrial DNA o MELAS (mitochondrial encephalopathy lactic acidosis stroke like episode) o Leber hereditary optic neuropathy o Leigh syndrome o Kearns sayre syndrome o NARP syndrome REFERENCE SATYANARAYANA BIOCHEMISTRY p.g no 563 Q. 11 Protein not synthesized in liver:- 1. Albumin 2. Immunoglobulins CORRECT 3. Plasma enzymes 4. Acute phase proteins SOLUTION Immunoglobulins – are synthesized from plasma cell. Other all plasma protein are synthesized from liver  They are glycoproteins  Consist of two identical heavy (H) Chains and two identical light (L) chains  Is a Y-shaped tetramer (H2L2)  Humans have five classes of immunoglobulins-namely IgG, IgA, IgM, IgD and IgE- containing the heavy chains ϒ, α, µ, 6 and ε, respectively.
  • 13. REFERENCE Harper’s 31st ed pg. 289 Q. 12 Vitamin A is stored in:- 1. Kupfer cells 2. Hepatocytes 3. Ito cells CORRECT 4. Endothelial cells of liver SOLUTION HSCs (Hepatic Stellate Cells) are:  Pericytes which exist in the space between parenchymal cells and liver sinusoidal endothelial cells of the hepatic lobule.  Also called Vitamin A-storing cells, Lipocytes, Interstitial cells, Fat-storing cells or Ito cells, Perisinusoidal cells
  • 14.  Store 50-80% of vitamin A in the whole body as Retinyl Palmitate. Extra information: Vitamin A –  The fat-soluble vitamin A, as such is present only in foods of animal origin. However, its provitamins carotenes are found in plants  Retinol, retinal and retinoic acid are regarded as vitamers of vitamin A. Function of vit A -  Vitamin A is necessary for a variety of functions such as vision, proper growth and differentiation, reproduction and maintenance of epithelial cells.  Deficiency manifestations of the eyes – nyctalopia is one of the earliest symptoms of vitamin A deficiency. REFERENCE Satyanarayana BIOCHEMISTRY BOOK PG NO 122 Q. 13 In Wilson disease there is less urinary excretion of:- 1. Phosphorous 2. Methyl-Histidine CORRECT 3. Phosphotyrosine 4. Serine SOLUTION Wilson disease is a condition of excess copper in body:  There exists a renal lesion in Wilson’s disease.  Plasma amino acid levels are normal both qualitatively & quantitatively, both in fed & fasting state.  Aminoaciduria (specially lysine, threonine, cystine) occurs.  But Taurine & 1 methyl-histidine & 3 methyl-histidine are less in urine.
  • 15. This is a journal based very rare question, just cram it. REFERENCE Harpers Illustrated Biochemistry 31th Edition PG NO Q. 14 Which type of cholesterol is present in gall stones? 1. Amorphous cholesterol dihydrate 2. Amorphous cholesterol monohydrate 3. Crystalline cholesterol dihydrate 4. Crystalline cholesterol monohydrate CORRECT SOLUTION Gallstones are composed:  Principally of cholesterol monohydrate crystals or  Acid salt of calcium bilirubinate.
  • 16. REFERENCE Harper 31st/e pg. 597 Q. 15 A 20 year old alcoholic malnourished patient presented to hospital with respiratory distress. His pulse was 112/minute. Patient had edema, hypertension, systolic murmur along the left sternal edge. Bilateral murmur along the left sternal edge. Bilateral crepitations were felt in the lungs. A diagnosis of congetsive high output cardiac failure was made. Which vitamin is deficient? 1. Vitamin B1 CORRECT 2. Vitamin C 3. Vitamin B2 4. Vitamin B 6 SOLUTION vitamin B 1 – water soluble vitamin  It has a specific coenzyme TPP (thiamine pyrophosphate ) this help in carbohydrate metabolism .  TPP Dependent enzyme
  • 17. 1. pyruvate dehydrogenase - conversion of pyruvate to acetyl CoA. 2. alpha -Ketoglutarate dehydrogenase – CTA CYCLE 3. Transketolas - (HMP shunt) 4. The branched chain ALPHA - keto acid dehydrogenase -: oxidative decarboxylation pyrophosphate of branched chain amino acids .  RDA ( Recommended dietary allowance) o Adult – 1 – 1.5 mg/day o Children – 0.7 – 1.2 mg /day  The requirement increases in pregnancy and lactation (2 mg/ day), old age and alcoholism.  Deficiency of vitamin b 1 is known as beriberi  In adults, two types of beri-beri  WET BERI – BERI - This is characterized by edema of legs, face, trunk and serous cavities. o Dyspnea and palpitation are present o The systolic blood pressure is elevated while diastolic is decreased o Pulse is fast and bouncing o Death is due to heart failure  Dry beri-beri - This is associated with neurological manifestations resulting in peripheral neuritis. o edema is not seen o muscle become week so patient has difficulty in walking.  Infantile beri-beri o infant born to mother who is suffering from thiamine deficiency o Brest milk is contain low thiamine o Sleeplessness , vomiting , convulsions o Bouts of screaming o resembles to abdominal colic o These all symptoms are due cardiac dilation o Death occur suddenly due to heart failure  Wernickes Korsakoff syndrome o Seen in chronic alcoholics o insufficient intake or impaired intestinal absorption of thiamine will lead to this syndrome o Symptoms of wernickes encephalopathy - G – GLOBAL CONFUSION O – OPHTHALMOPLEGIA A – ATAXIA  o Korsakoff syndrome – chronic neurological complication 1. 1. Amnesia - mainly anterograde amnesia
  • 18. 2. Confabulations – false story and honest lying  o Thiamin Nutritional Status Can Be Assessed by Erythrocyte Transketolase Activaity REFERENCE Satyanarayana biochemistry book pg no 135 &136 Q. 16 Type of collagen maximum in skin:- 1. Type I CORRECT 2. Type II 3. Type III 4. Type IV SOLUTION Collagen - the most abundant protein in mammals contains 4-hydroxyproline and 5- hydroxylysine. Vitamin C plays the role of a coenzyme in hydroxylation of proline and lysine while protocollagen is converted to collagen  The hydroxylation reaction is catalysed by lysyl hydroxylase (for lysine) and prolyl hydroxylase (for proline)  This reaction is dependent on vitamin C, molecular oxygen and a-ketoglutarate Type Distribution I Noncartilaginous connective tissues, including bone, tendon, skin II Cartilage, vitreous humor III Extensible connective tissues, including skin, lung, vascular system IV Basement membranes REFERENCE Harper Illustrated biochemistry 31st edition PG NO 1450
  • 19. Q. 17 In type I diabetes, which of the following is true:- 1. Increased lipolysis CORRECT 2. Decreased protein catabolism 3. Decreased hepatic glucose output 4. Increase glucose uptake SOLUTION diabetes mellitus 1 Metabolic change in DM 1. Hyperglycemia - Hyperglycemia is primarily due to reduced glucose uptake by tissues and its increased production via gluconeogenesis and glycogenolysis. 2. ketoacidosis – due to increase in plasma free fatty acid
  • 20. 3. Hypertriglyceridemia – conversion of fatty acid into triacylglycerols  The activity of the enzyme lipoprotein lipase is low in diabetic patients. Consequently, the plasma levels of VLDL, chylomicrons and triacylglycerols are increased 4 protein metabolism – insulin increase protein synthesis. But in DM 1 NUSULIN deficiency is present so protein synthesis is decrease and protein degradation is increase. Function of Insulin Metabolism Net effect Crabohydrate metabolism Glycolysis Gluconeogenesis Glycogenesis Glycogenolysis HMP shunt Increased Decreased Increased Decreased Increased Lipid metabolism Lipogenesis Lipolysis Ketogenesis Increased Decreases Decreased Protein Metabolism Protein synthases Protein Degradation Increased Decreased REFERENCE Satyanarayana biochemistry book pg no 682 Q. 18 Amino acid linking Kreb's cycle & urea cycle:-
  • 21. 1. Aspartate CORRECT 2. Fumarate 3. Alanine 4. Arginine SOLUTION Urea cycle andTCA Cycle is linked by three way 1. Fumarate in urea cycle is the most important integrating point with TCA cycle. Fumarate is converted to malate and then to oxaloacetate in TCA cycle. 2. Oxaloacetate undergoes transamination to produce aspartate which enters urea cycle. However, Fumarate is not an amino acid so amino acid linking Kreb's cycle and Urea cycle is aspartate. Fig- Kreb’s Bicycle Extra information: 1. 12 ATPs are generated in the TCA cycle while 4 ATPs are utilized for UREA CYCLE. 2. The CO2 liberated in the TCA cycle (in the mitochondria) can be utilized in the urea cycle REFERENCE HARPER 31ST/275
  • 22. Q. 19 In case of LPL deficiency, which of the following will increase after a fat rich diet? 1. Chylomicron CORRECT 2. HDL 3. Lipoprotein (a) 4. LDL SOLUTION Lipoprotein Lipase deficiency causes Familial Hyperchylomicronemia or Type I Hyperlipoproteinemia. This enzyme breaks the TGs of Chylomicrons into Fatty Acids and Glycerol. Chylomicrons move throughout the body in blood and they are acted upon by enzyme Lipoprotein Lipase. Hence In LPL deficiency, chylomicrons and TG will rise in plasma. Extra Reading: Hyperlipoproeinemia Type Increased plasma lipoproteins Increased plasma lipid (most) Probable metabolic defect I Chylomicrons Triacyglycerols Deficiency of lipoprotein lipase IIa LDL Cholesterol Deficiency of LDL receptors IIb LDL and VLDL Triacylglycerols and cholesterol Overproductionn of apo-B III IDL Triacylglycerols and cholesterol Abnormality in apo-E IV VLDL Triacylglycerols Overproduction of TG V Chylomicrons and VLDL Triacylglycerols - REFERENCE Satyanarayana biochemistry book pg no 321 Q. 20
  • 23. Which of the following is not a Ribozyme? 1. Transpeptidase 2. Ribonuclease 3. Peptidyl transferase 4. Poly A polymerase CORRECT SOLUTION Ribozyme  RNA having enzymatic activity  E.g. Ribonuclease P, peptidyl transferase or transpeptidase & snRNA (small nuclear RNA). Poly A Polymerase  Not a ribozyme.  Adds poly A tail in mRNA. REFERENCE Biochemistry 5th Edition by Satyanarayana pg no 82
  • 24. Q. 21 Retinitis pigmentosa patients doesn't have:- 1. DHA CORRECT 2. Eicosa pentaenoic acid 3. Arachidonic acid 4. Timnodonic acid SOLUTION Not all, but many studies have found that patients with retinitis pigmentosa tend to have lower blood levels of DHA (Docosa Hexaenoic Acid, ω3, 22:6), an omega -3 fatty acid found in the photoreceptor cells. ADDITIONAL READING ON DHA:  Also known as Cervonic acid. This is having 22 C and 6 double bonds (Docosa means 22, Hexaenoic means 6 double bonds).  These days health drinks (bournvita, horlicks) are fortified with DHA, as it is important for the brain and retina development in infants and children.  Decreased amounts lead to increase risk of Retinitis Pigmentosa.  DHA is present in high concentration in sperms, retina, cerebral cortex.  Constant source of DHA is breast milk.  Synthesized to a limited extent from α-linolenic acid or obtained directly from fish oils. REFERENCE Harper Illustrated biochemistry 31st edition PG NO 553. Q. 22 Which apolipoprotein is responsible for Alzheimer's disease:- 1. APOE3 2. APOE1
  • 25. 3. APOE4 CORRECT 4. APOE2 SOLUTION Relative risk of Alzheimer's disease is maximum with Apo E4 Apo E Risk of Alzheimer's E2, E2 0.6 E2, E4 2.6 E3, E4 3.2 E4, E4 14.9 REFERENCE https://medlineplus.gov/alzheimersdisease.html Q. 23 Which amino acid is used to synthesis Nitric oxide? 1. Glycine 2. Arginine CORRECT 3. Tyrosine 4. Histidine SOLUTION Nitric oxide (NO) –  Arginine is the substrate for the production of nitric oxide (NO)  ENZYME - nitric oxide synthase  Half life – about 5 sec.  Functions – o vasodilator and relaxation of smooth muscles o regulation of blood flow and BP o inhibitor of platelet aggregation o neurotransmitter o NO mediates the bactericidal actions of macrophages o erection of penis
  • 26.  Nitric oxide promotes the synthesis of cGMP. Glycine – 1. Formation of purine ring (2) 2. Synthesis of glutathione (3) 3. Conjugation reactions – two reactions A - The bile acids-cholic acid and chenodeoxy cholic acid-are conjugated with glycine B - glycine is important for detoxification of benzoic acid 4. Heme synthesis 5. Biosynthesis of creatine Histidine - generation of histamine  This amino acid is oxidatively deaminated by histidase to urocanic acid, which subsequently forms N-formiminoglutamate ([FIGlu]  Individuals deficient in folic acid excrete increased amounts of FIGlu in the urine, particularly after ingestion of a large dose of histidine. The FIGlu excretion test has been used in diagnosing a deficiency of folic acid. Tyrosine - Tyrosine is formed from phenylalanine by phenylalanine hydroxylase  Biosynthesis of thyroid hormones ,catecholamines ,melanin REFERENCE SATYANARAYANA BIOCHEMISTRY PG NO 366 Q. 24 Type-I hyperlipoproteinemia is characterized by:- 1. Elevated LDL 2. Elevated lipoprotein lipase 3. Elevated cholesterol 4. Elevated triglycerides in plasma CORRECT SOLUTION Hyperlipoproeinemia Type Increased plasma lipoproteins Increased plasma lipid (most) Probable metabolic defect
  • 27. I Chylomicrons Triacyglycerols Deficiency of lipoprotein lipase IIa LDL Cholesterol Deficiency of LDL receptors IIb LDL and VLDL Triacylglycerols and cholesterol Overproductionn of apo-B III IDL Triacylglycerols and cholesterol Abnormality in apo-E IV VLDL Triacylglycerols Overproduction of TG V Chylomicrons and VLDL Triacylglycerols - REFERENCE SATYANARAYANA BIOCHEMISTRY PG NO 321 Q. 25 Gestational Trophoblastic neoplasm does not include:- 1. Choriocarcinoma 2. Placental site trophoblastic tumour 3. Invasive mole 4. Partial mole CORRECT SOLUTION Benign trophoblastic conditions Gestational trophoblastic Neoplasms 1. Normal pregnancy 2. Vesicular mole 3. Abortion 1. Invasive mole 2. Choriocarcinoma 3. Placental site trophoblastic tumor REFERENCE WILLIAMS obstetrics 25th ed/p- 393
  • 28. Q. 26 Which vaccine is contraindicated in pregnancy:- 1. Chicken pox CORRECT 2. Rabies 3. Tetanus toxoid 4. Hepatitis B SOLUTION All live vaccines are contraindicated in pregnancy and chicken pox is live vaccine Q. 27 45 years female with 3 months menorrhagia. USG showing 2 cm submucosal fibroid. First line of management is:- 1. OCP for 3 months 2. Progesterone for 3 months 3. Endometrial sampling CORRECT 4. Hysterectomy SOLUTION First line management always will be endometrial sampling ( curettage )to rule out . this is polyps or cancerous part Then treatment always be next step. Q. 28 In low ovarian reserve, Anti Mullerian hormone level will be:-
  • 29. 1. <1 CORRECT 2. 1-4 3. >7 4. >10 SOLUTION Ovarian Reserve < 1 is low , more than 2 is normal Markers of a REDUCED ovarian reserve are:  Increasing age, especially beyond 35  Decreased ovarian volume  Decrease estrogen  Decrease Antral Follicular count  High serum FSH, especially more than 15  Reduced Inhibin, especially lesser than 45 pg/ml  Low AMH - (SINGLE BEST TEST ) REFERENCE Shaws Textbook of Gynecology 17e p54 Q. 29 Presenting diameter of full flexed head:- 1. Suboccipito-bregmatic diameter CORRECT 2. Suboccipito-frontal diameter 3. Occipito-frontal diameter 4. Occipito-posterior position SOLUTION Important diameters of the skull Important Diameters of the skull Diameters Measurement in cm (inches) Attitude of the head Presentation
  • 30. Suboccipitobregmatic- extends from the nape of the neck to the center of the bregma 9.5 cm (3¼") Complete flexion Vertex Subocipitofrontal - extends from the nape of the neck to anterior end of the anterior fontanel or center of the sinciput 10 cm (4") Incomplete flexion Vertex Occipitofrontal- extends from the occipital eminence to the root of the nose (glabella) 11.5 cm (4½") Marked deflexion Vertex Mentovertical - extend from the midpoint of the chin to the highest point on the sagittal suture 14 cm (5½") Partial extension Brow Submentovertical - extends from junction of floor of the mouth and neck to the highest point on the sagittal suture 11.5 cm (4½") Incomplete extension Face Submentobregmatic - extends from junction of floor of the mouth and neck to the center of the bregma Complete extension Complete extension Face Q. 30 When used for emergency contraception, What is the dose of ulipristal acetate? 1. 300mg 2. 30mg CORRECT 3. 300ug 4. 30ug SOLUTION Emergency Contraceptives Drug Dose Levonorgestrel (POP) 1 dose 1.5 mg within 72 hrs Copper IUDs ( NOT THE DOC ! ) Insertion within 5 days Ulipristal acetate (SPRM) 30 mg orally within 120 hours Ethinyl estradiol 50 µg + Norgestrel 0.25 mg (COC) 2 tab stat and 2 after 12 hour Mifepristone RU 486 (PA) 25 to 100 mg single dose ICUDs are the most effective methods, no doubt, but they can never be the DOC. Thats because they require seeing a gynecologist take a prescription and then a specialist to insert the device.
  • 31. Emergency contraception is based on the principle of 'over the counter' easy to use medication to prevent a pregnancy and when there is unplanned intercourse, which can happen many times in a year, a person cannot be expected to take an IUCD every time! However, a LNG tablet can be easily, and effectively, be taken whenever there is unplanned intercourse * Please note: We are in the early part of year 2020 now. Mifepristone or Ullipristal may be soon termed as the best drugs for EC. Please be on the look out for recent updates. REFERENCE inputs from Williams 25th Ed Contraception chapter Q. 31 The major contribution of the amniotic fluid after 20 weeks of gestation:- 1. Ultrafiltrate and maternal plasma 2. Fetal urine CORRECT 3. Fetal lung fluid 4. Fetal skin SOLUTION Amniotic fluid (Main Source) -  First 12 weeks: Ultrafiltrate of Maternal plasma  12-20 weeks: Transudate across fetal skin  >20 weeks: Fetal urine REFERENCE William’s obstetrics 25th edition Page no: 188 Q. 32
  • 32. Nuchal translucency in USG can be detected at ------weeks of gestation:- 1. 11-13 CORRECT 2. 18-20 3. 8-10 4. 20-22 SOLUTION Nuchal translucency is a collection of fluid under the skin at the back of your baby's neck. The amount of fluid is measured during a nuchal translucency (NT) ultrasound scan: between 11 weeks and 14 weeks of pregnancy. Q. 33 Firsl line of treatment of mastitis in a lactating mother is:- 1. Cloxacillin CORRECT 2. Cefazolin 3. Ceftriaxone 4. Ampicillin SOLUTION FIRST LINE TREATMENT of MASTITIS in lactating mother is cloxacillin Doc is co-amoxiclav REFERENCE Bailey & love 27th e , page 864
  • 33. Q. 34 Patient with recurrent abortion diagnosed to have antiphospholipid syndrome. What will be the treatment:- 1. Aspirin only 2. Aspirin + Low molecular weight heparin CORRECT 3. Aspirin + Low molecular weight heparin + Prednisolone 4. No treatment SOLUTION DOC for APLA - Warfarin (Teratogenic) DOC for APLA in Pregnancy - Heparin + Aspirin  Heparin to be added only when there is H/O Recurrent abortions or Thrombosis  Otherwise we give only low dose Aspirin REFERENCE Williams obstetrics 25th Ed,p-1143-1146 Q. 35 A 60 year woman comes with 3rd degree uterine prolapse. What will be the management? 1. Vaginal hysterectomy with pelvic floor repair CORRECT 2. Pelvic floor repair 3. Sacrospinous fixation 4. Pessary SOLUTION Uterine prolapse is classified using a four part grading system:
  • 34. Grade1:Descent of the uterus to above the hymen Grade2:Descent of the uterus to the hymen Grade3:Descent of the uterus beyond the hymen Grade4:Total prolapse Treatment ,  if 14year girl then do sling operation  If 30year female then do Manchester/ fothergill operation  If postmenopausal women do vaginal hysterectomy with pelvic floor repair (WARD & MAYO) REFERENCE Williams obstetrics 25th edition Q. 36 Day 20 of menstrual cycle falls under which phase? 1. Menstrual phase 2. Follicular phase 3. Ovulation phase 4. Luteal phase CORRECT SOLUTION Menstrual Cycle Cycle days 1-5 6-14 15-28 Endometrial phase Menstrual (bleeding) phase Proliferation phase Secretory phase Ovarian phase Early follicular Late follicular Luteal Estrogen/Progesterone Low Estrogen↑ Progesterone↑ Gonadotropins (FSH/LH Low FSH↑ LH↑
  • 35. Q. 37 Chromosome number of partial hydatidiform mole most commonly is:- 1. 46 XX 2. 45 XO 3. 46 XXY 4. 69 XXX CORRECT SOLUTION Refer to Q 16 in this section REFERENCE williams obstetrics 25th ed/p- 392 Q. 38 Vulvar atrophy and itching are treated by- 1. Estrogen ointment CORRECT 2. Antihistamines 3. Tamoxifen 4. Oral antifungal treatment SOLUTION Vulvar atrophy and itching in menopausal age group are due to deficiency of estrogen that cause dryness of vulva which causes itching. Ideal treatment of this manifestation of post menopausal urogenital atrophy is some form of Hormone Replacement. Topical estrogen is most effective with minimal systemic side effects REFERENCE Shaws Textbook of Gynecology 17e p 374
  • 36. Q. 39 PGF2 alpha maximum dose in PPH management which can be given over 24 hours is- 1. 250 μg 2. 200 μg 3. 2 mg CORRECT 4. 20 mg SOLUTION PGF2∝ is CARBOPROST 250 µg I.M. up to 8 doses and maximum dose is 2 mg given I.M. always ( i.v. can cause sudden hypertension ) REFERENCE Williams Obstetrics 25th ed/p-760 Q. 40 Which of the following is false as physiological change in pregnancy? 1. Increase cardiac output 2. Increase total protein 3. Increase residual volume CORRECT 4. Increase GFR SOLUTION In Pregnancy, diaphragm pushed upwards by enlarged Uterus thus reducing Residual volume of lungs.  Diaphragm is elevated by around 4cm
  • 37.  subcostal angle increases from 68º to 104º.  Transverse diameterof chest increases by 2 cm Changes in Respiratory System Parameters Non pregnant Pregnancy near term Change Respiratory rate/min 15 15 unaffected Vital capacity (mL) 3200 3300 Almost unaltered Tidal volume (mL) 500 700 + 40% Residual volume (mL) 965 765 - 20% Inspiration capacity (IC) 2500 2650 + 10% Minute ventilation 7.5 L/min 10.5 L/min + 40% Total lung capacity (mL) 4200 4000 - 5%( almost unchanged) REFERENCE Williams obstetrics 25th Ed,p-151 Q. 41 MgSO4 have no role in prevention of- 1. Seizures in severe pre-eclampsia 2. Recurrent seizures in eclampsia 3. RDS in premature baby CORRECT 4. Neuroprotection SOLUTION MgSO4 is the Drug of choice which prevents seizures in severe pre ecclapmsia and also for seizure control in eclampsia This is also used for control of recurrent seizures Neuroprotective action: low-birthweight neonates whose mothers were treated with magnesium sulfate for preterm labor or preeclampsia were found to have a reduced incidence of cerebral palsy at 3 years REFERENCE williams 25th ed/p-736-737
  • 38. Q. 42 Green frothy vaginal discharge is produced by – 1. Herpes simplex 2. Candida albicans 3. Trichomonas vaginalis CORRECT 4. Normal vaginal flora SOLUTION Greenish frothy vaginal discharged is produced by trichomonas vaginalis , also seen strawberry vagina White curdy discharged is produced by candida albicans Q. 43 Which of the following is an absolute CONTRAINDICATION to OCP use: 1. Chronic renal disease 2. DVT CORRECT 3. Diabetes mellitus 4. History of amenorrhea SOLUTION OCP use is absolute CONTRAINDICATION IN DVT Q. 44 Which of the following statement is correct about acute fatty liver of pregnancy?
  • 39. 1. Occurs in 1 in 1000 pregnancy 2. Mostly seen in last trimester CORRECT 3. Common if female fetus is present 4. May be associated with decreased uric acid SOLUTION OCCUR in 1 in 10,000 pregnancy Mostly seen in last trimester 37-38 weeks Common if male fetus is present Q. 45 Female with 41 wk gestation confirmed by radiological investigation, very sure of her LMP, no uterine contractions, no effacement and no dilatation. What should not be done? 1. Intracervical foley’s 2. PGE1 tab 3. PGE2 gel 4. PGF2alpha CORRECT SOLUTION We can give misoprostol, Dinoproston , intracervical foley’s PGF2 alpha – carbo prost don’t give here {this is used in PPH ) Q. 46 Testosterone secreted by:- 1. Gonadotropic cells 2. Sertoli cells
  • 40. 3. Acidophilic cells 4. Leydig's cells CORRECT SOLUTION LEYDIG CELLS:  Testosterone is secreted by Leydig cells located in interstitium of testis.  It is essential for growth & division of testicular germinal cells, which is 1st stage in forming sperm.  L.H secreted by anterior pituitary gland, stimulates Leydig cell to secrete testosterone. Sertoli cells:  FSH secreted by anterior pituitary gland, stimulates Sertoli cells  This stimulation causes conversion of spermatids to sperm (spermiogenesis)  Without this stimulation, spermiogenesis do not occur.  Secrete inhibin, Mullerian inhibiting substance  Forms blood testes barrier
  • 41. Gonadotropic cells:  These cells are present in anterior pituitary gland which secrete both FSH & L.H under the influence of GnRH. Acidophilic cells:  Somatotrophs in anterior pituitary stain strongly with acid dyes so they are called acidophil cells.  These secrete growth hormone, which stimulates lipolysis, inhibits action of insulin on carbohydrate & lipid metabolism. REFERENCE Guyton; chapter 81, page no. 1023; 1022, figure 81-2
  • 42. Guyton, chapter 81, page 1031 Guyton, chapter 76, page 940 Q. 47 Aquaporin channel mediated through ADH:- 1. GLUT 2. Aquaporin 1 3. Aquaporin 2 CORRECT 4. Aquaporin 3 SOLUTION ADH:  Most important renal action of ADH is to increase water permeability of distal tubule, collecting tubule & collecting duct epithelia.  This helps the body to conserve H2O in circumstances like dehydration  ADH binds to specific V2 receptors in late distal tubules, collecting tubules & collecting ducts, increases formation of cAMP & activates protein kinases.  This action in turn, stimulates movement of intracellular protein called aquaporin – 2 (AQP 2) to luminal side of cell membranes  Molecules of AQP 2 cluster together & fuse with cell membrane by exocytosis to form H2O channels that permit rapid diffusion of water through cells.  Other aquaporins, AQP 3 & AQP 4, on basolateral side of cell membrane provide path for H2O to rapidly exit the cells, but these aquaporins are not believed to be regulated by ADH [REFER GUYTON AND HALL TEXTBOOK OF PHYSIOLOGY,CHAPTER-76, Page-948,949] GLUT:  Group of membrane proteins that facilitate transport of glucose across the plasma membrane
  • 43. Q. 48 All are true about Decerebrate posture except:- 1. Extension of both upper and lower extremity 2. Flexion of upper extremity and extension of lower extremity CORRECT 3. Exaggerated gamma motor neuron discharge 4. Reticulo spinal tract is also involved SOLUTION Decerebrate rigidity:  When brain stem of animal is sectioned below midlevel of mesencephalon but pontine & medullary reticular systems as well as vestibular system are intact, this is decerebrate animal  This results in rigidity of antigravity muscles à muscles of neck & trunk & extensors (i.e all extensors go into spasm)  This decerebrate rigidity is due to blockage of normally strong input to medullary reticular nuclei from cerebral cortex, Red nuclei & the basal ganglion.  Lacking this input, medullary reticular inhibitor system becomes nonfunctional, full overactivity of pontine excitatory system occurs & rigidity develops.  (Due to exaggerated gamma motor neuron discharge) Decorticate rigidity:  Thick layer of cortex is removed & all other structures are preserved.  This results in decorticate rigidity à flexors of upper limb & extensors of lower limb goes into spasm (flexion). REFERENCE Guyton, chapter 56, page 714 Q. 49 Bainbridge reflex is due to:- 1. Stretching of atria CORRECT
  • 44. 2. Baroreceptor reflex 3. Decreased venous return 4. Increased ventricular activity SOLUTION Bainbridge reflex:  Increase in atrial pressure causes increase in heart rate by nervous reflex called Bainbridge reflex 40 – 60 % increase in rate is due to this reflex while 15% increase is by direct effect of increased atrial volume to stretch sinus node.  Stretch receptors of atria that elicit Bainbridge reflex transmit their afferent signals through vagus nerve to medulla of brain.  These efferent signals are transmitted back through vagal & sympathetic nerves to increase H.R & strength of heart contraction.  This reflex helps prevent damming of blood in atria, veins & pulmonary circulation  Best known nerve mechanisms for arterial pressure control  Reflex is initiated by stretch receptors called baroreceptors /pressoreceptors located at specific points in walls of several large systemic arteries  A rise in arterial pressure stretches baroreceptor & causes them to transmit signals into CNS.  “feedback” signals are then sent back through autonomic nervous system to circulation to reduce arterial pressure downward towards the normal level REFERENCE Guyton, chapter 18, page 223] Baroreceptor reflex Guyton, chapter 18, page 219, 220 Q. 50 Function of Golgi tendon organ is to detect:- 1. Tension CORRECT 2. Length 3. Pressure 4. Proprioception SOLUTION
  • 45. → Muscles & their tendons are supplied abundantly with 2 special types of sensory receptors 1.muscle spindles → Distributed throughout belly of muscle & send information to nervous system about muscle length / rate of change of length. 2. Golgi tendon organs → Located in muscle tendons & transmit information about tendon tension or rate of change of tension. → Signals from these 2 Receptors are almost entirely for intrinsic muscle REFERENCE Guyton, chapter 55, page 697 Q. 51 Peripheral chemoreceptors respond to hypoxia using which channel? 1. Calcium channel 2. Sodium channel 3. Potassium channel CORRECT 4. Chloride channel SOLUTION PERIPHERAL CHEMORECEPTOR: → Located in several areas outside the brain, mostly in carotid bodies & aortic bodies → They are especially important for detecting changes in O2 in the blood (lesser extent to changes in CO2 & H+ ion concentration) → Aortic & carotid bodies have multiple, highly characteristic glandular like cells called “GLOMUS CELLS” that synapse directly or indirectly with nerve endings, (they serve as chemoreceptor) → Low Po2 stimulate glomus cells in aortic & carotid bodies ↓
  • 46. O2 sensitive potassium channels in glomus cells are inactivated ↓Resulting in Cell depolarization ↓Which in turn Opens voltage gated calcium channels & Increases Intracellular Ca ion concentration ↓ Stimulate neuro transmitter release ↓ Which activates afferent neurons that sends signals to CNS ↓ Stimulates respiration → During hypoxia ATP (mainly), ach / dopamine are excitatory Neurotransmitters.
  • 47.
  • 48. REFERENCE Guyton page 543, 544, figure 42.6 Q. 52 Functional residual capacity represents the volume of air remaining in lungs:- 1. After normal inspiration 2. After normal expiration CORRECT 3. After forceful expiration 4. After forceful inspiration SOLUTION Functional residual capacity: → It equals the EXPIRATORY RESERVE VOLUME + RESIDUAL VOLUME
  • 49. → This capacity is amount of air that remains in lungs at end of normal expiration (about 2300 ML) Pulmonary capacities: Inspiratory capacity: → Tidal volume + inspiratory reserve volume [about 3500ml] → This capacity is amount of air a person can breathe in, beginning at normal expiratory level & distending the lungs to maximum amount. Vital capacity: → Inspiratory reserve volume + tidal volume + expiratory reserve volume. → This capacity is max amount of air a person can expel from lungs after first filling the lungs to their maximum extent & then expiring to maximum extent (about 4600ml) Total lung capacity: → Max vol to which lungs can be expanded with greatest possible effort [5800 ml] → Vital capacity + residual volume REFERENCE Guyton and Hall Textbook of Medical Physiology 13th Edition, Page No:501, 502.
  • 50. Q. 53 Tubuloglomerular feedback control is useful for which one of the following:- 1. GFR 2. Plasma sodium 3. Plasma volume CORRECT 4. Determining tubular secretion SOLUTION Tubulo-glomerular feedback:  Increase in blood pressure (changes in B.P) Increase GFR ↓ Sodium filtration ↓ Sensed by macula densa cells of thick ascending limb ↓ Sends tubulo-glomerular feedback by adenosine ↓ Adenosine results in constriction of afferent arteriole ↓ Resulting in decrease GFR & decrease Na+ filtration This feedback is to prevent loss of sodium & avoid electrolyte imbalance.
  • 51. REFERENCE GUYTON AND HALL TEXTBOOK OF MEDICAL PHYSIOLOGY 13TH EDITION-PAGE- 343, 344, fig (27-11) Q. 54 Antegrade peristalsis due to? 1. Ach CORRECT 2. Serotonin 3. Substance P 4. VIP SOLUTION → Excitatory neurotransmitters in ENS are Acetylcholine & substance P
  • 52. → Inhibitory neurotransmitters in enteric nervous system are NO & VIP → Acetylcholine is main excitatory neurotransmitter responsible for antegrade peristalsis REFERENCE Guyton and Hall Textbook of Medical Physiology 13th Edition, Page No. 800 Q. 55 C- peptide is seen:- 1. In pre-proinsulin 2. In Proinsulin CORRECT 3. As a combined entity with insulin after secretion 4. As a gastrointestinal proactive molecule SOLUTION → Insulin is synthesized in BETA- cells of pancreas by usual cell machinery for protein synthesis. → Beginning with translation of insulin RNA by ribosomes attached to E.R to form pre proinsulin. → Pre proinsulin is cleaved in E.R to form pro-insulin.
  • 53. → Insulin & C peptide are packaged in secretory granules & secreted in equimolar amounts. → So, c – peptide levels can be measured by radioimmunoassay in insulin-treated diabetic patients to determine how much of their own natural insulin they are still producing REFERENCE Guyton, chapter 79, page 984, figure (79.2) Q. 56 In forceful expiration, which of the following neurons gets fired?
  • 54. 1. VRG CORRECT 2. DRG 3. Pneumotaxic centre 4. Chemoreceptors SOLUTION → Located in each side of medulla, about 5mm anterior & lateral to dorsal respiratory group of neurons is ventral respiratory group of neurons, found in nucleus ambigus rostrally & nucleus retroambigus caudally. → Neurons of VRG remain almost totally inactive during normal quiet respiration. so, normal quiet breathing is caused only by repetitive inspiratory signals from DRG transmitted mainly to diaphragm & expiration results from elastic recoil of lungs & thoracic cage. → When respiratory drive for increased pulmonary ventilation becomes greater than normal, respiratory signals spill over into VRG from basic oscillating mechanism of DRG → As a consequence, VRG contributes extra respiratory drive as well → Electrical stimulation of few of neurons in VRG cause inspiration & others cause expiration, thus these neurons contribute to both inspiration & expiration. → They are especially important in providing powerful expiratory signals to abdominal muscles during very heavy expiration. Pneumotaxic center: Primary effect is to control “switch off” point of inspiratory ramp, thus controlling duration of filling phase of lung cycle. → Main function is to limit inspiration which has secondary effect of increasing rate of breathing because limitation of inspiration also shortens expiration & entire period of each respiration Chemoreceptors: → Sensors that detect changes in Co2, O2, pH. → Based on anatomical position divided into  o Central chemoreceptors o Peripheral chemoreceptors
  • 55. REFERENCE Guyton, chapter 42, page 540, figure 42.1 Q. 57 Slow conduction velocity is seen with which of the following nerve fibers? 1. Preganglionic autonomic nerve fibers 2. Postganglionic autonomic nerve fibers
  • 56. CORRECT 3. Motor nerves 4. Sympathetic nerve fibers SOLUTION  Slow conduction velocity is seen in ‘C’- fibers because they are unmyelinated.  These are post ganglionic sympathetic nerve fibers REFERENCE Ganong’s Review of Medical Physiology 26th Edition, Page No. 256 Q. 58 Which of the following is water soluble contrast? 1. Barium 2. Iodine CORRECT 3. Bromium 4. Calcium SOLUTION Radiocontrast Positive Negative Barium (Water isoluble) Iodinated Compounds (water soluble) Air Water • Iohexol • Iopamidol • Iodixanol Q. 59 Identify the infection from the chest x-ray of patient with low grade fever?
  • 57. 1. ILD 2. Bronchopenumonia 3. Miliary TB CORRECT 4. Consolidation SOLUTION • Chest X-ray shows Millet seed like opacities which is a feature of miliary TB. • Millet seed like opacities are also seen in - Healed varicella pneumonia - Mitral stenosis REFERENCE GRAINGER AND ALLISON’S DIAGNOSTIC RADIOLOGY 6TH EDITION Page No: 259- 261 Q. 60
  • 58. A patient presented with history of diplopia and restricted eye movements. A clinical image and CT image is shown below What will be the diagnosis? 1. Le-fort fracture 2. Fracture maxilla 3. Fracture zygomatic 4. Blow out fracture CORRECT SOLUTION • Clinical image- shows Enophthalmos, congestion and edema of left eye • Coronal CT- shows orbital floor fracture causing orbital contents hanging in left maxillary sinus Blow out # or tear drop # • Best X-ray view for Orbital floor and maxillary sinus is WATER’S VIEW REFERENCE GRAINGER AND ALLISON’S DIAGNOSTIC RADIOLOGY 6THEDITION Page No: 1037 Q. 61 Identify the radiological procedure shown below?
  • 59. 1. Barium meal follow through CORRECT 2. Barium enema 3. Enteroclysis 4. Proctography SOLUTION • The given radiological image of barium study shows the presence of contrast in stomach, duodenum, jejunum (shows feathery appearance- valvulae conniventes) and ileum (featureless app.) - indicating that it is a barium meal follow through study. • Barium studies: → Barium swallow – esophagus → Barium meal – stomach & duodenum → Barium meal follow through – small intestine → Barium enema – large intestine • Enteroclysis: NJ Tube is inserted and contrast is injected into jejunum directly to visualize small bowel
  • 60. Enteroclysis: Used to detect small bowel lesions or any malabsorption. * CT vs MRI  Look at CORTICAL BONE – if dark = MRI REFERENCE GRAINGER AND ALLISON’S DIAGNOSTIC RADIOLOGY 6TH EDITION Page No : 653 Q. 62 Identify the condition in the below image? 1. Lacunar infarct CORRECT 2. Embolic infarct 3. Thrombotic infarct 4. Intracerebral hemorrhage
  • 61. SOLUTION • The given CT image shows hypodensity in thalamic region→ lacunar infarct • Lacunar infarct occurs due to occlusion of lenticulostriate branches or thalamostriate branches or deep penetrating branches of brain. • Lacunar infarct appears hypointense on T1 and hyperintense on T2 image REFERENCE GRAINGER AND ALLISON’S DIAGNOSTIC RADIOLOGY 6TH EDITION Page No: 1456 Q. 63 Not true about somnambulism among the following is? 1. Sleep walking 2. Patient consciousness is preserved CORRECT
  • 62. 3. Disorder of sleep arousal 4. Low level motor skill/function is present SOLUTION Sleepwalking (also called somnambulism) is a disorder of arousal(usually seen in children). It generally occurs in deep nonrapid eye movement (NREM) (stages 3 and 4) sleep, most often takes place in the first third of the night when these sleep stages are most common. The event may last from several minutes to an hour. a child appears to wake up during the night and walk or do other activities without any memory of having engaged in the activities.Difficult in waking up the child and may physical attacks by the child after wakeup. Rx- as the child grows up these symptoms will improve. Q. 64 Confusion assessment scale used for which of the following:- 1. Schizophrenia 2. Delirium CORRECT 3. Dementia 4. Depression SOLUTION Confusion Assessment method (CAM) is used for delerium. METHOD- -Ask patient to answer simple questions and perform simple task. -Can be used readily at the bedside. REFERENCE Kaplan and Sadock's Comprehensive Textbook of Psychiatry, 10th edition, Page No 1183 Q. 65
  • 63. Which of the following type of OCD has the poor response exposure and response prevention? 1. Magical thinking 2. Dirt contamination 3. Pathological doubt 4. Hoarding CORRECT SOLUTION Obsessive Compulsive Disorder (OCD) Obsessions:-  Repetitive, intrusive thoughts, images or impulses which cause anxiety.  Patient consider them as product of their own mind (D/D – thought insertion) Compulsions:- Behaviour on mental acts that follow & reduce anxiety temporarily #Both obsessions and compulsion are Ego dystonic – not agreeable to self Prevalence – 2 – 3 % 50% - acute onset symptoms persist for 2 weeks Etiology:- seratonergic dysregulation Nor epinephrine CSTC Tract Cortico – striato – thalamico – cotical tract Orbito – frontal – Straita – Thalamus Cortex Presentantion  Obsession of contamination followed by compulsion of washing (MC) –  Pathological doubt – Obsession of doubt with compulsion of checking.  Intrusive thoughts ↓ Aggressive / Religious / Sexual content  Symmetry – obsession of symmetry or precision Compulsion of slowness Rx – combination of pharmacotherapy and psychotherapy Hoarding is no longer subtype of OCD. In DMS 5 AND ICD-11 hoarding disorder is
  • 64. separate diagnosis. People with hoarding disorder excessively save items that others may view as worthless. They have persistent difficulty getting rid of or parting with possessions, leading to clutter that disrupts their ability to use their living or work spaces.Hoarding is not the same as collecting. Collectors look for specific items, such as model cars or stamps, and may organize or display them. People with hoarding disorder often save random items and store them haphazardly. In most cases, they save items that they feel they may need in the future, are valuable or have sentimental value. Some may also feel safer surrounded by the things they save. Q. 66 Intense depression & misery without any cause is? 1. Melancholia CORRECT 2. Major depressive disorder 3. Mania 4. Schizophrenia SOLUTION Melancholic depression – A. Prominent biological symptom 1. Early morning awakening (waking up at least 2 hours before usual time) 2.Significant anorexia and weight loss. B. Significant psychomotor agitation or retardation. C. Anhedonia and lack of mood reactivity.(the patients mood doesn’t improve even if some positive event occurs ) D. A distinct quality of intensely depressed mood, often referred to as a state of despondency or despair,also called empty mood.the patients feels miserable. E. Depression is worse in morning. F. Excessive guilt.
  • 65. REFERENCE Kaplan and sadocks synopsis of psychiatry edition 11 page 347 Q. 67 Which of the following is not true regarding delusional disorder? 1. Early immigration 2. Social isolation 3. Sensory impairment 4. Occurs at early age CORRECT SOLUTION Delusional disorders do not occur at an early age. Risk factors for development of delusional disorders-  Advanced age  Social isolation  Sensory impairment or isolation (e.g. auditory or visual disturbances)  Family history of delusional disorder  Recent immigration (is also risk factor for schizophrenia)  Certain personality features ,like excessive interpersonal sensitivity (even trivial interpersonal problems cause lot of negative emotions) REFERENCE Kaplan and sadocks synopsis of psychiatry edition 11 page 330-336 Q. 68 Identify the instrument:-
  • 66. 1. Maddox rod CORRECT 2. Maddox wing 3. Maddox glass 4. Red glass SOLUTION Maddox rod test is used for diagnosis of heterophoria. The patient is placed in front of a bright spot of light in a dark room. A Maddox rod, which consists of four or five cylinders of red glass side by side in a supporting disc, is placed in the trial frame before one eye . The spot of light seen through the red cylinders appears as a long red line perpendicular to the direction of the cylinders or grooves. If the cylinders are placed with their axes horizontal, the red line will be vertical. If there is orthophoria the bright spot will appear to be in the centre of the vertical red line; if there is eso- or exophoria the red line will be to one side of the spot. The angle of the deviation is measured by the strength of the prism which is necessary to be placed in front of the Maddox rod (or the other eye) to bring the red line and the spot together. The nature of the deviation is indicated by the position of the base of the prism, whether out (eso- phoria) or in (exophoria). Q. 69 Esotropia is commonly seen in which type of refractive error? 1. Myopia 2. Hypermetropia CORRECT 3. Astigmatism 4. Presbyopia SOLUTION
  • 67. These are generally more common in childhood and hyper- metropes. The continuous effort of accommodation in the hypermetrope to see clearly, even in the distance, stimulates convergence to a greater degree than is compatible with binocular fixation and results in esotropia. REFERENCE Comprehensive Ophthalmology AK Khurana Q. 70 In the given condition, cause of defective vision is:- 1. Visual axis is obscured 2. Astigmatism CORRECT 3. Hypermetropia 4. Diplopia SOLUTION The above image shows a pterygium. Defective vision occurs when it encroaches the pupillary area or due to corneal astigmatism induced by fibrosis in the regressive stage. Q. 71 This test is used in:-
  • 68. 1. Squint 2. Heterophoria 3. Esotropia 4. All of the above CORRECT SOLUTION The above diagram shows cover uncover test used for squint diagnosis(both tropias and phorias). Interpretation of cover, cover–uncover and alternate cover tests.
  • 70. What is your diagnosis:- 1. Orbital blow out fracture CORRECT 2. Orbital roof fracture 3. Oculomotor nerve paralysis 4. Retrobulbar hemorrhage SOLUTION Following image shows a defect in the orbital floor and the ‘tear drop’ sign in the antrum characteristic of blow out fractures of orbit. WATER'S view on X-RAY is helpful in eliciting orbital blow out fracture REFERENCE Comprehensive ophthalmology A K Khurana 6th edition page 423 Q. 73 Which one is against the rule astigmatism? 1. +2.0 DC x 90 2. -2.0 DC x 90 CORRECT 3. +1.5 DC x 180 4. -1.5 DC x 180 SOLUTION Against-the-rule astigmatism (ATR) refers to an astigmatic condition in which the horizontal meridians is more curved than the vertical meridian. Therefore, correction of this astigmatism
  • 71. will require the prescription of convex cylindrical lens at 180° ± 7 20° or concave cylindrical lens at 90° ± 20° axis. Therefore, both 2 and 3 are the correct options. Q. 74 Intravenous mannitol is indicated in:- 1. Primary open angle glaucoma 2. Acute angle closure attack CORRECT 3. Normal tension glaucoma 4. Sympathetic ophthalmitis SOLUTION Mannitol is a hyperosmotic agent. Hyperosmotic agents increase the plasma tonicity. Thus, the osmotic pressure gradient created between the blood and vitreous draws sufficient water out of the eyeball, thereby significantly lowering the IOP. Indications These are used as additive therapy for rapidly lowering the IOP in emergency situations, such as acute angle-closure glaucoma or secondary glaucomas with very high IOP. They are also used as a prophylactic measure prior to intraocular surgery. Q. 75 The ocular hypotensive agent causing apnoea in infants is:- 1. Latanoprost 2. Timolol 3. Betaxolol 4. Brimonidine CORRECT SOLUTION
  • 72. Brimonidine is selective alpha 2 receptor agonist. It can cause drowsiness and respiratory depression in children and infants and therefore contraindicated in them. REFERENCE AK khurana 7th edition pg no. 243,469,473 Q. 76 The dosage of vitamin A in keratomalacia in a 2 year old boy who is 12 kg weight is:- 1. Vitamin A: 2 lakh in oral, 1st, 2nd, 14th day CORRECT 2. Vitamin A: 1 lakh in oral, 1st, 2nd, 14th 3. Vitamin A: 2 lakh in oral, 1st, 2nd, 3rd 4. Vitamin A: 1 lakh in oral, 1st, 2nd, 3rd SOLUTION 1. All patients above the age of 1 year (except women of reproductive age): 200,000 IU of vitamin A orally or 100,000 IU by intramuscular injection should be given immediately on diagnosis and repeated the following day and 4 weeks later. 2. Children under the age of 1 year and children of any age who weigh less than 8 kg should be treated with half the doses for patients of more than 1 year of age. iii.Women of reproductive age, pregnant or not: (a) Those having night blindness (XN), conjunctival xerosis (X1A) and Bitot’s spots (X1B) should be treated with a daily dose of 10,000 IU of vitamin A orally (1 sugar coated tablet) for 2 weeks. (b) For corneal xerophthalmia, administration of full dosage schedule (described for patients above 1 year of age) is recommended. Q. 77 Which of the parameters is decreased in Retinitis pigmentosa? 1. Arachidonic acid
  • 73. 2. Docosahexanoic acid CORRECT 3. Thromboxane 4. Trielonic acid SOLUTION Mean plasma DHA levels (as percentages of total plasma fatty acids in plasma) were reduced below the mean normal levels in patients with X-linked and isolate forms of RP. Q. 78 The most common cause of proptosis in adults:- 1. Orbital cellulitis 2. Preseptal cellulitis 3. Thyroid eye disease CORRECT 4. Capillary hemangioma SOLUTION Most common cause of unilateral proptosis in children is orbital cellulitis and in adults is thyroid ophthalmopathy. Most common cause of bilateral proptosis in children is neuroblastoma and leukemia (chloroma) and in adults is thyroid ophthalmopathy. REFERENCE Ak khurana 7th edition 421 Q. 79 Which one of the procedure involves using glaucoma drainage device? 1. Seton operation
  • 74. CORRECT 2. Deep sclerectomy 3. Viscocanalostomy 4. Trabeculectomy SOLUTION 1. Seton operation- Glaucoma drainage device or the so called glaucoma valve implants are plastic devices which allow aqueous outflow by creating a communication between the anterior chamber and sub-Tenon’s space. The operation using glaucoma valve implant is also known as Seton operation 2. 2. Deep sclerectomy-A Descemet window is created to allow aqueous migration from the AC. Subsequent egress is subconjunctival, resulting in a shallow filtration bleb, as well as along deeper suprachoroidal routes. 3. Viscocanalostomy-Viscocanalostomy involves the creation of a filtering window, with identification and dilatation of the Schlemm canal with high density viscoelastic. The superficial scleral flap is sutured tightly so that subconjunctival fluid outflow and bleb formation are minimized 4. Trabeculectomy-Trabeculectomy is glaucoma filtration surgery that lowers IOP by creating a fistula, protected by a superficial scleral flap, to allow aqueous outflow from the anterior chamber to the sub-Tenon space. Q. 80 Oral factor Xa inhibitor is:- 1. Dabigatran etexilate 2. Rivaroxaban CORRECT 3. Fondaparinux 4. Bivalirudin SOLUTION  Any drug ending with XABAN is X a blocker,  Rivaroxaban  Apixaban  Edxaban  Betrixaban Dabigatran: Direct thrombin inhibitor
  • 75. Only oral agent in class(DTI) Can reverse with Idarucizumab. Fondaparinux: LMWH which acts only on Factor X a Bivalirudin : Direct Thrombin Inhibitor. REFERENCE Lippincott pharmacology 6th edition page 303 Q. 81 A nursing mother presented with fever and breast tenderness after two weeks postpartum. Which oral antibiotics is ideal for her condition? 1. Ampicillin 2. Dicloxacillin CORRECT 3. Ceftazidime 4. Ciprofloxacin SOLUTION  Dicloxacillin:- Anti - staphylococcal penicillin :- B lactamase resistant penicillin. Their use is restricted to the treatment of infection cause by penicillinase producing staphylococci including MSSA. Other drug of this class:- Methicillin Oxacillin  Methicillin because of its toxicity (interstitial Nephritis), Methicillin is not used.  Ampicillin/Amoxicillin:-Extended- Spectrum penicillin. More effective against gram - ve bacilli.  Ampicillin is a drug of choice for gram positive bacilli : listeria monocytogenes  Ceftazidime:-3rd gen cephalosporin, has activity against P. aeruginosa  Ciprofloxacin: of the fluoroquinolone it has best activity against P. Aeruginosa and is commonly used in cystic fibrosis patient. REFERENCE KD Tripathi Essentials of Medical Pharmacology 8th edition/page 771
  • 76. Q. 82 High plasma protein binding of a drug results in:- 1. Decreased glomerular filtration CORRECT 2. High volume of distribution 3. Lowers duration of action 4. Less drug interaction SOLUTION • High plasma protein binding (PPB) of a drug decreases its distribution into tissue and thus lower the volume of distribution. • As only free drug can be metabolized or excreted, high PPB usually increases the duration of action. • With higher PPB, there is more chance of getting displaced by other drugs leading to more drug interactions. • Drugs with high plasma protein binding have lesser glomerular filteration as proteins cannot be filtered across the glomerulus. REFERENCE KD Tripathi Essentials of Medical Pharmacology 8th edition/page 26 Q. 83 DOC for prophylaxis against Diphtheria is:- 1. Erythromycin CORRECT 2. Rifampicin 3. Cloxacillin 4. Ciprofloxacin SOLUTION  Erythromycin is macrolide group of drug which binds irreversibly to a site on the 50s subunit of bacterial ribosome thus inhibiting translocation steps of protein synthesis.
  • 77.  It is used to eliminate the carrier state of Corynebacterium Diphtheriae.  Rifampicin: Blocks RNA transcription by interacting with B subunit of mycobacterial DNA dependent RNA polymerase. REFERENCE Lippincott pharmacology 6th edition page 431 Q. 84 Variation in drug responsiveness to a particular dose of the drug in different individuals can be obtained from:- 1. Graded Dose Response Curve 2. Efficacy 3. Potency 4. Quantal Dose response Curve CORRECT SOLUTION
  • 78.  ED50 : dose at which 50% of subject will responds to drug.  Potency :- Measure of amount of drug necessary to produce an effect of a given magnitude.  Efficacy: Magnitude of response a drug can cause when it interacts with a receptor. Depends on of drug receptor complex formed. • In this Quantal DRC, x-axis represents the log dose and y-axis represents the percentage of population responding. • It is used when responses are like all or none. The middle point of curve when projected to x- axis gives effective dose and lethal dose in 50% of population. REFERENCE KD Tripathi Essentials of Medical Pharmacology 8th edition/page 64
  • 79. Q. 85 Most specific antiemetic for chemotherapy induced vomiting is:- 1. Doxylamine 2. Tegaserod 3. Granisetron CORRECT 4. Domperidone SOLUTION DOC for chemotherapy induced vomiting are 5HT3 antagonist which are drugs ending with setron.  Granisetron  Ondansetron  Dolasetron These agent selectively block 5HT3 receptor in periphery (Visceral Vagal afferent fibers) and in brain(CTZ) Doxylamine are somnifacient (have strong sedative properties and are used in treatment of insomnia) these are 1st generation antihistamine like diphenhydramine . REFERENCE ESSENTIALS OF MEDICAL PHARMACOLOGY -8TH EDITION BY KD TRIPATHI (Pg-711) Q. 86 Identify the true statement regarding clinical trials:- 1. Phase 1 is done to determine efficacy in patients 2. Healthy volunteers are recruited for the first time in phase II 3. Randomized controlled trials in patients is done in phase III CORRECT 4. Phase IV is pharmacokinetics study in animals SOLUTION
  • 80. Clinical Trials Phase Done On Aim Cannot find I Healthy individual To know MTD Efficacy II Patient (20-200) Efficacy indication III Patient (upto 5000) Confirm efficacy IV (post marketing) Large number of patients To know rare side effect and chronic s/e.  MTD :- Maximum tolerable dose  1st phase in human: Phase I  1st phase in patient : phase II  Indication of efficacy of drug :- Phase II  Confirmation of efficacy of drug :- Phase III  Maximum no. of patient :- phase IV  Double Blinding and Randomized control Trial is done in phase III REFERENCE KD Tripathi Essentials of Medical Pharmacology 8th edition/page 90 Q. 87 Fluoroquinolone with highest oral bioavilability:- 1. Levofloxacin CORRECT 2. Gemifloxacin 3. Ciprofloxacin 4. Norfloxacin SOLUTION Bioavailability : rate and extend to which an administered drug reaches the systemic circulation . Of the above mentioned drugs:-  Levofloxacin has maximum bioavailability  Norfloxacin has minimum bioavailability REFERENCE
  • 81. KD Tripathi Essentials of Medical Pharmacology 8th edition/page 764 Q. 88 Which of the following statement is true regarding fixed dose combination of drugs? 1. Adverse effect of one drug may be reduced by the other drug CORRECT 2. Two drugs with different pharmacokinetics can be combined easily 3. Dose of one drug can be altered independently as per requirement 4. Adverse effect can be ascribed to a single drug SOLUTION Fixed dose combination (FDC):- Combination of two or more active drugs in single dose form. Rationality of FDC’s should be based on certain aspect such as:-  The drugs in combination should act by different mechanism.  Pharmacokinetic must not be widely different.  Combination should not have supraadditive toxicity of the ingredients. Demerits:-  Dosage alteration of one drug is not possible without alteration of other drugs.  Differing pharmacokinetic of constituent drugs poses the problem of frequency of administration of the formula.  Increased chance of adverse drug effect and interaction compared with both drugs given individually. REFERENCE Lippincott pharmacology 6th edition page 598 Q. 89 Which of the following is a K+ channel opener?
  • 82. 1. Ranolazine 2. Nicorandil CORRECT 3. Verapamil 4. Lignocaine SOLUTION NICORANDIL:-  K+ channel opener  NO releaser  Given for Angina  Drug activates ATP sensitive K+ channels --> Hyperpolarizing vascular smooth muscle.  Nicorandil is believed to exert cardioprotective action by simulating ‘ischaemic preconditioning’ as a result of activation of mitochondrial K-ATP channels. Ranolazine - Inhibits late phase of sodium current (Late I Na) improving oxygen supply and demand equation. It has antianginal as well as antiarrhythmic properties. Verapamil :- calcium channel blocker, slows AV conduction directly and decrease HR, contractility, BP and oxygen demand. Lidocaine : class IB antiarrhythmic drug, shortens phase 3 repolarization and decrease the duration of action REFERENCE KD Tripathi Essentials of Medical Pharmacology 8th edition/page- 552 Q. 90 A rheumatoid Arthritis patient on Methotrexate, steroids and NSAIDs for past 4 months has had no retardation of disease progression. What is the next rational step in management? 1. Start monotherapy with anti-TNF alpha drugs 2. Continue Methotrexate and steroids 3. Stop oral Methotrexate and start parenteral Methotrexate 4. Add Sulfasalazine CORRECT
  • 83. SOLUTION  For patient with RA, Steroid and NSAIDS one given for symptomatic relief.  Methotrexate used alone or in combination therapy has become a mainstay of treatment in patient with RA or psoriatic arthritis.  Methotrexate is a folic acid antagonist that inhibits cytokine production and purine nucleotide biosynthesis leading to immunosuppressive and anti-inflammatory effects.  If methotrexate alone is not working, We add sulfasalazine, If it still doesn’t work, we give triple therapy which are:-  Methotrexate  Sulfasalazine  Hydroxychloroquine/ Leflunomide  If triple therapy doesn’t work, we Start INF inhibitor. REFERENCE KD Tripathi Essentials of Medical Pharmacology 8th edition/page : 734 Q. 91 Tolvaptan is used for:- 1. SIADH CORRECT 2. Central DI 3. Von willebrand disease 4. Catecholamine resistant Shock SOLUTION VAPTAN are Vasopressin antagonist it is given to patient who have excess ADH (SIADH). For patient with central DI and von Willebrand disease- Desmopressin. New indication of Tolvaptan is Autosomal dominant polycystic kidney Disease (ADPKD) REFERENCE KD Tripathi Essentials of Medical Pharmacology 8th edition/page: 639
  • 84. Q. 92 Mechanism of action of Oseltamivir and Zanamivir is:- 1. DNA polymerase inhibition 2. Inhibition of viral mRNA 3. Blocking viral uncoating 4. Neuraminidase inhibition CORRECT SOLUTION Oseltamivir acts by inhibiting influenza virus neuraminidase enzyme which is needed for release of progeny virions from the infected cell. Other neuraminidase inhibitor includes Zanamivir & Peramivir. These are used against Flu Virus (Bird flu/swine Flu). Drugs like Amantadine and Rimantadine are viral uncoating inhibitors. Route of administration of neuraminidase inhibitors in influenza: Zanamivir: Inhalational Oseltamivir: Oral Peramivir: Intravenous REFERENCE KD Tripathi Essentials of Medical Pharmacology - 8th Edition, Page No. 853 Q. 93 Buspirone acts on:- 1. 5HT1A CORRECT 2. 5HT1B
  • 85. 3. 5HT2 4. 5HT3 SOLUTION Drugs like : Buspirone Azapirone Ipsapirone Gepirone Are used for anxiety and it acts on 5HT 1A Clozapine has high affinity for D1, D4, 5HT2, X adrenergic receptor. Ondansetron Granisetron Palonosetron Dolasetron 5HT3 receptor Antagonist Palonsetron - Maximum potency REFERENCE KD Tripathi Essentials of Medical Pharmacology 8th edition/page 185 Q. 94 Carbapenem with maximum seizure risk is:- 1. Imipenem CORRECT 2. Meropenem 3. Ertapenem 4. Doripenem SOLUTION
  • 86. Carbapenems are synthetic Beta lactam antibiotic. Imipenem is compounded with cilastatin to protect it from metabolism by renal dehydropeptidase. Cilastatin is a dehydropeptidase inhibitor. High level of imipenem may provoke seizure, however other carbapenem like:-  Meropenem  Ertapenem  Doripenem Are less likely to do so. REFERENCE Lippincott pharmacology 6th edition page 504 Q. 95 Cilastatin is given in combination with Imipenem because:- 1. Cilastatin prevents degradation of Imipenem in kidney CORRECT 2. Cilastatin increase absorption of Imipenem 3. Inhibits the enzymes that digest Imipenem in stomach 4. Reduces side effects of Imipenem SOLUTION Carbapenems are synthetic Beta lactam antibiotic. Imipenem is compounded with cilastatin to protect it from metabolism by renal dehydropeptidase. Cilastatin is a dehydropeptidase inhibitor. High level of imipenem may provoke seizure, however other carbapenem like:-  Meropenem  Ertapenem  Doripenem Are less likely to do REFERENCE KD Tripathi Essentials of Medical Pharmacology 8th edition/page 43
  • 87. Q. 96 Mechanism of action of Triazoles:- 1. Inhibits ergosterol biosynthesis CORRECT 2. Inhibits tubulin 3. Inhibits glucan synthesis 4. Inhibits cell wall synthesis SOLUTION  The azoles act by inhibiting fungal cytochrome P450 enzyme 'lanosterol 14-demethylase' and thus impair ergosterol synthesis.  These are presently the most extensively used antifungal drugs.  Mechanism of action of other antifungal drugs: o Polyene (Amphotericin B/Nystatin): Binds to ergosterol and creates pores. o 5 Flucytosine : DNA polymerase inhibitor. o Allylamine (Terbinafine, Butenafine, Naftifine): Inhibit enzyme squalene epoxidase. o Griseofulvin : Act mitotic spindle o Echinocandin (Caspofungin): Inhibit Beta 1-3 REFERENCE KD Tripathi Essentials of Medical Pharmacology - 8th Edition, Page No. 843 Q. 97 Identify the false statement regarding Teduglutide is:- 1. It is Used for short-Bowel Syndrome 2. It is a GLP-2 agonist 3. It it a pancreatic enzyme CORRECT 4. It is a gut Hormone SOLUTION
  • 88. Drug ending with GLUTIDE :- Glucagon like peptide (GLP) receptor agonists.  GLP-1 agonist --> Exenatide/ Liraglutide  GLP-2 agonist --> Teduglutide GLP-1:-  Increase Insulin release  Decrease gastric motility. GLP-2 :-  Used for Short Bowel Syndrome to increase gastric emptying time.  It is the hormone released by GI Tract not pancreas. REFERENCE Goodman & Gilman’s the pharmacological basis of therapeutics- 13th edition/page- 939 Q. 98 Hypertension and pulmonary edema associated with scorpion sting is managed by :- 1. Carvedilol 2. Prazosin CORRECT 3. Spironolactone 4. Phentolamine SOLUTION  Scorpion sting cause excessive stimulation of alpha 1 receptor. Of the above mentioned drug:- Prazosin acts by blocking alpha 1 receptor. Therefore drug of choice for Scorpion sting is Prazosin.  Carvedilol : Non selective β antagonist that also blocks alpha adrenoreceptor  Spironolactone: Direct antagonist of aldosterone, thereby preventing salt retention, myocardial hypertrophy and Hypokalemia.  Phentolamine : produce a competitive block of alpha1 and alpha 2 receptor. REFERENCE Lippincott pharmacology 6th edition page 96/143
  • 89. Q. 99 Mannitol is used in the management of:- 1. Congestive cardiac failure 2. Acute congestive glaucoma CORRECT 3. Acute renal failure 4. Pulmonary edema SOLUTION Mannitol:- Uses:- Cerebral edema Acute Congestive Glaucoma CI:- Cerebral Edema due to Haemorrhage Pulmonary Edema Acute Renal Failure REFERENCE Lippincott pharmacology 6th edition page 265 Q. 100 Drug inhibiting granulocyte migration is:- 1. Montelukast 2. Cromoglycate 3. Colchicine CORRECT 4. Felbamate SOLUTION Colchicine : used for the treatment of acute gouty attack. Binds to tubulin, a microtubular
  • 90. protein, causing it’s depolymerization. This disrupts cellular function such as inability of granulocyte, thus deceasing their migration into affected area. Furthermore, colchicine block cell division by binding to mitotic spindle. Montelukast/ Zafirlukast : Blocks leukotriene receptor. Felbamate : Broad spectrum of anticonvulsant action with multiple mechanism including blocking of voltage depended sodium channel, blocking calcium channel and potentiating GABA action. It is reserved for use in refractory epilepsies particularly Lennox- Gastaut Syndrome because of the risk of aplastic anemia and hepatic failure. Sodium cromoglycate :- Inhibit degranulation of mast cell (Mast cell Stabilizer). REFERENCE Lippincott pharmacology 6th edition page 477 Q. 101 Drug capable of causing ocular hypotension with apnoea in infants is:- 1. Acetazolamide 2. Latanoprost 3. Brimonidine CORRECT 4. Apraclonidine SOLUTION Brimonidine is contraindicated in infant <2 yo because it cause brain suppression leading to apnea. Apraclonidine and Brimonidine both are topical alpha adrenergic agonist which decrease aqueous humor production and increase aqeous outflow. Apraclonidine cause lid retraction. Latanoprost :- PGF2alpha  P- Pigmentation of Iris  G-Growth of eyelash (Hypertrichosis)  F2x-fluid in macula (Macular edema) REFERENCE
  • 91. Lippincott pharmacology 6th edition page 101 Q. 102 Inhalational anesthetic with highest respiratory irritation is:- 1. Desflurane CORRECT 2. Nitrous oxide 3. Sevoflurane 4. Halothane SOLUTION  Max respiratory irritation:- Desflurane  MAC of Nitrous Oxide: >104%  Smooth inducing agent, preferred in children : Sevoflurane  Halothane:- Sensitize heart to arrythmogenic agent( Eg:- Ardenaline). Therefore contraindicated in pheochromocytoma REFERENCE KD Tripathi Essentials of Medical Pharmacology 8th edition(page-413) Q. 103 Which of the following drug is used for smoking cessation? 1. Varenicline CORRECT 2. Acamprostate 3. Nalmefene 4. Gabapentim SOLUTION DOC for smoking cessation : Varenicline
  • 92. Nicotine patch and gum are also used. Bupropion is also used in smoking cessation. Varenicline: is partial agonist at neuronal nicotinic acetylcholine receptor in CNS. Patient taking varenicline should be monitored for suicidal thoughts, vivid nightmares and mood changes. Acamprosate : agent used in alcohol dependent treatment programs with an as yet poorly understood mechanism of action. Gabapentin: analog of GABA: Does not act at GABA receptor. It’s precise mechanism is not known. It is approved as adjunct therapy for focal seizure and treatment of post herpetic neuralgic. Nalmefene :- is opioid antagonist use for opioid poisoning REFERENCE Lippincott pharmacology 6th edition page 215/218 Q. 104 Which of the following statements about pegfilgrastim is true? 1. It decrease the risk of neutropenia in patients on anti cancer therapy CORRECT 2. It is a short acting drug 3. It can be given orally 4. It is recombinant GM CSF SOLUTION PEGFILGRASTIM:-  Long acting granulocyte colony stimulating factor (GCSF).  It is used along with anticancer drug to decrease neutropenia.  Commonly given by SC route. Drug ending with GRASTIM are granulocytes stimulator. Drug ending with GRAMOSTIM are granulocyte monocyte stimulator. Eg:- SARGRAMOSTIM. REFERENCE Katzung's Basic & Clinical Pharmacology- 14th edition/page 603
  • 93. Q. 105 Agent used for eliciting diagnostic differentiation of Myasthenia Gravis from Cholinergic crisis is:- 1. Ecothiophate 2. Edrophonium CORRECT 3. Neostigmine 4. Ambenonium SOLUTION - Myasthenia Gravis  In Myasthenia Gravis there are auto antibodies directed against the Nm receptors.  This causes weakness and easy fatigability , with recovery after rest. - Edrophonium which is Acetyl cholinesterase inhibitor if given to patient with MG , a short- lasting improvement in the strength of affected muscles occurs only in myasthenia gravis and not in other muscular dystrophies. This is known as Ameliorative test. - Whereas, if the patient is of Cholinergic crisis edrophonium will further worsens the symptoms. So it can be used to differentiate MG and cholinergic crisis. REFERENCE KD Tripathi Essentials of Medical Pharmacology 8th edition/page 121 Q. 106 Which of the following anesthetic agent is not painful on intravenous administration? 1. Ketamine CORRECT 2. Propofol 3. Etomidate
  • 94. 4. Methohexital SOLUTION Ketamine: Short acting, non barbiturate anaesthetic induces a dissociated state in which the patient is unconcious and does not feel pain. This dissociative anaesthesia provide sedation, amnesia and immobility. Not painful on IV administration Propofol: potent anaesthetic but a weak analgesic . Maximum pain on injection. Etomidate:- Hypnotic agent used to induce anaesthetic but lacks analgesic activity. Usually only used for patient with coronary artery diseases or cardiovascular dysfunction. Methohexital:- Barbiturate derivative, short acting and has rapid onset of action. REFERENCE Lippincott pharmacology 6th edition page 171/178 Q. 107 Which IV anesthetic not cause cardiac depression? 1. Etomidate CORRECT 2. Propofol 3. Methohexital 4. Thiopentone SOLUTION Etomidate:-  Preferred inducing agent in cardiovascular surgery  Major disadvantage is adrenal suppression Thiopental:-  Ultrashort acting barbiturate with high lipid solubility.  Potent anaesthetic but weak analgesic. Methohexital & Thiopental - Histamine release
  • 95. REFERENCE KD Tripathi Essentials of Medical Pharmacology 8th edition(page409) Q. 108 Mechanism of action of curare like drugs is:- 1. Competitive, Non-depolarizing block at the Nm cholinergic receptors CORRECT 2. Noncompetitive, Non depolarizing block at the Nm cholinergic receptors 3. Competitive, depolarizing block at the Nm cholinergic receptors 4. Noncompetitive, depolarizing block at the Nm cholinergic receptors SOLUTION Neuromuscular Blocker Deploarising Non Competitive Non - Dep/Competitive Succinyl Choline Atracurium Cis-atracurium Mivacurium Pancuronium Pipecuronium Vacuronium REFERENCE Lippincott pharmacology 6th edition page 70 Q. 109 Reason for preferring cis-Atracurium over Atracurium is:- 1. Faster acting than Atracurium 2. Lesser provocation of histamine release CORRECT 3. Shorter action than atracurium 4. Does not undergo Hoffman elimination
  • 96. SOLUTION Cis-atracurium and Atracurium both are NMJ blocking agent. Cis atracurium has same pharmacokinetic properties as atracurium. Atracurium release histamine and is metabolized to Laudonasine, which can provoke seizures. Both undergo Hoffman elimination. REFERENCE Lippincott pharmacology 6th edition Q. 110 Drug of choice for Digoxin induced Ventricular Tachycardia:- 1. Propranolol 2. Dilitiazem 3. Verapamil 4. Lignocaine CORRECT SOLUTION Lignocaine:- DOC in Digoxin Induced Ventricular Tachycardia. DOC in post Mi Ventricular Tachycardia. It is class IB Ant arrythmia drug. Propranolol:- Non selective antagonist, having both negative inotropic and chronotropic effect It doesnot reduce blood pressure in people with (N) Blood pressure. It lower BP in Hypertension. Verapamil and diltiazem should be avoided in patient with heart failure or with AV block due to their negative inotropic and dromotropic REFERENCE Lippincott pharmacology 6th edition page 69 Q. 111
  • 97. Which of the following is a contraindication to the use of Beta Blockers:- 1. Glaucoma 2. Tachycardia 3. Bronchial asthma CORRECT 4. Hypertension SOLUTION • Contraindication of Blocker:- 1. Asthma 2. AV Block 3. Acute CHF 4. Diabetes Mellitus • ß Blocker uses:-  Hypertension  Migraine  Hyperthyroidism  Angina pretoirs  MI REFERENCE Lippincott pharmacology 6th edition page 99 Q. 112 Screening test, following genetic counselling, in a family with familial adenomatous polyposis is:- 1. APC gene testing CORRECT 2. Occult blood in stool 3. Flexible sigmoidoscopy 4. Colonoscopy
  • 98. Q. 113 Identify the nerve supply of the marked muscle:- 1. Radial nerve 2. Median nerve CORRECT 3. Ulnar nerve 4. Anterior interosseous nerve SOLUTION • Radial nerve → extensor compartment • Median nerve → 1st and 2nd lumbricals • Ulnar nerve → 3rd and 4th lumbricals • Anterior interosseous nerve → Flexor digitorum profundus (lateral half), Flexor pollicis longus, Pronator quadratus REFERENCE Gray’s anatomy,2nd south Asian edition, pg. 204-205 Q. 114 Tenosynovitis of flexor tendon. What is the correct option? 1. The affected finger is extended at all joints
  • 99. 2. Every case is surgically managed . 3. Tenderness along tendon sheath . CORRECT 4. Patient present with minimal pain SOLUTION Tenosynovitis  Very painful condition  The cardinal signs of flexor sheath infection are: o Finger held in slight flexion o Fusiform swelling of the affected digit o Tenderness along the flexor tendon sheath o Pain with passive extension of the digit  Treatment- o Conservative management o Surgery (closed method) REFERENCE Apley & Solomon’s, 10th edition: pg.449 Q. 115 12 years male came with swelling of lower end tibia which is surrounded by rim of reactive bone. What is most likely diagnosis? 1. GCT 2. Brodie's Abscess CORRECT 3. Hyper PTH
  • 100. 4. Osteomyelitis SOLUTION • GCT – occurs around 20-40 years of age • Hyperparathyroidism – typically in a female around 30 years of age; presents with lytic lesion • Brodie’s abscess – swelling in the lower end of tibia and radiological evidence of cavity surrounded by a halo of sclerosis (rim of a reactive bone) point towards brodie’s abscess. REFERENCE Apley & Solomon’s, 10th edition: pg.39 Q. 116 A patient with GCT which of the following is false:- 1. Epiphyseo-metaphyseal location 2. Eccentric
  • 101. 3. Defined margins 4. Chemotherapy is the mainstay of treatment CORRECT SOLUTION • GCT occurs in epiphyseao-metaphyseal area. • It is eccentric in location and goes till the joint. • It is a locally aggressive tumor with defined margins. • The treatment of choice is extended curettage with autograft. REFERENCE Apley & Solomon’s, 10th edition: pg.198-199 Q. 117 75 yrs female has chronic backache, x-ray spine is shown. What is the most likely diagnosis:-
  • 102. 1. Pott's spine 2. Osteoporosis CORRECT 3. Spondylolisthesis 4. Spondylodiscitis SOLUTION • Spondylodiscitis – Inflammation of disc along with paradiscal margin and shows slight narrowing the disc space at first in X-rays. It is a very painful condition. • Pott’s spine – occurs in paradiscal area. X-ray shows narrowing of the disc space and erosion of adjacent vertebral bodies.
  • 103. • Spondylolisthesis – Slipping of one vertebra into other. Oblique X- ray view demonstrates “Scotty Dog Neck” which is pathognomonic Osteoporosis – Patient’s age and the chronic condition points towards osteoporosis. The given X-ray shows Cod fish / fish mouth vertebrae which is characteristic of osteoporosis. REFERENCE Apley & Solomon’s, 10th edition: pg.480-481,522 Q. 118 A 70 yrs male patient has single well defined lytic lesion of skull. The patient had no other complaint and urine examination had no abnormality. What is the most likely diagnosis? 1. L.C.H CORRECT 2. Localized myeloproliferative disorder 3. Generalized myeloproliferative disorder 4. Tumor of osteoblasts SOLUTION
  • 104. Langerhans’s cell histiocytosis  X-ray- well-demarcated lytic lesion, “hole within a hole”.  M/E - “Tennis-racket” inclusion bodies and Birbeck granules Plasmacytoma  Localized myeloproliferative disorder, does not present with lytic lesions in the skull. Generalized myeloproliferative disorder is multiple myeloma which presents with characteristic “punched-out” lytic lesions in the skull. Serum or urinary electrophoresis measures immunoglobulins over-produced by the plasma cells. Lytic lesions of skull:  M- Metastasis  E- EG(eosinophilic granuloma)  L- LCH/Lymphoma  T-T.B  H-Hyperparathyroidism  O- Osteomyelitis  R- RT(Radiotherapy)  M- MM(Multiple myeloma)  E- Epidermoid REFERENCE Apley & Solomon’s, 10th edition: pg.250,214 Q. 119 Foot drop is caused due to injury to?
  • 105. 1. Common peroneal nerve CORRECT 2. Tibial Nerve 3. Femoral Nerve 4. Obturator Nerve SOLUTION Foot drop = it occurs due to injury to common peroneal nerve which usually gets injured at the neck of fibula. Femoral nerve injury – unable to extend the knee actively. Tibial nerve injury – unable to plantar flex the ankle or foot and no sensation over the sole and part of calf. REFERENCE Apley & Solomon’s, 10th edition: pg.299-302 Q. 120 Image of dorsum of hand, which nerve gives sensory supply to this region:-
  • 106. 1. PIN 2. Radial CORRECT 3. Median 4. Ulnar SOLUTION REFERENCE Gray’s anatomy,2nd south Asian edition, pg.44
  • 107. Q. 121 Scaphoid fracture which area has maximum chances of AVN/Non-union/Malunion:- 1. Proximal 1/3 CORRECT 2. Middle 1/3 3. Distal 1/3 4. Scaphoid Tubercle fractrue SOLUTION • Scaphoid has retrograded blood supply (distal to proximal) hence the distal pole is highly vascularized. • Due to fracture at the waist of scaphoid, blood supply to proximal pole is compromised. Hence proximal 1/3rd has the maximum chance to undergo AVN/non-union or malunion. REFERENCE Apley & Solomon’s, 10th edition: pg.806 Q. 122 Painful arc syndrome which movement is painful:- 1. Initial abduction 2. Terminal abduction
  • 108. 3. Mid abduction CORRECT 4. Full range of abduction SOLUTION • Painful arc syndrome is subacute tendinitis of the rotator cuff tendons due to vascular congestion. • Treatment is usually rest +physiotherapy + NSAIDS. If not treated, local steroids and surgery can be done. • Pain is aggravated as arm transverses an arc between 60 and 120 degrees. REFERENCE Apley & Solomon’s, 10th edition: pg.357-358 Q. 123 The most likely diagnosis for the tumor at upper end of tibia is:-
  • 109. 1. GCT CORRECT 2. UBC 3. ABC 4. CB SOLUTION • Giant cell tumor – It is the only tumor to involve the joint. It is seen in 20-40 years of age (after skeletal maturity). It occurs in epiphysis (mainly) and extends into metaphysis, so called as epiphyseal-metaphysis tumor • Unicameral bone cyst –Unilocular cystic bone cavity filled with serosanguinous fluid. X-ray shows fallen leaf sign. It often abuts but does not cross the physis. • Aneurysmal bone cyst –Blood-filled cystic spaces. X-ray shows expansile, lytic cystic lesions and MRI shows fluid-fluid levels within the lesion. • Chondroblastoma – presence of “chicken-wire” calcification is pathognomonic. REFERENCE Apley & Solomon’s, 10th edition: pg.195-202 Q. 124 Left-Right movement of skull occurs at:- 1. Atlento occipital joint 2. Atlanto-axial joint CORRECT 3. C2-C3 4. C6-C7 SOLUTION  Atlanto-occipital joint – flexion and extension of neck."yes joint"  Atlanto-axial joint (C1 -C2) movements -side to side movements of head (“no joint”) i.e left and right movements of skull.
  • 110.  Atlanto-occipital joint Atlanto-axial joint (C1 -C2) movements REFERENCE Apley & Solomon’s, 10th edition: pg.456 Q. 125 Degloving injury occurs at:- 1. Skin 2. Skin + Subcutaneous tissue CORRECT 3. Skin + Fascia + Subcutaneous tissue 4. Everything above from bone SOLUTION Degloving injury – it is the separation of skin and subcutaneous tissues from the underlying bone, muscle or vasculature. Degloving occurs when the skin and subcutaneous fat are stripped by avulsion from the underlying fascia, leaving neuro vascular structures, tendon or bone exposed.
  • 111. REFERENCE Bailey & Love,27th edition, pg.27 Q. 126 14 years old child with Naive Rheumatoid Arthritis patient (image of both hands give) what is treatment? 1. Dmards after initial 3 months of NSAIDs 2. Only NSAID 3. Dmards with short course of steroids CORRECT 4. Monotherapy with TNF drugs SOLUTION • Treatment should be aimed at controlling the inflammation as rapidly as possible. This is likely to require the use of corticosteroids for their rapid onset. In addition, disease-modifying antirheumatic drugs (DMARDS) should be started at this time. • Control of pain and stiffness with NSAIDS may be needed. • If there is no satisfactory response to DMARDS, it’s wise to progress to TNF therapies such as Infliximab. REFERENCE Apley & Solomon’s, 10th edition: pg.71 Q. 127 Caspase involved in activation of IL_1 is which of the following?
  • 112. 1. Caspase 1 CORRECT 2. Caspase 3 3. Caspase 5 4. Caspase 8 SOLUTION Pyroptosis :-  Example of process in which injury is initiated because of exposure to microbes. Whenever there is exposure to microbes there is activation of enzyme caspase 1 and caspase 11.  Caspase 1 convert inactive form of IL 1 to active form of IL1. As IL1 has central action, it enters CNS and results in development of fever.  Caspase-1 along with closely related caspase-11 also cause death of the infected cell. →Caspase 3, a typical executioner caspase that cleaves DNA and other substrates to cause cell death. REFERENCE Robbins and Cotran Pathology 10th E Page 302 Q. 128 Which of the following is positive in Follicular lymphoma? 1. Bcl 2 CORRECT 2. Bcl 6 3. Bcl 1 4. Bcl 10 SOLUTION Follicular lymphoma likely arises from germinal center B cells and is strongly associated with chromosomal translocations involving BCL2. Its hallmark is a (14;18) translocation that juxtaposes the IGH locus on chromosome 14 and the BCL2 locus on chromosome 18. The t(14;18) is seen in up to 90% of follicular lymphomas, and leads to overexpression of BCL2. BCL2 antagonizes apoptosis and promotes the survival of follicular lymphoma cells. Notably, while normal germinal centers contain numerous B cells undergoing apoptosis, follicular
  • 113. lymphoma is characteristically devoid of apoptotic cells. BCL 10 : associated with extra nodal marginal zone lymphoma The t(14;18)(q32;q21) and t(1;14) (p22;q32) translocations cause increased expression of intact MALT1 and BCL-10 protein respectively. This translocation is associated with gastric MALToma. Each of the three translocations has the same net effect, the constitutive activation of NF-κB, a transcription factor that promotes B-cell growth and survival. Antigen dependent activation of NF-κB in normal B and T cells. requires both BCL-10 and MLT, which work together in a pathway downstream of the B- and T-cell antigen receptors. Thus, H. pylori–induced inflammation may trigger NF-κB activation through the MLT/BCL-10 pathway in MALTomas that lack these translocations. Removal of this stimulus may explain why these tumors tend to respond to H. pylori eradication. In contrast, NF-κB is constitutively active in tumors bearing translocations involving MLT or BCL10, and H. pylori treatment is ineffective. BCL 6 : associated with diffuse large b cell lymphoma(DLBCL). DLBCL is most common subtype of non Hodgkin lymphoma globally. BCL 1 : also called cyclin D1 and is associated with Mantle Cell Lymphoma REFERENCE Robbins and Cotran Pathology : Page 773 Q. 129 Vitamin A is stored in:- 1. Hepatocyte 2. Ito cell CORRECT 3. Hepatic endothelial cell 4. Kupffer cell SOLUTION Vitamin A is stored in Ito cell The principal cell type involved in scar deposition is the hepatic stellate cell.  In its quiescent form, it is a lipid (vitamin A) storing cell. However, in several forms of acute and chronic injury, the stellate cells( ito) can become activated and are converted
  • 114. into highly fibrogenic myofibroblasts. REFERENCE Robbins and Cotran Pathology 9th ed: Pg 823 Q. 130 Glanzmann thromasthenia is due to defect in:- 1. Gp IIb/IIIa CORRECT 2. Gp Ib-IX 3. CD68 4. Von Willebrand factor SOLUTION The conformational change in glycoprotein IIb/IIIa that occurs with platelet activation allows binding of fibrinogen, a large bivalent plasma polypeptide that forms bridges between adjacent platelets, leading to their aggregation. Predictably, inherited deficiency of GpIIb-IIIa results in a bleeding disorder called Glanzmann thrombasthenia. Bernard-Soulier syndrome illustrates the consequences of defective adhesion of platelets to subendothelial matrix. Bernard-Soulier syndrome is caused by an inherited deficiency of the platelet membrane glycoprotein Complex Ib-IX. This glycoprotein is a receptor for vWF and is essential for normal platelet adhesion to the subendothelial extracellular matrix. Affected patients have a variable, often severe, bleeding tendency. Factor VIII and vWF are encoded by separate genes and are synthesized in different cells. Factor VIII is an essential cofactor of factor IX, which converts factor X to factor Xa. It is made in several tissues; sinusoidal endothelial cells and Kupffer cells in the liver seem to be major sources. Type 1 and type 3 von Willebrand disease are associated with quantitative defects in vWF. Type 1, an autosomal dominant disorder characterized by a mild to moderate vWF deficiency, accounts for about 70% of all cases. Type 2 von Willebrand disease is characterized by qualitative defects in vWF.
  • 115. REFERENCE Robbins and Cotran Pathology Page : 662 Q. 131 Which of the following is an antiapoptotic gene? 1. Bcl2 CORRECT 2. Bcl- XS 3. BAX 4. BAD SOLUTION Antiapoptotic genes are:  BCL-2  BCL-XL  MCL-1  o These genes possess four BH domains (called BH1-4). o These proteins reside in the outer mitochondrial membranes as well as the cytosol and ER membranes. Proapoptotic genes are:  BAX
  • 116.  BAK  o They also have four BH domains. o Upon activation, BAX and BAK oligomerize within the outer mitochondrial protein and promote mitochondrial outer membrane permeability. Sensor genes are:  BAD  BIM  BID  Puma  Noxa  o They contain only one BH domain, the third of the four BH domains, and hence are sometimes called BH3-only proteins. o These proteins act as sensors of cellular stress and damage, and regulate the balance between the other two groups, thus acting as arbiters of apoptosis. REFERENCE Robbins and Cotran Pathology 9th edition pg 52 Q. 132 Which of the following is true about alpha 1 antitrypsin? 1. Inhibits elastase CORRECT 2. Inhibits trypsinogen activation in pancreas 3. Inhibits trypsin activating protease 4. Inhibits chymotrypsin SOLUTION Alpha 1 antitrypsin : anti elastase protein coded by gene PiMM gene. Any mutation in this gene lead to abnormal formation of alpha 1 antitrypsin protein which leads to
  • 117. 1. Storage of this abnormal enzyme inside hepatocyte instead of it’s secretion resulting in manifestation of micronodular cirrhosis of liver and 2. Pan acinar emphysema in basal part of lung. REFERENCE Robbin 10th edition Page : 498 Q. 133 Which of the following is autosomal recessive disorder? 1. Huntington's chorea 2. Neurofibromatosis 1 3. Marfan's syndrome 4. Sickle cell anemia CORRECT SOLUTION Autosomal Dominant (A Very DOMINANT Hereditary Family )  Achondroplasia/ADPKD  Von Hipple Lindau disease  Dystrophia myotonica  Osteogenesis imperfecta  Marfan Syndrome  Intermittent porphyria  NF 1(von reckinghausen disease)  Achondroplasia  NF 2  Tuberous Sclerosis  Huntington Disease  Familial hypercholesterolemia Autosomal Recessive (ABCDEFGH’S)  Albinism, Ataxia Telengiactasia, Alkaptonuria  Beta thalassemia  Cystic Fibrosis  Dubin Johnson  Emphysema(alpha 1 antitrypsin deficiency)  Friedreich Ataxia  Galactosemia  Hemochromatosis  Sickle cell anemia REFERENCE ROBBINS AND CORTON 10e/172
  • 118. Q. 134 Fish mouth stenosis in rheumatic heart disease is due to which of the following mechanisms? 1. Calcification and fibrosis bridging across valvular commissures CORRECT 2. Fibrinoid necrosis 3. Acute inflammation leading to valvular damage 4. Myxomatous degeneration of the valve SOLUTION Fish mouth stenosis in RHD is manifestation of Chronic RHD involving mitral valve which is due to calcification and fibrosis bridging across valvular commissures. RHD is virtually the only cause of mitral stenosis. Acute and chronic rheumatic heart disease. A) Acute rheumatic mitral valvulitis superimposed on chronic rheumatic heart disease. Small vegetations (verrucae) are visible along the line of closure of the mitral valve leaflet (arrows). Previous episodes of rheumatic valvulitis have caused fibrous thickening and fusion of the chordae tendineae. B) Microscopic appearance of an Aschoff body in a patient with acute rheumatic carditis. The myocardium exhibits a circumscribed nodule of mixed mononuclear inflammatory cells with associated necrosis; within the inflammation, large activated macrophages show prominent nucleoli, as well as chromatin condensed into long, wavy ribbons (caterpillar cells; arrows).
  • 119. C/D). Mitral stenosis with diffuse fibrous thickening and distortion of the valve leaflets and commissural fusion (arrows, C), and thickening of the chordae tendineae (D). Note neovascularization of anterior mitral leaflet (arrow, D). E). Surgically resected specimen of rheumatic aortic stenosis, demonstrating thickening and distortion of the cusps with commissural fusion. Comparison of the four major forms of vegetative endocarditis. The rheumatic fever phase of rheumatic heart disease (RHD) is marked by small, warty vegetations along the lines of closure of the valve leaflets. Infective endocarditis (IE) is characterized by large, irregular masses on the valve cusps that can extend onto the chordae. Nonbacterial thrombotic endocarditis (NBTE) typically exhibits small, bland vegetations, usually attached at the line of closure. One or many may be present. Libman-Sacks endocarditis (LSE) has small or medium-sized vegetations on either or both sides of the valve leaflets. REFERENCE Robbins and Cotran Pathology 9th edition Page : 558 Q. 135 Which of the following feature distinguishes Crohn's disease from Ulcerative colitis ? 1. Transmural involvement CORRECT 2. Presence of polyps 3. Mucosal edema 4. Lymphocyte infiltrate SOLUTION → Crohn's disease is associated with transmural inflammation whereas only superficial layers [mucosa, submucosa] are involved in ulcerative colitis. Feature Crohn Disease Ulcerative Colitis Macroscopic
  • 120. Bowel region involved Any site of GIT Colon only Distribution Skip lesions Diffuse Stricture Yes Rare Wall appearance Thick Thin Microscopic Inflammation Transmural Limited to mucosa, submucosa Pseudopolyps may/ may not be present Markedly present Ulcers Deep, knife-like Superficial, broad-based Lymphoid reaction Marked Moderate Fibrosis Marked Mild to none Serositis Marked Mild to none Granulomas Yes ( 35%) [non-caseating] No Fistulae/sinuses Yes No Clinical Perianal fistula Yes (in colonic disease) No Fat/vitamin malabsorption Yes No Malignant potential With colonic involvment yes Yes Recurrence after surgery Common No Toxic megacolon No Yes All feature may not be present in a single case. REFERENCE Robbins 10th edition Page : 621-625 Q. 136 Which of the following is a cause of intravascular hemolysis? 1. Warm type autoimmune hemolytic anemia 2. PNH CORRECT 3. Cold agglutinin disease 4. Hereditary spherocytosis SOLUTION → Causes of intravascular hemolysis are: